Podcast
Questions and Answers
Which of the following is the most common orthopedic injury, accounting for 40% of all athletic injuries?
Which of the following is the most common orthopedic injury, accounting for 40% of all athletic injuries?
- Meniscus tear
- ACL tear
- Wrist fracture
- Ankle sprain (correct)
What is the primary difference between a strain and a sprain?
What is the primary difference between a strain and a sprain?
- A strain is always more severe than a sprain.
- A strain involves muscles or tendons, while a sprain involves ligaments. (correct)
- A sprain is an injury that only occurs in the ankle.
- A strain involves ligaments, while a sprain involves tendons.
Which of the following is a characteristic of a second-degree or moderate strain?
Which of the following is a characteristic of a second-degree or moderate strain?
- Severe muscle spasms, extensive inflammation, and ecchymosis. (correct)
- Complete rupture of the muscle or tendon.
- Minimal inflammation and pain with unaffected range of motion.
- Ability to ambulate with slight discomfort.
The RICE acronym is commonly used in the treatment of first- and second-degree strains and sprains. What does RICE stand for?
The RICE acronym is commonly used in the treatment of first- and second-degree strains and sprains. What does RICE stand for?
What is a potential complication of strains and sprains that involves excessive pressure within a limited space, leading to neurovascular compromise?
What is a potential complication of strains and sprains that involves excessive pressure within a limited space, leading to neurovascular compromise?
Which of the following chronic bone disorders is NOT associated with an increased risk of pathological fractures?
Which of the following chronic bone disorders is NOT associated with an increased risk of pathological fractures?
What type of fracture involves pieces of bone protruding through the skin, creating an external wound?
What type of fracture involves pieces of bone protruding through the skin, creating an external wound?
Which term describes a fracture in which bone fragments are well approximated within the site of disruption?
Which term describes a fracture in which bone fragments are well approximated within the site of disruption?
What is the primary purpose of using irrigation and debridement in the surgical management of fractures?
What is the primary purpose of using irrigation and debridement in the surgical management of fractures?
Which of the following interventions is most important in preventing muscle atrophy and contractures after definitive fracture treatment?
Which of the following interventions is most important in preventing muscle atrophy and contractures after definitive fracture treatment?
What is the primary goal of traction in fracture treatment?
What is the primary goal of traction in fracture treatment?
Which of the 'six Ps' of compartment syndrome are considered late findings?
Which of the 'six Ps' of compartment syndrome are considered late findings?
What is the primary reason for administering intravenous fluids to a patient with rhabdomyolysis secondary to a crush injury?
What is the primary reason for administering intravenous fluids to a patient with rhabdomyolysis secondary to a crush injury?
Which of the following assessment findings would MOST strongly suggest the presence of a pulmonary embolus in a patient recovering from a fracture?
Which of the following assessment findings would MOST strongly suggest the presence of a pulmonary embolus in a patient recovering from a fracture?
What is the PRIMARY reason for avoiding the use of a pillow under the stump of a lower extremity amputation?
What is the PRIMARY reason for avoiding the use of a pillow under the stump of a lower extremity amputation?
Which of the following is a common symptom of phantom limb pain following an amputation?
Which of the following is a common symptom of phantom limb pain following an amputation?
What is the PRIMARY rationale for wrapping a residual limb with a compression dressing postoperatively?
What is the PRIMARY rationale for wrapping a residual limb with a compression dressing postoperatively?
Which of the following is a risk factor specifically associated with acute meniscus injuries?
Which of the following is a risk factor specifically associated with acute meniscus injuries?
A patient reports experiencing a sudden onset of sharp knee pain, followed by an insidious onset of effusion. Which assessment finding is MOST indicative of a meniscal tear?
A patient reports experiencing a sudden onset of sharp knee pain, followed by an insidious onset of effusion. Which assessment finding is MOST indicative of a meniscal tear?
Which of the following MOST accurately describes the purpose of an arthroscopic surgical repair of a meniscus tear?
Which of the following MOST accurately describes the purpose of an arthroscopic surgical repair of a meniscus tear?
Which jobs have a higher occurrence of carpal tunnel syndrome (CTS)?
Which jobs have a higher occurrence of carpal tunnel syndrome (CTS)?
Which test involves having the patient relax their hand in the flexed position for 60 seconds to aid in diagnosing Carpal Tunnel Syndrome?
Which test involves having the patient relax their hand in the flexed position for 60 seconds to aid in diagnosing Carpal Tunnel Syndrome?
What does TENS unit stand for?
What does TENS unit stand for?
Which of the following best describes aseptic nonunion?
Which of the following best describes aseptic nonunion?
When there is a suspected crushing injury on a patient, what is the correct protocol?
When there is a suspected crushing injury on a patient, what is the correct protocol?
What are possible sources of edema that can increase swelling and compress blood vessels?
What are possible sources of edema that can increase swelling and compress blood vessels?
What stimulates increased capillary permeability, causing more blood and fluid to leak into the already restricted compartmental space?
What stimulates increased capillary permeability, causing more blood and fluid to leak into the already restricted compartmental space?
Which of the following can be caused by compartment syndrome?
Which of the following can be caused by compartment syndrome?
What electrolytes are part of multiple electrolyte disorders that are frequently complicated by rhabdomyolysis?
What electrolytes are part of multiple electrolyte disorders that are frequently complicated by rhabdomyolysis?
Which of the following sports is least associated with a high frequency of ankle sprains?
Which of the following sports is least associated with a high frequency of ankle sprains?
A patient reports pain, inflammation, and ecchymosis after a sports injury, but can still move the affected joint with some discomfort. Which degree of sprain is most consistent with these findings?
A patient reports pain, inflammation, and ecchymosis after a sports injury, but can still move the affected joint with some discomfort. Which degree of sprain is most consistent with these findings?
When educating a patient on the use of ice for a mild ankle sprain, which instruction should be included?
When educating a patient on the use of ice for a mild ankle sprain, which instruction should be included?
Which of the following findings would be most concerning and warrant immediate notification of the provider in a patient being treated for a lower extremity strain?
Which of the following findings would be most concerning and warrant immediate notification of the provider in a patient being treated for a lower extremity strain?
An elderly patient is diagnosed with a fragility fracture attributed to decreased bone density. Which of the following chronic conditions would most likely contribute to this type of fracture?
An elderly patient is diagnosed with a fragility fracture attributed to decreased bone density. Which of the following chronic conditions would most likely contribute to this type of fracture?
A patient has a fracture in which a bone fragment is forced inward. Which type of fracture is this?
A patient has a fracture in which a bone fragment is forced inward. Which type of fracture is this?
When obtaining a history from a patient with a suspected fracture, what information is most critical for the nurse to gather regarding the injury?
When obtaining a history from a patient with a suspected fracture, what information is most critical for the nurse to gather regarding the injury?
After a closed reduction for a fractured tibia, the patient is placed in a cast. Which of the following nursing interventions is most important to prevent complications associated with immobilization?
After a closed reduction for a fractured tibia, the patient is placed in a cast. Which of the following nursing interventions is most important to prevent complications associated with immobilization?
A patient with a fractured femur is placed in skeletal traction. What is the primary rationale for this intervention?
A patient with a fractured femur is placed in skeletal traction. What is the primary rationale for this intervention?
A patient recovering from a fractured humerus reports severe pain that is unrelieved by pain medication, along with increasing swelling and numbness in the fingers. Which complication should the nurse suspect?
A patient recovering from a fractured humerus reports severe pain that is unrelieved by pain medication, along with increasing swelling and numbness in the fingers. Which complication should the nurse suspect?
A patient with a crush injury is at risk for rhabdomyolysis. Which of the following laboratory findings would the nurse expect to see in this patient?
A patient with a crush injury is at risk for rhabdomyolysis. Which of the following laboratory findings would the nurse expect to see in this patient?
When caring for a patient with a long bone fracture, the nurse should be vigilant for signs and symptoms of fat embolism syndrome (FES). Which assessment finding is MOST indicative of FES?
When caring for a patient with a long bone fracture, the nurse should be vigilant for signs and symptoms of fat embolism syndrome (FES). Which assessment finding is MOST indicative of FES?
What is the most appropriate nursing intervention to prevent flexion contracture after a patient has a lower extremity amputation?
What is the most appropriate nursing intervention to prevent flexion contracture after a patient has a lower extremity amputation?
A patient who underwent an amputation reports persistent burning pain in the absent limb. Which intervention would be most appropriate for the nurse to suggest?
A patient who underwent an amputation reports persistent burning pain in the absent limb. Which intervention would be most appropriate for the nurse to suggest?
After an above-the-knee amputation, what is the primary purpose of wrapping the residual limb with a figure-eight compression bandage?
After an above-the-knee amputation, what is the primary purpose of wrapping the residual limb with a figure-eight compression bandage?
Which of the following factors contributes most significantly to the occurrence of acute meniscus injuries?
Which of the following factors contributes most significantly to the occurrence of acute meniscus injuries?
A patient diagnosed with a meniscal tear is scheduled for arthroscopic surgery. What should the nurse explain as the primary goal of this procedure?
A patient diagnosed with a meniscal tear is scheduled for arthroscopic surgery. What should the nurse explain as the primary goal of this procedure?
Which of the following occupations carries the highest risk for developing carpal tunnel syndrome?
Which of the following occupations carries the highest risk for developing carpal tunnel syndrome?
The Phalen's test is performed to assess for carpal tunnel syndrome. A positive test is indicated by:
The Phalen's test is performed to assess for carpal tunnel syndrome. A positive test is indicated by:
Which of the following treatments is considered a nonoperative measure for managing carpal tunnel syndrome?
Which of the following treatments is considered a nonoperative measure for managing carpal tunnel syndrome?
Which of the following best describes the initial stage of bone healing following a fracture?
Which of the following best describes the initial stage of bone healing following a fracture?
A patient has a fracture that has failed to heal, and cultures reveal no infection. What type of nonunion is this considered?
A patient has a fracture that has failed to heal, and cultures reveal no infection. What type of nonunion is this considered?
What is the primary rationale for administering intravenous fluids to a patient diagnosed with rhabdomyolysis secondary to a crush injury?
What is the primary rationale for administering intravenous fluids to a patient diagnosed with rhabdomyolysis secondary to a crush injury?
A patient with a tibial fracture reports increased pain and pressure in the lower leg despite receiving pain medication. On assessment, the nurse notes pallor and decreased sensation in the foot. What is the priority nursing intervention?
A patient with a tibial fracture reports increased pain and pressure in the lower leg despite receiving pain medication. On assessment, the nurse notes pallor and decreased sensation in the foot. What is the priority nursing intervention?
Which of the following complications is most likely to be associated with fractures in the pelvis due to its high vascularity?
Which of the following complications is most likely to be associated with fractures in the pelvis due to its high vascularity?
Which laboratory finding would the nurse anticipate in a patient experiencing rhabdomyolysis as a complication of a fracture?
Which laboratory finding would the nurse anticipate in a patient experiencing rhabdomyolysis as a complication of a fracture?
Which of the following is a key nursing intervention in preventing complications associated with immobilization devices such as casts and splints?
Which of the following is a key nursing intervention in preventing complications associated with immobilization devices such as casts and splints?
Which of the following is a late sign of compartment syndrome that a nurse should be aware of when assessing a patient with a fracture?
Which of the following is a late sign of compartment syndrome that a nurse should be aware of when assessing a patient with a fracture?
Why are women at a higher risk for ACL tears compared to men?
Why are women at a higher risk for ACL tears compared to men?
A football player experiences a twisting injury to the knee that causes immediate, severe pain and the inability to bear weight. Which type of injury is most likely?
A football player experiences a twisting injury to the knee that causes immediate, severe pain and the inability to bear weight. Which type of injury is most likely?
Following a knee injury, a patient has pain, mild edema, and no loss of joint function, although ambulation is slightly uncomfortable. How would this injury be classified?
Following a knee injury, a patient has pain, mild edema, and no loss of joint function, although ambulation is slightly uncomfortable. How would this injury be classified?
A patient is diagnosed with a third-degree strain of the hamstring muscle. What treatment approach is most likely?
A patient is diagnosed with a third-degree strain of the hamstring muscle. What treatment approach is most likely?
What is the rationale behind the recommendation to apply ice to a sprain?
What is the rationale behind the recommendation to apply ice to a sprain?
How does compression help in the management of sprains and strains?
How does compression help in the management of sprains and strains?
What is the rationale for elevating an injured extremity in the treatment of sprains and strains?
What is the rationale for elevating an injured extremity in the treatment of sprains and strains?
A patient recovering from a second-degree ankle sprain is prescribed exercise therapy. What is the primary focus of these exercises?
A patient recovering from a second-degree ankle sprain is prescribed exercise therapy. What is the primary focus of these exercises?
What potential long-term complication can arise from recurrent strains and sprains?
What potential long-term complication can arise from recurrent strains and sprains?
Which of the following is the initial step in bone healing?
Which of the following is the initial step in bone healing?
An elderly patient has a Colles' fracture as a result of a fall. What underlying condition is most likely to have contributed to this fracture?
An elderly patient has a Colles' fracture as a result of a fall. What underlying condition is most likely to have contributed to this fracture?
A patient has a fracture in which the bone is broken into multiple fragments. How would this type of fracture be described?
A patient has a fracture in which the bone is broken into multiple fragments. How would this type of fracture be described?
A child falls from a tree and sustains a fracture in which one side of the bone is bent and the other is fractured. What type of fracture is this?
A child falls from a tree and sustains a fracture in which one side of the bone is bent and the other is fractured. What type of fracture is this?
How is a fracture in which bone fragments have pierced the skin classified?
How is a fracture in which bone fragments have pierced the skin classified?
In the context of fracture management, what does closed reduction refer to?
In the context of fracture management, what does closed reduction refer to?
What is a primary purpose of using casts or splints in the treatment of fractures?
What is a primary purpose of using casts or splints in the treatment of fractures?
What is the priority nursing action when compartment syndrome is suspected?
What is the priority nursing action when compartment syndrome is suspected?
Why is it essential to manage pain effectively in patients with fractures?
Why is it essential to manage pain effectively in patients with fractures?
What is the rationale for administering anticoagulants to patients with fractures?
What is the rationale for administering anticoagulants to patients with fractures?
What pulmonary complication is the nurse monitoring for when assessing a patient with fat embolism syndrome (FES)?
What pulmonary complication is the nurse monitoring for when assessing a patient with fat embolism syndrome (FES)?
What is the primary goal of administering intravenous fluids to a patient diagnosed with rhabdomyolysis as a complication of a crush injury?
What is the primary goal of administering intravenous fluids to a patient diagnosed with rhabdomyolysis as a complication of a crush injury?
What is a symptom of rhabdomyolysis?
What is a symptom of rhabdomyolysis?
What is a key risk following a traumatic amputation?
What is a key risk following a traumatic amputation?
What class of medication has demonstrated effectiveness in treating phantom limb pain?
What class of medication has demonstrated effectiveness in treating phantom limb pain?
What is the most sensitive and reliable physical finding indicative of a meniscal tear?
What is the most sensitive and reliable physical finding indicative of a meniscal tear?
A patient with a suspected meniscal tear undergoes the McMurray's test. What finding would indicate a positive test for a meniscal injury?
A patient with a suspected meniscal tear undergoes the McMurray's test. What finding would indicate a positive test for a meniscal injury?
Which patient population may benefit from a synovectomy during a carpal tunnel release?
Which patient population may benefit from a synovectomy during a carpal tunnel release?
In assessing a patient who reports symptoms of Carpal Tunnel Syndrome, which of the following would be the MOST important?
In assessing a patient who reports symptoms of Carpal Tunnel Syndrome, which of the following would be the MOST important?
A patient with compartment syndrome requires a fasciotomy. What does this procedure involve?
A patient with compartment syndrome requires a fasciotomy. What does this procedure involve?
Which component of connective tissue facilitates cellular adhesion and the movement of substances between capillaries and cells?
Which component of connective tissue facilitates cellular adhesion and the movement of substances between capillaries and cells?
What is the primary role of osteophytes in the context of osteoarthritis?
What is the primary role of osteophytes in the context of osteoarthritis?
How does synovium contribute to joint function in a healthy joint?
How does synovium contribute to joint function in a healthy joint?
Which of the following is the MOST common initial clinical manifestation in patients with osteoarthritis?
Which of the following is the MOST common initial clinical manifestation in patients with osteoarthritis?
What radiographic finding is MOST indicative of osteoarthritis in the affected joints?
What radiographic finding is MOST indicative of osteoarthritis in the affected joints?
Why is monitoring renal function particularly important when using NSAIDs to treat osteoarthritis?
Why is monitoring renal function particularly important when using NSAIDs to treat osteoarthritis?
What is the rationale for using intra-articular hyaluronan injections in the treatment of osteoarthritis?
What is the rationale for using intra-articular hyaluronan injections in the treatment of osteoarthritis?
What instruction should the nurse provide to a patient taking analgesics and anti-inflammatories for osteoarthritis regarding medication use?
What instruction should the nurse provide to a patient taking analgesics and anti-inflammatories for osteoarthritis regarding medication use?
Which of the following activities is MOST directly linked to promoting joint mobility, muscle strength, cardiac health, and reducing stress in patients with osteoarthritis?
Which of the following activities is MOST directly linked to promoting joint mobility, muscle strength, cardiac health, and reducing stress in patients with osteoarthritis?
What role does joint irrigation and débridement serve in the surgical management of osteoarthritis?
What role does joint irrigation and débridement serve in the surgical management of osteoarthritis?
What is the primary immunological process involved in rheumatoid arthritis (RA) that leads to damage of the synovial tissue?
What is the primary immunological process involved in rheumatoid arthritis (RA) that leads to damage of the synovial tissue?
A patient with rheumatoid arthritis reports increased joint pain, visible swelling, and prolonged morning stiffness. What finding would be MOST indicative of synovitis?
A patient with rheumatoid arthritis reports increased joint pain, visible swelling, and prolonged morning stiffness. What finding would be MOST indicative of synovitis?
To effectively manage rheumatoid arthritis, which laboratory values are MOST important to monitor alongside clinical manifestations?
To effectively manage rheumatoid arthritis, which laboratory values are MOST important to monitor alongside clinical manifestations?
A patient with RA is prescribed methotrexate. What key information should the nurse emphasize regarding alcohol consumption?
A patient with RA is prescribed methotrexate. What key information should the nurse emphasize regarding alcohol consumption?
What is the rationale for routinely monitoring liver enzymes in patients receiving methotrexate for rheumatoid arthritis?
What is the rationale for routinely monitoring liver enzymes in patients receiving methotrexate for rheumatoid arthritis?
Which statement accurately reflects the action of glucocorticoids in managing rheumatoid arthritis?
Which statement accurately reflects the action of glucocorticoids in managing rheumatoid arthritis?
Why is early diagnosis and treatment with DMARDs crucial for patients with rheumatoid arthritis?
Why is early diagnosis and treatment with DMARDs crucial for patients with rheumatoid arthritis?
What is the mechanism of action of nonbiological DMARDs like methotrexate in the treatment of rheumatoid arthritis?
What is the mechanism of action of nonbiological DMARDs like methotrexate in the treatment of rheumatoid arthritis?
Which nursing intervention is MOST important for patients on anti-inflammatory and immunosuppressive therapy for rheumatoid arthritis?
Which nursing intervention is MOST important for patients on anti-inflammatory and immunosuppressive therapy for rheumatoid arthritis?
A patient with RA has an elevated platelet count during a routine checkup. What is the MOST likely reason for this finding?
A patient with RA has an elevated platelet count during a routine checkup. What is the MOST likely reason for this finding?
Which of the following best describes a 'diarthrosis' joint?
Which of the following best describes a 'diarthrosis' joint?
When assessing a patient's posture, which anatomical landmarks should ideally align with a plumb line?
When assessing a patient's posture, which anatomical landmarks should ideally align with a plumb line?
During a musculoskeletal assessment, what is the BEST method for assessing muscle strength?
During a musculoskeletal assessment, what is the BEST method for assessing muscle strength?
Why is it crucial to avoid using the thumb when assessing a patient's peripheral pulses?
Why is it crucial to avoid using the thumb when assessing a patient's peripheral pulses?
Which laboratory finding is MOST indicative of increased fracture risk due to reduced bone integrity or density?
Which laboratory finding is MOST indicative of increased fracture risk due to reduced bone integrity or density?
When preparing a patient for a CT scan with IV contrast, which nursing action is MOST important regarding the patient's medication history?
When preparing a patient for a CT scan with IV contrast, which nursing action is MOST important regarding the patient's medication history?
What nursing intervention is MOST crucial when preparing a patient for an MRI to ensure patient safety and test accuracy?
What nursing intervention is MOST crucial when preparing a patient for an MRI to ensure patient safety and test accuracy?
A nurse is preparing a patient for an arthrogram. Which action is MOST relevant to ensure patient safety and accurate examination results?
A nurse is preparing a patient for an arthrogram. Which action is MOST relevant to ensure patient safety and accurate examination results?
A patient is scheduled for a bone mineral density study. Which statement indicates the patient requires further teaching prior to the procedure?
A patient is scheduled for a bone mineral density study. Which statement indicates the patient requires further teaching prior to the procedure?
What instruction is MOST important for the nurse to provide to a breastfeeding mother who needs to undergo a bone scan with radioactive tracers?
What instruction is MOST important for the nurse to provide to a breastfeeding mother who needs to undergo a bone scan with radioactive tracers?
In an electromyography (EMG) test, what is the expected finding when a muscle is at rest?
In an electromyography (EMG) test, what is the expected finding when a muscle is at rest?
Which preoperative instruction is MOST important for a patient scheduled for an arthroscopic examination?
Which preoperative instruction is MOST important for a patient scheduled for an arthroscopic examination?
Which of the following best explains sarcopenia?
Which of the following best explains sarcopenia?
What is the primary rationale for advising older adults to exercise regularly to prevent falls?
What is the primary rationale for advising older adults to exercise regularly to prevent falls?
Which of the following age-related changes contributes MOST directly to an increased risk of fractures in older adults?
Which of the following age-related changes contributes MOST directly to an increased risk of fractures in older adults?
Which statement accurately describes joint changes associated with aging?
Which statement accurately describes joint changes associated with aging?
An elderly patient has been diagnosed with Dowager's hump. What age-related condition most likely contributed to this condition?
An elderly patient has been diagnosed with Dowager's hump. What age-related condition most likely contributed to this condition?
What is the MOST important consideration when reviewing medications as a fall prevention strategy for older adults?
What is the MOST important consideration when reviewing medications as a fall prevention strategy for older adults?
What is the primary purpose of conducting annual eye examinations to prevent falls in older adults?
What is the primary purpose of conducting annual eye examinations to prevent falls in older adults?
Which of the following findings in a patient presenting with joint pain would be MOST indicative of osteoarthritis rather than rheumatoid arthritis?
Which of the following findings in a patient presenting with joint pain would be MOST indicative of osteoarthritis rather than rheumatoid arthritis?
A patient with osteoarthritis is prescribed acetaminophen for pain relief. What is the MOST important instruction for the nurse to provide regarding the use of this medication?
A patient with osteoarthritis is prescribed acetaminophen for pain relief. What is the MOST important instruction for the nurse to provide regarding the use of this medication?
Which of the following nonpharmacological interventions would be MOST appropriate to recommend to a patient with osteoarthritis to improve joint mobility and reduce stiffness?
Which of the following nonpharmacological interventions would be MOST appropriate to recommend to a patient with osteoarthritis to improve joint mobility and reduce stiffness?
A patient diagnosed with rheumatoid arthritis (RA) is starting methotrexate therapy. Which of the following instructions is MOST critical for the nurse to emphasize?
A patient diagnosed with rheumatoid arthritis (RA) is starting methotrexate therapy. Which of the following instructions is MOST critical for the nurse to emphasize?
An older adult patient is being evaluated for falls. Which age-related musculoskeletal change is MOST likely to contribute to an increased risk of falls?
An older adult patient is being evaluated for falls. Which age-related musculoskeletal change is MOST likely to contribute to an increased risk of falls?
Which component of connective tissue aids in cellular adhesion and facilitates substance movement between capillaries and cells?
Which component of connective tissue aids in cellular adhesion and facilitates substance movement between capillaries and cells?
What is the MOST accurate description of the role that obesity plays in the development of osteoarthritis?
What is the MOST accurate description of the role that obesity plays in the development of osteoarthritis?
Why is it important to monitor kidney function in osteoarthritis patients who are taking NSAIDs regularly?
Why is it important to monitor kidney function in osteoarthritis patients who are taking NSAIDs regularly?
In a patient with rheumatoid arthritis (RA), what finding during joint palpation is the MOST indicative of synovitis?
In a patient with rheumatoid arthritis (RA), what finding during joint palpation is the MOST indicative of synovitis?
When initiating methotrexate for a patient with rheumatoid arthritis, what education should be provided?
When initiating methotrexate for a patient with rheumatoid arthritis, what education should be provided?
Why are a tuberculin skin test and hepatitis testing performed before initiating biological therapies for rheumatoid arthritis (RA)?
Why are a tuberculin skin test and hepatitis testing performed before initiating biological therapies for rheumatoid arthritis (RA)?
What is the underlying process that leads to the increased risk of fractures in individuals with osteoporosis?
What is the underlying process that leads to the increased risk of fractures in individuals with osteoporosis?
Which factor contributes to the higher incidence of osteoporosis and related fractures in small-framed women of European descent?
Which factor contributes to the higher incidence of osteoporosis and related fractures in small-framed women of European descent?
How do changes in parathyroid hormone levels contribute to the pathophysiology of osteoporosis?
How do changes in parathyroid hormone levels contribute to the pathophysiology of osteoporosis?
Why is osteoporosis often referred to as a 'silent disease'?
Why is osteoporosis often referred to as a 'silent disease'?
What is the rationale behind recommending weight-bearing exercises for individuals at risk of osteoporosis?
What is the rationale behind recommending weight-bearing exercises for individuals at risk of osteoporosis?
What is the significance of the erythrocyte sedimentation rate (ESR) in diagnosing osteomyelitis?
What is the significance of the erythrocyte sedimentation rate (ESR) in diagnosing osteomyelitis?
Why are radiographs (X-rays) not generally the first-line diagnostic tool for acute osteomyelitis?
Why are radiographs (X-rays) not generally the first-line diagnostic tool for acute osteomyelitis?
What is the primary goal of surgical debridement in the management of osteomyelitis?
What is the primary goal of surgical debridement in the management of osteomyelitis?
Why is it important to provide nutritional support with a high-protein diet for patients with osteomyelitis?
Why is it important to provide nutritional support with a high-protein diet for patients with osteomyelitis?
What is the purpose of using a cemented approach in total joint arthroplasty (TJA), especially in older patients?
What is the purpose of using a cemented approach in total joint arthroplasty (TJA), especially in older patients?
What is the significance of monitoring temperature and white blood cell (WBC) count postoperatively in patients undergoing TJA?
What is the significance of monitoring temperature and white blood cell (WBC) count postoperatively in patients undergoing TJA?
What is the MOST accurate definition of heterotrophic ossification (HO) as a complication of total hip arthroplasty (THA)?
What is the MOST accurate definition of heterotrophic ossification (HO) as a complication of total hip arthroplasty (THA)?
Why is vitamin D intake important for maintaining bone health and preventing osteoporosis?
Why is vitamin D intake important for maintaining bone health and preventing osteoporosis?
In the context of osteomyelitis, what does a 'positive periosteal elevation' indicate?
In the context of osteomyelitis, what does a 'positive periosteal elevation' indicate?
What factor primarily determines the choice of antibiotic for the treatment of osteomyelitis?
What factor primarily determines the choice of antibiotic for the treatment of osteomyelitis?
According to the Bone Health and Osteoporosis Foundation (BHOF), approximately how many Americans are estimated to have osteoporosis or low bone density?
According to the Bone Health and Osteoporosis Foundation (BHOF), approximately how many Americans are estimated to have osteoporosis or low bone density?
What is the estimated overall mortality rate within the first year after a hip fracture related to osteoporosis?
What is the estimated overall mortality rate within the first year after a hip fracture related to osteoporosis?
Which of the following best describes how osteomyelitis spreads exogenously?
Which of the following best describes how osteomyelitis spreads exogenously?
Which of the following accurately reflects the pathophysiology of bone loss (osteopenia) in osteoporosis?
Which of the following accurately reflects the pathophysiology of bone loss (osteopenia) in osteoporosis?
Why are X-rays not typically used for osteoporosis screening and assessment?
Why are X-rays not typically used for osteoporosis screening and assessment?
Which population is most commonly affected by acute hematogenous osteomyelitis?
Which population is most commonly affected by acute hematogenous osteomyelitis?
For a patient diagnosed with osteomyelitis because of an unknown bacteria, and awaiting culture and sensitivity results, what is the recommended first step in antibiotic therapy?
For a patient diagnosed with osteomyelitis because of an unknown bacteria, and awaiting culture and sensitivity results, what is the recommended first step in antibiotic therapy?
What laboratory finding is considered conclusive data to confirm Osteomyelitis?
What laboratory finding is considered conclusive data to confirm Osteomyelitis?
Why is surgical debridement a crucial component in the management of osteomyelitis?
Why is surgical debridement a crucial component in the management of osteomyelitis?
Which of the following is considered a common clinical manifestation of acute osteomyelitis?
Which of the following is considered a common clinical manifestation of acute osteomyelitis?
In the context of total joint arthroplasty (TJA), what is one of the MOST concerning immediate postoperative complications?
In the context of total joint arthroplasty (TJA), what is one of the MOST concerning immediate postoperative complications?
A surgeon may order prophylactic antibiotics for a patient with a total joint replacement until which of the following occurs?
A surgeon may order prophylactic antibiotics for a patient with a total joint replacement until which of the following occurs?
Why is a cemented approach to joint replacement typically favored for older adults?
Why is a cemented approach to joint replacement typically favored for older adults?
What is the mechanism behind heterotrophic ossification (HO) as a complication of total hip arthroplasty (THA)?
What is the mechanism behind heterotrophic ossification (HO) as a complication of total hip arthroplasty (THA)?
What statement is true regarding Total Joint Arthroplasty(TJA)?
What statement is true regarding Total Joint Arthroplasty(TJA)?
Which of the following best explains why PPIs are considered more effective than H2RAs in reducing gastric acid secretion?
Which of the following best explains why PPIs are considered more effective than H2RAs in reducing gastric acid secretion?
A patient taking a PPI for an extended period is advised to monitor for bone fractures. Which of the following mechanisms explains this potential side effect?
A patient taking a PPI for an extended period is advised to monitor for bone fractures. Which of the following mechanisms explains this potential side effect?
Why is it important to time the administration of sucralfate (Carafate) in relation to antacids?
Why is it important to time the administration of sucralfate (Carafate) in relation to antacids?
A patient with a history of GERD is prescribed metoclopramide (Reglan). What is the primary mechanism by which this medication helps to manage GERD symptoms?
A patient with a history of GERD is prescribed metoclopramide (Reglan). What is the primary mechanism by which this medication helps to manage GERD symptoms?
A patient is prescribed a triple therapy regimen for H. pylori eradication. Which of the following medication combinations is most likely to be included in this regimen?
A patient is prescribed a triple therapy regimen for H. pylori eradication. Which of the following medication combinations is most likely to be included in this regimen?
A patient taking cimetidine chronically needs monitoring for potential drug interactions. Which of the following drugs has a known interaction with cimetidine?
A patient taking cimetidine chronically needs monitoring for potential drug interactions. Which of the following drugs has a known interaction with cimetidine?
When initiating therapy with misoprostol (Cytotec) for a patient taking NSAIDs, what key monitoring parameter should the nurse prioritize?
When initiating therapy with misoprostol (Cytotec) for a patient taking NSAIDs, what key monitoring parameter should the nurse prioritize?
A patient is prescribed a PPI for long-term GERD management. What potential long-term adverse effect should the nurse educate the patient about?
A patient is prescribed a PPI for long-term GERD management. What potential long-term adverse effect should the nurse educate the patient about?
A patient with GERD is prescribed ranitidine. What is the MOST important education to provide to this patient?
A patient with GERD is prescribed ranitidine. What is the MOST important education to provide to this patient?
An older adult patient is prescribed calcium carbonate (TUMS) for GERD. Given the patient's age, what is the most important nursing consideration related to this medication?
An older adult patient is prescribed calcium carbonate (TUMS) for GERD. Given the patient's age, what is the most important nursing consideration related to this medication?
Which of the following medications requires monitoring of kidney and liver function serum levels?
Which of the following medications requires monitoring of kidney and liver function serum levels?
Which of the following instructions should be provided to a patient prescribed sucralfate (Carafate) for peptic ulcer disease?
Which of the following instructions should be provided to a patient prescribed sucralfate (Carafate) for peptic ulcer disease?
A patient is prescribed a proton pump inhibitor (PPI). What is the MOST significant difference between PPIs and H2-receptor antagonists (H2RAs) regarding their mechanism of action?
A patient is prescribed a proton pump inhibitor (PPI). What is the MOST significant difference between PPIs and H2-receptor antagonists (H2RAs) regarding their mechanism of action?
A patient is prescribed a medication to increase gastric emptying for GERD. Which of the following medications is MOST likely prescribed?
A patient is prescribed a medication to increase gastric emptying for GERD. Which of the following medications is MOST likely prescribed?
A patient is prescribed misoprostol (Cytotec) while taking NSAIDs. What is the primary reason for prescribing misoprostol in this case?
A patient is prescribed misoprostol (Cytotec) while taking NSAIDs. What is the primary reason for prescribing misoprostol in this case?
A patient is prescribed a triple therapy regimen for H. pylori eradication. Which of the following instructions is MOST important for the nurse to emphasize?
A patient is prescribed a triple therapy regimen for H. pylori eradication. Which of the following instructions is MOST important for the nurse to emphasize?
A patient with GERD is prescribed ranitidine. Which of the following should the nurse include in patient teaching regarding this medication?
A patient with GERD is prescribed ranitidine. Which of the following should the nurse include in patient teaching regarding this medication?
A patient has H. pylori and is allergic to penicillin. What is a likely first-line therapy?
A patient has H. pylori and is allergic to penicillin. What is a likely first-line therapy?
A patient taking omeprazole reports persistent heartburn despite taking the medication daily for the past four weeks. What is the MOST appropriate action?
A patient taking omeprazole reports persistent heartburn despite taking the medication daily for the past four weeks. What is the MOST appropriate action?
A patient on long-term omeprazole therapy is advised to have their serum magnesium levels checked regularly. What condition is the provider MOST concerned about?
A patient on long-term omeprazole therapy is advised to have their serum magnesium levels checked regularly. What condition is the provider MOST concerned about?
A patient is prescribed both sucralfate and an antacid for peptic ulcer disease. Which instruction regarding the timing of these medications is MOST accurate?
A patient is prescribed both sucralfate and an antacid for peptic ulcer disease. Which instruction regarding the timing of these medications is MOST accurate?
Which of the following BEST describes the rationale for utilizing a combination of antibiotics, such as clarithromycin and amoxicillin, along with a PPI when treating H. pylori infections?
Which of the following BEST describes the rationale for utilizing a combination of antibiotics, such as clarithromycin and amoxicillin, along with a PPI when treating H. pylori infections?
Cimetidine is prescribed to a patient with heartburn. The patient also takes warfarin for atrial fibrillation. What concern is MOST relevant with this drug combination?
Cimetidine is prescribed to a patient with heartburn. The patient also takes warfarin for atrial fibrillation. What concern is MOST relevant with this drug combination?
A female patient is prescribed misoprostol while taking high doses of NSAIDs for arthritis. Which information is MOST crucial to review with the patient?
A female patient is prescribed misoprostol while taking high doses of NSAIDs for arthritis. Which information is MOST crucial to review with the patient?
A patient with GERD is prescribed metoclopramide (Reglan). Which potential side effect should the nurse monitor for?
A patient with GERD is prescribed metoclopramide (Reglan). Which potential side effect should the nurse monitor for?
A patient with a history of GERD has been taking ranitidine for several years but reports that it is no longer effectively controlling symptoms. What alternative medication class might be considered?
A patient with a history of GERD has been taking ranitidine for several years but reports that it is no longer effectively controlling symptoms. What alternative medication class might be considered?
What is TRUE regarding antacids?
What is TRUE regarding antacids?
A patient is being evaluated for GERD. What diagnostic test will test gastric emptying?
A patient is being evaluated for GERD. What diagnostic test will test gastric emptying?
In osteoarthritis, how does the synovial membrane contribute to the pathophysiology of the disease?
In osteoarthritis, how does the synovial membrane contribute to the pathophysiology of the disease?
Which of the following is a modifiable risk factor that can significantly impact the progression of osteoarthritis?
Which of the following is a modifiable risk factor that can significantly impact the progression of osteoarthritis?
A patient with osteoarthritis reports experiencing crepitus in the knee. What physiological change is most directly associated with this symptom?
A patient with osteoarthritis reports experiencing crepitus in the knee. What physiological change is most directly associated with this symptom?
A patient with osteoarthritis is advised to undergo laboratory testing for diagnostic purposes. What is the purpose of ordering laboratory tests in this case?
A patient with osteoarthritis is advised to undergo laboratory testing for diagnostic purposes. What is the purpose of ordering laboratory tests in this case?
Which intervention should a nurse prioritize when caring for a patient with osteoarthritis experiencing chronic pain?
Which intervention should a nurse prioritize when caring for a patient with osteoarthritis experiencing chronic pain?
A patient with osteoarthritis is prescribed acetaminophen. What information should the nurse emphasize when teaching the patient about this medication?
A patient with osteoarthritis is prescribed acetaminophen. What information should the nurse emphasize when teaching the patient about this medication?
Which of the following nonpharmacological interventions would be most suitable for a patient with osteoarthritis to improve joint mobility and reduce stiffness?
Which of the following nonpharmacological interventions would be most suitable for a patient with osteoarthritis to improve joint mobility and reduce stiffness?
A patient with osteoarthritis is scheduled for arthroscopic irrigation and débridement. What should the nurse explain as the primary goal of this procedure?
A patient with osteoarthritis is scheduled for arthroscopic irrigation and débridement. What should the nurse explain as the primary goal of this procedure?
Which nursing assessment finding would be most indicative of osteoarthritis rather than rheumatoid arthritis?
Which nursing assessment finding would be most indicative of osteoarthritis rather than rheumatoid arthritis?
A patient newly diagnosed with rheumatoid arthritis (RA) asks the nurse about the underlying cause of the disease. Which explanation is most accurate?
A patient newly diagnosed with rheumatoid arthritis (RA) asks the nurse about the underlying cause of the disease. Which explanation is most accurate?
What is the primary role of disease-modifying antirheumatic drugs (DMARDs) in the management of rheumatoid arthritis?
What is the primary role of disease-modifying antirheumatic drugs (DMARDs) in the management of rheumatoid arthritis?
Which of the following is a significant risk factor for the development of rheumatoid arthritis (RA)?
Which of the following is a significant risk factor for the development of rheumatoid arthritis (RA)?
A patient with rheumatoid arthritis reports experiencing morning stiffness that lasts for several hours each day. How does this manifestation correlate with the pathophysiology of RA?
A patient with rheumatoid arthritis reports experiencing morning stiffness that lasts for several hours each day. How does this manifestation correlate with the pathophysiology of RA?
Which of the following laboratory findings is most indicative of active inflammation in a patient with rheumatoid arthritis?
Which of the following laboratory findings is most indicative of active inflammation in a patient with rheumatoid arthritis?
A patient with rheumatoid arthritis develops swan-neck deformities in the fingers. What pathological process contributes most directly to this deformity?
A patient with rheumatoid arthritis develops swan-neck deformities in the fingers. What pathological process contributes most directly to this deformity?
Which type of exercise is most beneficial for a patient with rheumatoid arthritis to maintain joint function and overall well-being?
Which type of exercise is most beneficial for a patient with rheumatoid arthritis to maintain joint function and overall well-being?
Early diagnosis and treatment with DMARDs is crucial for patients with rheumatoid arthritis. What is the primary rationale behind this approach?
Early diagnosis and treatment with DMARDs is crucial for patients with rheumatoid arthritis. What is the primary rationale behind this approach?
A rheumatoid arthritis patient on anti-inflammatory and immunosuppressive therapy should be taught about what nursing intervention?
A rheumatoid arthritis patient on anti-inflammatory and immunosuppressive therapy should be taught about what nursing intervention?
In the case of Shannon Logan, a 32-year-old with complaints of multiple joint pain and swelling, fatigue, and morning stiffness, what is the most likely underlying condition?
In the case of Shannon Logan, a 32-year-old with complaints of multiple joint pain and swelling, fatigue, and morning stiffness, what is the most likely underlying condition?
What laboratory findings are MOST consistent with an initial diagnosis of Rheumatoid Arthritis (RA) in Ms. Logan?
What laboratory findings are MOST consistent with an initial diagnosis of Rheumatoid Arthritis (RA) in Ms. Logan?
Which of the following medications is the rheumatologist most likely to begin to use to try and decrease the disease progression for Ms. Logan?
Which of the following medications is the rheumatologist most likely to begin to use to try and decrease the disease progression for Ms. Logan?
During a nursing assessment of Ms. Logan, what key lifestyle factor should the nurse prioritize related to planning her treatment?
During a nursing assessment of Ms. Logan, what key lifestyle factor should the nurse prioritize related to planning her treatment?
In the case of Ms. Logan, which nursing intervention would be most important when administering methotrexate?
In the case of Ms. Logan, which nursing intervention would be most important when administering methotrexate?
Ms. Logan has begun treatment for Rheumatoid Arthritis, the nurse should include which lifestyle modifications in her plan of care?
Ms. Logan has begun treatment for Rheumatoid Arthritis, the nurse should include which lifestyle modifications in her plan of care?
Regarding diagnostic criteria, what clinical manifestation differentiates Rheumatoid Arthritis from Osteoarthritis?
Regarding diagnostic criteria, what clinical manifestation differentiates Rheumatoid Arthritis from Osteoarthritis?
A medication used to treat Rheumatoid Arthritis is DMARDs, what does DMARD stand for?
A medication used to treat Rheumatoid Arthritis is DMARDs, what does DMARD stand for?
In Rheumatoid Arthritis (RA), increased synovial fluid in the joints often leads to what subsequent symptoms?
In Rheumatoid Arthritis (RA), increased synovial fluid in the joints often leads to what subsequent symptoms?
What is the goal of surgical management for patients with osteoarthritis?
What is the goal of surgical management for patients with osteoarthritis?
In Rheumatoid Arthritis, what are the four primary clinical manifestations?
In Rheumatoid Arthritis, what are the four primary clinical manifestations?
What assessment finding is most important for the nurse to focus on when a patient has rheumatoid arthritis?
What assessment finding is most important for the nurse to focus on when a patient has rheumatoid arthritis?
After a nurse administers DMARD therapy to a patient, what teaching intervention should be included?
After a nurse administers DMARD therapy to a patient, what teaching intervention should be included?
Before administering analgesics and anti-inflammatories for Rheumatoid Arthritis, the nurse should?
Before administering analgesics and anti-inflammatories for Rheumatoid Arthritis, the nurse should?
If a patient with Arthritis had a nursing diagnosis of self-care-deficit, what assessment finding is most likely?
If a patient with Arthritis had a nursing diagnosis of self-care-deficit, what assessment finding is most likely?
What is the main goal when evaluating care outcomes surrounding Rheumatoid Arthritis?
What is the main goal when evaluating care outcomes surrounding Rheumatoid Arthritis?
Which of the following is true regarding Rheumatic Arthritis?
Which of the following is true regarding Rheumatic Arthritis?
Why are patients encouraged to participate in regular physical activity who are diagnosed with osteoarthritis?
Why are patients encouraged to participate in regular physical activity who are diagnosed with osteoarthritis?
A patient with a history of Rheumatoid Arthritis, is most likely affected by:
A patient with a history of Rheumatoid Arthritis, is most likely affected by:
Symptoms of Osteoarthritis include which of the following:
Symptoms of Osteoarthritis include which of the following:
Which pathophysiological process is most directly associated with the progression of osteoarthritis?
Which pathophysiological process is most directly associated with the progression of osteoarthritis?
A 60-year-old female presents with joint pain. Considering the risk factors of osteoarthritis, which factor from the patient's history would MOST significantly contribute to her risk?
A 60-year-old female presents with joint pain. Considering the risk factors of osteoarthritis, which factor from the patient's history would MOST significantly contribute to her risk?
A patient with osteoarthritis reports increased pain and stiffness in the knees, especially in the morning. Which of the following interventions would be MOST appropriate to suggest?
A patient with osteoarthritis reports increased pain and stiffness in the knees, especially in the morning. Which of the following interventions would be MOST appropriate to suggest?
What is the primary rationale for using radiographs (X-rays) in the diagnosis of osteoarthritis?
What is the primary rationale for using radiographs (X-rays) in the diagnosis of osteoarthritis?
A nurse is teaching a patient with rheumatoid arthritis about managing fatigue. Which strategy is MOST likely to be beneficial?
A nurse is teaching a patient with rheumatoid arthritis about managing fatigue. Which strategy is MOST likely to be beneficial?
Ms. Logan, a 32-year-old with RA, has been started on methotrexate. Which statement indicates effective teaching about managing this medication?
Ms. Logan, a 32-year-old with RA, has been started on methotrexate. Which statement indicates effective teaching about managing this medication?
After starting Ms. Logan on methotrexate, the nurse should prioritize monitoring for:
After starting Ms. Logan on methotrexate, the nurse should prioritize monitoring for:
What is the primary role of red bone marrow within the skeletal system?
What is the primary role of red bone marrow within the skeletal system?
Which of the following hormones directly contributes to the regulation of calcium levels in the blood and bone remodeling?
Which of the following hormones directly contributes to the regulation of calcium levels in the blood and bone remodeling?
A patient is being assessed for musculoskeletal function. If the patient has a limited range of motion in the knee, which instrument would be MOST appropriate to quantify the joint's mobility?
A patient is being assessed for musculoskeletal function. If the patient has a limited range of motion in the knee, which instrument would be MOST appropriate to quantify the joint's mobility?
When documenting a patient's posture, what key anatomical landmarks should be considered to assess alignment?
When documenting a patient's posture, what key anatomical landmarks should be considered to assess alignment?
After a musculoskeletal injury, a patient reports altered sensation in the affected limb. What underlying physiological process is MOST likely responsible for this symptom?
After a musculoskeletal injury, a patient reports altered sensation in the affected limb. What underlying physiological process is MOST likely responsible for this symptom?
During a musculoskeletal assessment, diminished or absent pulses in an injured extremity suggest which of the following complications?
During a musculoskeletal assessment, diminished or absent pulses in an injured extremity suggest which of the following complications?
What age-related change in the musculoskeletal system contributes to decreased range of motion?
What age-related change in the musculoskeletal system contributes to decreased range of motion?
Which of the following imaging techniques provides the MOST detailed visualization of soft tissues, such as ligaments and tendons, in a musculoskeletal injury?
Which of the following imaging techniques provides the MOST detailed visualization of soft tissues, such as ligaments and tendons, in a musculoskeletal injury?
A patient is scheduled for an arthrogram. What information regarding allergies is MOST critical for the nurse to obtain prior to the procedure?
A patient is scheduled for an arthrogram. What information regarding allergies is MOST critical for the nurse to obtain prior to the procedure?
What information is most important for the nurse to convey to the patient after an arthroscopic examination?
What information is most important for the nurse to convey to the patient after an arthroscopic examination?
Considering the role of fascia within the musculoskeletal system, how does its decreased water content in older adults affect mobility?
Considering the role of fascia within the musculoskeletal system, how does its decreased water content in older adults affect mobility?
What age-related change increases the risk of falls in older adults?
What age-related change increases the risk of falls in older adults?
What activity is MOST effective in preventing falls in older adults by improving strength, balance, and coordination?
What activity is MOST effective in preventing falls in older adults by improving strength, balance, and coordination?
The physician orders lab work to test for Vitamin D, what function(s) is the physician trying to evaluate?
The physician orders lab work to test for Vitamin D, what function(s) is the physician trying to evaluate?
If a physician is performing a joint mobility assessment, what would the physician be assessing?
If a physician is performing a joint mobility assessment, what would the physician be assessing?
If a patient comes in with a musculoskeletal injury, what question should the nurse ask during the past medical and present illness history assessment?
If a patient comes in with a musculoskeletal injury, what question should the nurse ask during the past medical and present illness history assessment?
When addressing age-related changes in older adults it is important for the nurse to educate on..
When addressing age-related changes in older adults it is important for the nurse to educate on..
A male patient in his 30s presents to the clinic reporting pain and stiffness in his spinal column. Which follow-up question by the nurse will elicit the MOST useful information?
A male patient in his 30s presents to the clinic reporting pain and stiffness in his spinal column. Which follow-up question by the nurse will elicit the MOST useful information?
What is MOST important question to start with when assessing musculoskeletal systems of adults?
What is MOST important question to start with when assessing musculoskeletal systems of adults?
What type of joint provides no movement?
What type of joint provides no movement?
The orthopedic surgeon requests a bone scan, what are they trying to evaluate?
The orthopedic surgeon requests a bone scan, what are they trying to evaluate?
How does the musculoskeletal system allow for movement of the body?
How does the musculoskeletal system allow for movement of the body?
What is the purpose of an arthrocentesis?
What is the purpose of an arthrocentesis?
A patient reports new onset of pain in her left knee. What is the MOST appropriate initial nursing assessment?
A patient reports new onset of pain in her left knee. What is the MOST appropriate initial nursing assessment?
The nurse evaluates lab results of a patient complaining of joint pain and suspects it may be musculoskeletal related. Which result would the nurse question and inform the provider?
The nurse evaluates lab results of a patient complaining of joint pain and suspects it may be musculoskeletal related. Which result would the nurse question and inform the provider?
An elderly patient is suspected of sarcopenia, what assessment finding goes with this diagnosis?
An elderly patient is suspected of sarcopenia, what assessment finding goes with this diagnosis?
What is the purpose of palpating pulses when assessing the musculoskeletal system?
What is the purpose of palpating pulses when assessing the musculoskeletal system?
How does the musculoskeletal system contribute to the protection of vital organs?
How does the musculoskeletal system contribute to the protection of vital organs?
A patient is being assessed for musculoskeletal function. What is the MOST important reason to assess the patient's gait?
A patient is being assessed for musculoskeletal function. What is the MOST important reason to assess the patient's gait?
Which of the following is a critical nursing consideration for patients completing diagnostic studies?
Which of the following is a critical nursing consideration for patients completing diagnostic studies?
How does sarcopenia primarily impact musculoskeletal function in older adults?
How does sarcopenia primarily impact musculoskeletal function in older adults?
Which statement from the patient requires immediate education regarding their musculoskeletal health?
Which statement from the patient requires immediate education regarding their musculoskeletal health?
If a patient has a flexion injury and the muscles are shortened, what will the physician ask the patient to do to evaluate the muscle?
If a patient has a flexion injury and the muscles are shortened, what will the physician ask the patient to do to evaluate the muscle?
What type of connective tissue connects muscle to bone?
What type of connective tissue connects muscle to bone?
What component of the musculoskeletal system facilitates joint movement by providing a smooth, gliding surface where bones articulate?
What component of the musculoskeletal system facilitates joint movement by providing a smooth, gliding surface where bones articulate?
The physician is inspecting the patient's feet. To ensure they are evaluating the muscles of the ankle and foot, what movement will the physician ask the patient to do?
The physician is inspecting the patient's feet. To ensure they are evaluating the muscles of the ankle and foot, what movement will the physician ask the patient to do?
What is the primary action of ligaments in the musculoskeletal system?
What is the primary action of ligaments in the musculoskeletal system?
During the physical assessment of the musculoskeletal system, what does the nurse specifically evaluate when assessing joint mobility?
During the physical assessment of the musculoskeletal system, what does the nurse specifically evaluate when assessing joint mobility?
While assessing a patient's medical history, which past medical condition is MOST likely to affect the musculoskeletal system?
While assessing a patient's medical history, which past medical condition is MOST likely to affect the musculoskeletal system?
Considering hormonal influences on bone density, which laboratory test would be MOST important to monitor in a postmenopausal woman?
Considering hormonal influences on bone density, which laboratory test would be MOST important to monitor in a postmenopausal woman?
Which of the following pathophysiological changes is MOST directly associated with bone loss in osteoporosis?
Which of the following pathophysiological changes is MOST directly associated with bone loss in osteoporosis?
A primary healthcare provider is assessing an elderly patient at risk of developing osteoporosis. Which of the following is classified as a primary risk factor?
A primary healthcare provider is assessing an elderly patient at risk of developing osteoporosis. Which of the following is classified as a primary risk factor?
A patient undergoing bone density testing using quantitative computed tomography (QCT) asks about the benefits of this method. Which of the following is an advantage of QCT over standard bone densitometry?
A patient undergoing bone density testing using quantitative computed tomography (QCT) asks about the benefits of this method. Which of the following is an advantage of QCT over standard bone densitometry?
What is the primary rationale for recommending weight reduction as a lifestyle intervention for a patient diagnosed with osteoporosis?
What is the primary rationale for recommending weight reduction as a lifestyle intervention for a patient diagnosed with osteoporosis?
When assessing a patient with suspected osteomyelitis, which clinical manifestation should prompt the nurse to suspect a more advanced or complicated infection?
When assessing a patient with suspected osteomyelitis, which clinical manifestation should prompt the nurse to suspect a more advanced or complicated infection?
Which of the following diagnostic findings is MOST indicative of osteomyelitis, requiring prompt intervention?
Which of the following diagnostic findings is MOST indicative of osteomyelitis, requiring prompt intervention?
What is the rationale for utilizing thermal therapy as an intervention for patients with osteomyelitis?
What is the rationale for utilizing thermal therapy as an intervention for patients with osteomyelitis?
A patient with chronic osteomyelitis is being discharged with a peripherally inserted central catheter (PICC) line for long-term antibiotic administration. Which teaching point is most important?
A patient with chronic osteomyelitis is being discharged with a peripherally inserted central catheter (PICC) line for long-term antibiotic administration. Which teaching point is most important?
What is the primary indication for total joint arthroplasty (TJA)?
What is the primary indication for total joint arthroplasty (TJA)?
Which imaging modality is MOST useful in assessing and diagnosing a patient who is a candidate for total joint arthroplasty (TJA)?
Which imaging modality is MOST useful in assessing and diagnosing a patient who is a candidate for total joint arthroplasty (TJA)?
Following a total hip arthroplasty, a patient experiences hypotension, hypovolemia, and persistent bleeding. Which immediate postoperative complication does this presentation MOST strongly suggest?
Following a total hip arthroplasty, a patient experiences hypotension, hypovolemia, and persistent bleeding. Which immediate postoperative complication does this presentation MOST strongly suggest?
What nursing intervention is most important to implement as part of the turning schedule and proper positioning for a patient after joint replacement?
What nursing intervention is most important to implement as part of the turning schedule and proper positioning for a patient after joint replacement?
A patient being evaluated for musculoskeletal problems reports experiencing increased difficulty walking and has noticed a progressive rounding of the upper back. These signs suggest which condition?
A patient being evaluated for musculoskeletal problems reports experiencing increased difficulty walking and has noticed a progressive rounding of the upper back. These signs suggest which condition?
To evaluate musculoskeletal symptoms from a 55 year old female, which of the following actions is MOST important for the nurse to focus on?
To evaluate musculoskeletal symptoms from a 55 year old female, which of the following actions is MOST important for the nurse to focus on?
The healthcare provider has prescribed Ms. Doherty begin bisphosphonate therapy with ibandronate (Boniva) by mouth. How often is Boniva typically taken?
The healthcare provider has prescribed Ms. Doherty begin bisphosphonate therapy with ibandronate (Boniva) by mouth. How often is Boniva typically taken?
What is the rationale for healthcare providers recommending that a osteoporosis patient takes calcium and vitamin D?
What is the rationale for healthcare providers recommending that a osteoporosis patient takes calcium and vitamin D?
In osteomyelitis, what accurately describes the pathophysiology of osteomyelitis?
In osteomyelitis, what accurately describes the pathophysiology of osteomyelitis?
Which of the following is a MAJOR postoperative complication for a surgery like a total joint replacement?
Which of the following is a MAJOR postoperative complication for a surgery like a total joint replacement?
What is the purpose of a surgeon ordering prophylactic antibiotics for a patient with a total joint replacement until specific criteria are met?
What is the purpose of a surgeon ordering prophylactic antibiotics for a patient with a total joint replacement until specific criteria are met?
The nurse teaches Ms. Doherty that she may need to remove throw rugs in the house to reduce the risk of falls. Why is that an important intervention?
The nurse teaches Ms. Doherty that she may need to remove throw rugs in the house to reduce the risk of falls. Why is that an important intervention?
Following joint replacement surgery, the nurse provides antiembolic stockings. What is the main purpose of antiembolic stockings?
Following joint replacement surgery, the nurse provides antiembolic stockings. What is the main purpose of antiembolic stockings?
What is the significance of bone resorption in the development of osteoporosis?
What is the significance of bone resorption in the development of osteoporosis?
What is the incidence of osteomyelitis associated with diabetes and foot puncture woulds?
What is the incidence of osteomyelitis associated with diabetes and foot puncture woulds?
What is the role of osteoblasts in the development of Osteoperosis?
What is the role of osteoblasts in the development of Osteoperosis?
What is the MOST accurate risk related to osteomyelitis?
What is the MOST accurate risk related to osteomyelitis?
What instruction should the nurse provide to Ms. Doherty on calcium and vitamin D supplementation?
What instruction should the nurse provide to Ms. Doherty on calcium and vitamin D supplementation?
What is the PRIMARY intervention for osteomylitis besides antibiotic therapy?
What is the PRIMARY intervention for osteomylitis besides antibiotic therapy?
When teaching a patient about osteoporosis, what intervention should the nurse include?
When teaching a patient about osteoporosis, what intervention should the nurse include?
Before a total knee arthroplasty, what information should the nurse seek from the patient?
Before a total knee arthroplasty, what information should the nurse seek from the patient?
An elderly is admitted to the hospital for management of osteomyelitis. Which intervention should the nurse include in the plan of care?
An elderly is admitted to the hospital for management of osteomyelitis. Which intervention should the nurse include in the plan of care?
Which of the following is included in the plan of care for a patient being discharged after a total hip arthroplasty?
Which of the following is included in the plan of care for a patient being discharged after a total hip arthroplasty?
If a patient presents to the hospital for severe fever following a puncture wound, what information should the nurse collect?
If a patient presents to the hospital for severe fever following a puncture wound, what information should the nurse collect?
What is the relationship between clinical interventions based on lab data, weight baring, and bone mass in a patient with osteoporosis?
What is the relationship between clinical interventions based on lab data, weight baring, and bone mass in a patient with osteoporosis?
How can an interprofessional team coordinate a treatment plan for Osteomyelitis, specifically antibiotics?
How can an interprofessional team coordinate a treatment plan for Osteomyelitis, specifically antibiotics?
Which diagnostic result is MOST definitive in order to evaluate the presence of osteoporosis?
Which diagnostic result is MOST definitive in order to evaluate the presence of osteoporosis?
What interprofessional care can be applied to a patient with osteomylitis?
What interprofessional care can be applied to a patient with osteomylitis?
Which of the following is a high priority nursing intervention for a post operative patient one day out of total knee arthroplasty surgery?
Which of the following is a high priority nursing intervention for a post operative patient one day out of total knee arthroplasty surgery?
What clinical finding provides the strongest indication that additional diagnostic testing and interventions are warranted for Ms. Doherty?
What clinical finding provides the strongest indication that additional diagnostic testing and interventions are warranted for Ms. Doherty?
If left unmanaged, what is a possible long term concern for Ms. Doherty's osteoporosis?
If left unmanaged, what is a possible long term concern for Ms. Doherty's osteoporosis?
Besides weight support, what is osteoporosis's effect on bone tissue in the body?
Besides weight support, what is osteoporosis's effect on bone tissue in the body?
Why is it essential for the nurse to assess for body image disturbance in patients with osteoporosis?
Why is it essential for the nurse to assess for body image disturbance in patients with osteoporosis?
Which intervention would be most appropriate for a patient with osteomyelitis who reports constant, unrelieved bone pain?
Which intervention would be most appropriate for a patient with osteomyelitis who reports constant, unrelieved bone pain?
What is the primary rationale for teaching patients with osteomyelitis about the importance of nutrition?
What is the primary rationale for teaching patients with osteomyelitis about the importance of nutrition?
Why is early mobilization considered a priority intervention following total joint arthroplasty (TJA)?
Why is early mobilization considered a priority intervention following total joint arthroplasty (TJA)?
A patient develops hypotension, bleeding, and hypovolemia following a TJA. What complication do these symptoms collectively suggest?
A patient develops hypotension, bleeding, and hypovolemia following a TJA. What complication do these symptoms collectively suggest?
What assessment finding should the nurse prioritize in a patient who has undergone a total hip arthroplasty?
What assessment finding should the nurse prioritize in a patient who has undergone a total hip arthroplasty?
What teaching point would be most critical to emphasize to a patient starting oral bisphosphonates for osteoporosis?
What teaching point would be most critical to emphasize to a patient starting oral bisphosphonates for osteoporosis?
A patient with osteomyelitis suddenly develops altered mental status. What is the MOST appropriate initial nursing action?
A patient with osteomyelitis suddenly develops altered mental status. What is the MOST appropriate initial nursing action?
What is the primary goal for administering antiembolic stockings such as sequential compression devices (SCDs) after total joint replacement surgery?
What is the primary goal for administering antiembolic stockings such as sequential compression devices (SCDs) after total joint replacement surgery?
What is the primary focus of rehabilitation exercises initiated postoperatively following joint replacement surgery?
What is the primary focus of rehabilitation exercises initiated postoperatively following joint replacement surgery?
What is the most common mechanism by which a medial meniscus tear occurs?
What is the most common mechanism by which a medial meniscus tear occurs?
Which statement best describes the pathophysiology of carpal tunnel syndrome?
Which statement best describes the pathophysiology of carpal tunnel syndrome?
Following an amputation, why is a multidisciplinary approach, including psychological support, crucial for patient care?
Following an amputation, why is a multidisciplinary approach, including psychological support, crucial for patient care?
An elderly patient presents with a fracture after a minor fall. What assessment finding would most strongly suggest that the fracture is related to osteoporosis?
An elderly patient presents with a fracture after a minor fall. What assessment finding would most strongly suggest that the fracture is related to osteoporosis?
Which nursing intervention is essential when caring for a patient with an external fixator to prevent infection?
Which nursing intervention is essential when caring for a patient with an external fixator to prevent infection?
Which patient education point is most important to emphasize to a patient being discharged after an open reduction and internal fixation (ORIF) of a fracture?
Which patient education point is most important to emphasize to a patient being discharged after an open reduction and internal fixation (ORIF) of a fracture?
Which strategy is most effective in preventing knee joint contractures following an above-the-knee amputation?
Which strategy is most effective in preventing knee joint contractures following an above-the-knee amputation?
What is the primary goal of applying a compression dressing to a residual limb following an amputation?
What is the primary goal of applying a compression dressing to a residual limb following an amputation?
A nurse is caring for a patient with carpal tunnel syndrome. Which intervention is most likely to alleviate the patient's symptoms?
A nurse is caring for a patient with carpal tunnel syndrome. Which intervention is most likely to alleviate the patient's symptoms?
A patient with a lower extremity fracture is at risk for fat embolism syndrome (FES). Which of the following findings would differentiate FES from a pulmonary embolism (PE)?
A patient with a lower extremity fracture is at risk for fat embolism syndrome (FES). Which of the following findings would differentiate FES from a pulmonary embolism (PE)?
A young athlete is diagnosed with a second-degree ankle sprain. What nursing education should be provided regarding weight-bearing?
A young athlete is diagnosed with a second-degree ankle sprain. What nursing education should be provided regarding weight-bearing?
For a patient with a confirmed meniscus tear, what is a primary focus in the immediate postoperative period following arthroscopic repair?
For a patient with a confirmed meniscus tear, what is a primary focus in the immediate postoperative period following arthroscopic repair?
An industrial worker reports numbness and tingling in the hands bilaterally, particularly at night. What diagnostic test would the nurse anticipate to confirm carpal tunnel syndrome?
An industrial worker reports numbness and tingling in the hands bilaterally, particularly at night. What diagnostic test would the nurse anticipate to confirm carpal tunnel syndrome?
What potential complication is most closely associated with injuries causing disruption of the blood supply in the pelvis?
What potential complication is most closely associated with injuries causing disruption of the blood supply in the pelvis?
Which of the following nursing assessments is most critical in the first 24 hours after a patient undergoes an amputation?
Which of the following nursing assessments is most critical in the first 24 hours after a patient undergoes an amputation?
A patient is diagnosed with traumatic rhabdomyolysis. What goal is the highest priority when administering intravenous fluids?
A patient is diagnosed with traumatic rhabdomyolysis. What goal is the highest priority when administering intravenous fluids?
When planning the care for a patient with a fracture, which intervention is most effective in minimizing the risk of venous thromboembolism?
When planning the care for a patient with a fracture, which intervention is most effective in minimizing the risk of venous thromboembolism?
A patient who had a recent knee arthroscopy for a meniscal tear is ready for discharge. What statement indicates a good understanding of discharge instructions?
A patient who had a recent knee arthroscopy for a meniscal tear is ready for discharge. What statement indicates a good understanding of discharge instructions?
What is the most important nursing intervention to include in the plan of care for a patient with an amputation related to trauma, to promote wound healing?
What is the most important nursing intervention to include in the plan of care for a patient with an amputation related to trauma, to promote wound healing?
A patient presents with acute pain, pallor, and pulselessness in the affected extremity. What condition is most closely associated with these symptoms?
A patient presents with acute pain, pallor, and pulselessness in the affected extremity. What condition is most closely associated with these symptoms?
Which of the following assessment findings would be most indicative of a complication following application of a cast for a fractured extremity?
Which of the following assessment findings would be most indicative of a complication following application of a cast for a fractured extremity?
What is the primary reason for encouraging deep breathing and coughing exercises in patients with musculoskeletal injuries?
What is the primary reason for encouraging deep breathing and coughing exercises in patients with musculoskeletal injuries?
Which of the following instructions would the nurse provide to a patient being discharged after surgical repair of a meniscus tear?
Which of the following instructions would the nurse provide to a patient being discharged after surgical repair of a meniscus tear?
A patient is being treated for carpal tunnel syndrome with wrist splinting. What sign indicates that the splinting is effective?
A patient is being treated for carpal tunnel syndrome with wrist splinting. What sign indicates that the splinting is effective?
What nursing intervention is most important when caring for a patient in skeletal traction?
What nursing intervention is most important when caring for a patient in skeletal traction?
A key nursing responsibility for a patient with an amputation includes:
A key nursing responsibility for a patient with an amputation includes:
A patient is diagnosed with a complete, displaced fracture of the radius and ulna. What nursing intervention would be performed?
A patient is diagnosed with a complete, displaced fracture of the radius and ulna. What nursing intervention would be performed?
A patient arrives in the ER, displaying signs of a fracture, what is the priority assessment?
A patient arrives in the ER, displaying signs of a fracture, what is the priority assessment?
A patient with a musculoskeletal injury is being discharged with a prescription for opioid pain medication. What instruction is most important?
A patient with a musculoskeletal injury is being discharged with a prescription for opioid pain medication. What instruction is most important?
What is a primary nursing intervention to assist the patient, after a traumatic amputation, to assist with psychosocial adaptation?
What is a primary nursing intervention to assist the patient, after a traumatic amputation, to assist with psychosocial adaptation?
What nursing action is essential to provide when providing pin care to a patient with an external fixator?
What nursing action is essential to provide when providing pin care to a patient with an external fixator?
What statement accurately describes the epidemiology of fractures?
What statement accurately describes the epidemiology of fractures?
What assessment would indicate to the nurse that the patient has compartment syndrome?
What assessment would indicate to the nurse that the patient has compartment syndrome?
After a fracture, the patient is started on medication, what information about the patients lifestyle should be asked?
After a fracture, the patient is started on medication, what information about the patients lifestyle should be asked?
What complication can fractures result in?
What complication can fractures result in?
After an amputation, what should be monitored?
After an amputation, what should be monitored?
During an ultrasound when splinting at night with signs and symptoms of carpal tunnel, what action is most needed?
During an ultrasound when splinting at night with signs and symptoms of carpal tunnel, what action is most needed?
What statement describes rotator cuff injuries?
What statement describes rotator cuff injuries?
For a patient that may have a meniscus tear, what assessment would be performed?
For a patient that may have a meniscus tear, what assessment would be performed?
A patient is presenting with muscle atrophy, what should you perform?
A patient is presenting with muscle atrophy, what should you perform?
A nurse is providing care to a patient with complaints of pain often presenting as a popping or clicking. What actions are needed during the nursing management?
A nurse is providing care to a patient with complaints of pain often presenting as a popping or clicking. What actions are needed during the nursing management?
What percentage range do musculoskeletal traumas account for of all unintentional injuries reported in the US?
What percentage range do musculoskeletal traumas account for of all unintentional injuries reported in the US?
Which of the following is the most likely reason an adolescent between the ages of 12 and 21 would experience a fracture?
Which of the following is the most likely reason an adolescent between the ages of 12 and 21 would experience a fracture?
Which potential fracture complication involves the disruption of blood supply to the affected bone, leading to tissue death?
Which potential fracture complication involves the disruption of blood supply to the affected bone, leading to tissue death?
Surgical intervention is likely indicated for what fracture-related circumstance?
Surgical intervention is likely indicated for what fracture-related circumstance?
A patient with a lower extremity fracture reports increased pain despite opioid administration, along with increasing swelling and numbness. What condition is most likely?
A patient with a lower extremity fracture reports increased pain despite opioid administration, along with increasing swelling and numbness. What condition is most likely?
What is the most immediate risk following a traumatic amputation?
What is the most immediate risk following a traumatic amputation?
Why should a pillow be avoided under the residual limb in the immediate postoperative period following a lower extremity amputation?
Why should a pillow be avoided under the residual limb in the immediate postoperative period following a lower extremity amputation?
Which nursing intervention is most important to include in the care plan for a patient with a new lower extremity amputation?
Which nursing intervention is most important to include in the care plan for a patient with a new lower extremity amputation?
Which movement of the knee is most likely to cause a medial meniscal tear?
Which movement of the knee is most likely to cause a medial meniscal tear?
Which of the following assessment findings would be most indicative of a meniscus injury during a physical examination?
Which of the following assessment findings would be most indicative of a meniscus injury during a physical examination?
What is the primary purpose of using nonsteroidal anti-inflammatory drugs (NSAIDs) in the conservative management of meniscus injuries?
What is the primary purpose of using nonsteroidal anti-inflammatory drugs (NSAIDs) in the conservative management of meniscus injuries?
A patient reports experiencing a sharp pain in their wrist that radiates up their arm when performing repetitive hand movements. What condition is most likely indicated?
A patient reports experiencing a sharp pain in their wrist that radiates up their arm when performing repetitive hand movements. What condition is most likely indicated?
What pathological process primarily contributes to the development of carpal tunnel syndrome?
What pathological process primarily contributes to the development of carpal tunnel syndrome?
During the acute phase of a strain or sprain, what nursing intervention is most appropriate to apply?
During the acute phase of a strain or sprain, what nursing intervention is most appropriate to apply?
What is the best position for a patient with a sprain or strain?
What is the best position for a patient with a sprain or strain?
Following a fracture, what assessment finding should a nurse immediately report?
Following a fracture, what assessment finding should a nurse immediately report?
3 million fractures occur annually in the US. Fractures in people 65 years of age or older are usually caused by what?
3 million fractures occur annually in the US. Fractures in people 65 years of age or older are usually caused by what?
In addition to pain and physical deformity; swelling and ________ are to be expected when a patient obtains an injury such as a fracture.
In addition to pain and physical deformity; swelling and ________ are to be expected when a patient obtains an injury such as a fracture.
What is one action, out of a variety of nursing interventions, a nurse would do for a patient with a bone fracture?
What is one action, out of a variety of nursing interventions, a nurse would do for a patient with a bone fracture?
What is the most common compression neuropathy of the hands?
What is the most common compression neuropathy of the hands?
The lateral meniscus tears with sliding and/or what type of force?
The lateral meniscus tears with sliding and/or what type of force?
While range of motion exercises are important, what other nursing intervention is important for a patient with bone fractures?
While range of motion exercises are important, what other nursing intervention is important for a patient with bone fractures?
During the nursing interventions for an amputation, why would you contact an occupational therapist?
During the nursing interventions for an amputation, why would you contact an occupational therapist?
What assessment is important for patients with Carpal Tunnel Syndrome?
What assessment is important for patients with Carpal Tunnel Syndrome?
What is an appropriate action a nurse can take for a patient diagnosed with Carpal Tunnel Syndrome?
What is an appropriate action a nurse can take for a patient diagnosed with Carpal Tunnel Syndrome?
During the interventions for amputations, what should the nurse remember about applying ice?
During the interventions for amputations, what should the nurse remember about applying ice?
What is a cause of fractures for young people ages 12-21?
What is a cause of fractures for young people ages 12-21?
What is the main difference between an open and closed fracture?
What is the main difference between an open and closed fracture?
How would a nurse evaluate activity intolerance in a patient with a femur fracture?
How would a nurse evaluate activity intolerance in a patient with a femur fracture?
What can a nurse do to prevent respiratory problems related to bone fractures?
What can a nurse do to prevent respiratory problems related to bone fractures?
Which meniscus tears with excessive twisting or rotational force?
Which meniscus tears with excessive twisting or rotational force?
What is an action the nurse initiates for a patient with a meniscus injury?
What is an action the nurse initiates for a patient with a meniscus injury?
A patient is experiencing pain, decreased grip strength and muscle atrophy in their hand. What is their diagnosis?
A patient is experiencing pain, decreased grip strength and muscle atrophy in their hand. What is their diagnosis?
What part of the body is removed during an amputation?
What part of the body is removed during an amputation?
What are the 2 categories of amputations?
What are the 2 categories of amputations?
What teaching is important for the nurse to initiate for a patient undergoing an amputation?
What teaching is important for the nurse to initiate for a patient undergoing an amputation?
Which primary function is associated with the gastrointestinal system?
Which primary function is associated with the gastrointestinal system?
Which of the following anatomical structures is primarily where digestion begins?
Which of the following anatomical structures is primarily where digestion begins?
What is the key function of the esophagus in the digestive system?
What is the key function of the esophagus in the digestive system?
Which of the following describes the primary role of the stomach in digestion?
Which of the following describes the primary role of the stomach in digestion?
What is one of the main functions of the liver related to digestion?
What is one of the main functions of the liver related to digestion?
What substances does the gallbladder primarily store and concentrate?
What substances does the gallbladder primarily store and concentrate?
What is the main digestive function of the pancreas?
What is the main digestive function of the pancreas?
Which process primarily takes place in the small intestine?
Which process primarily takes place in the small intestine?
What main role does the large intestine play in digestion?
What main role does the large intestine play in digestion?
What components constitute a comprehensive assessment of gastrointestinal function?
What components constitute a comprehensive assessment of gastrointestinal function?
The Malnutrition Universal Screening Tool (MUST) incorporates what key parameters to assess nutritional risk?
The Malnutrition Universal Screening Tool (MUST) incorporates what key parameters to assess nutritional risk?
When performing a physical examination focusing on gastrointestinal function, what is the primary sequence of techniques used?
When performing a physical examination focusing on gastrointestinal function, what is the primary sequence of techniques used?
During the inspection phase of a gastrointestinal assessment, what specific aspects of the oral mucosa are crucial to examine?
During the inspection phase of a gastrointestinal assessment, what specific aspects of the oral mucosa are crucial to examine?
What is indicated by tympany during percussion of the abdomen?
What is indicated by tympany during percussion of the abdomen?
During abdominal palpation, what is the primary purpose of assessing abdominal tone?
During abdominal palpation, what is the primary purpose of assessing abdominal tone?
What is the most likely finding during palpation in a patient with splenomegaly?
What is the most likely finding during palpation in a patient with splenomegaly?
In gastrointestinal diagnostics, what is the primary purpose of stool studies?
In gastrointestinal diagnostics, what is the primary purpose of stool studies?
What type of diagnostic is ultrasonography?
What type of diagnostic is ultrasonography?
What is the primary purpose of barium studies in the assessment of gastrointestinal function?
What is the primary purpose of barium studies in the assessment of gastrointestinal function?
What information from Ms. Mackey's case is MOST concerning and warrants immediate follow-up by the registered nurse?
What information from Ms. Mackey's case is MOST concerning and warrants immediate follow-up by the registered nurse?
What subjective and objective data support the order for fecal occult blood testing in Ms. Mackey's case?
What subjective and objective data support the order for fecal occult blood testing in Ms. Mackey's case?
Following a positive fecal occult blood test, what is the rationale for the healthcare provider ordering a colonoscopy for Ms. Mackey?
Following a positive fecal occult blood test, what is the rationale for the healthcare provider ordering a colonoscopy for Ms. Mackey?
Based on Ms. Mackey's current medication regimen (lisinopril, atorvastatin, furosemide) and her reports of lightheadedness and reduced fluid intake, what electrolyte imbalance would the nurse be MOST vigilant in monitoring?
Based on Ms. Mackey's current medication regimen (lisinopril, atorvastatin, furosemide) and her reports of lightheadedness and reduced fluid intake, what electrolyte imbalance would the nurse be MOST vigilant in monitoring?
What dietary modification is MOST directly indicated by Ms. Mackey's history of adjusting to new dentures and reporting difficulty chewing?
What dietary modification is MOST directly indicated by Ms. Mackey's history of adjusting to new dentures and reporting difficulty chewing?
Given Ms. Mackey's history and physical examination, which factor MOST likely contributed to the finding of 'dullness noted in left lower quadrant on percussion'?
Given Ms. Mackey's history and physical examination, which factor MOST likely contributed to the finding of 'dullness noted in left lower quadrant on percussion'?
What would be the reasoning for Ms. Mackey's doctor to refer her for nutrition management and treatment for hemorrhoids, based on the case study?
What would be the reasoning for Ms. Mackey's doctor to refer her for nutrition management and treatment for hemorrhoids, based on the case study?
Ms. Mackey's diagnostic tests revealed a Hemoglobin (Hgb) of 9 g/dL and Hematocrit (HCT) of 27%. What is the MOST concerning aspect of these findings?
Ms. Mackey's diagnostic tests revealed a Hemoglobin (Hgb) of 9 g/dL and Hematocrit (HCT) of 27%. What is the MOST concerning aspect of these findings?
Ms. Mackey had a colonoscopy revealing several polyps. Which patient statement indicates an understanding of having polyps?
Ms. Mackey had a colonoscopy revealing several polyps. Which patient statement indicates an understanding of having polyps?
Ms. Mackey reports that she usually drinks prune juice for constipation, but it is not working at this point. What should the nurse suggest to Ms. Mackey to better help relieve her constipation?
Ms. Mackey reports that she usually drinks prune juice for constipation, but it is not working at this point. What should the nurse suggest to Ms. Mackey to better help relieve her constipation?
Ms. Mackey reports feeling light-headed when moving from sitting to standing, along with taking furosemide. What is the most appropriate initial nursing action?
Ms. Mackey reports feeling light-headed when moving from sitting to standing, along with taking furosemide. What is the most appropriate initial nursing action?
Following Ms. Mackey's colonoscopy, the nurse finds that there are several hemorrhoids. What should the nurse include when educating Ms. Mackey?
Following Ms. Mackey's colonoscopy, the nurse finds that there are several hemorrhoids. What should the nurse include when educating Ms. Mackey?
Ms. Mackey returns to the unit after a colonoscopy procedure. The nurse will prioritize the interventions in which order?
Ms. Mackey returns to the unit after a colonoscopy procedure. The nurse will prioritize the interventions in which order?
What percentage range of the population in Western countries is estimated to have Gastroesophageal Reflux Disease (GERD)?
What percentage range of the population in Western countries is estimated to have Gastroesophageal Reflux Disease (GERD)?
Which pathophysiological factor directly contributes to the development of Gastroesophageal Reflux Disease (GERD)?
Which pathophysiological factor directly contributes to the development of Gastroesophageal Reflux Disease (GERD)?
Which of the following is a less common clinical manifestation of GERD?
Which of the following is a less common clinical manifestation of GERD?
Which lifestyle factor is known to increase the risk of developing GERD?
Which lifestyle factor is known to increase the risk of developing GERD?
A patient with suspected GERD is undergoing diagnostic testing. Which test involves monitoring esophageal pH over a 24-hour period?
A patient with suspected GERD is undergoing diagnostic testing. Which test involves monitoring esophageal pH over a 24-hour period?
Which category of medications is commonly used in the management of GERD to reduce gastric acid production?
Which category of medications is commonly used in the management of GERD to reduce gastric acid production?
A patient is scheduled for a surgical procedure to correct GERD. Which surgical intervention involves wrapping the fundus of the stomach around the esophagus?
A patient is scheduled for a surgical procedure to correct GERD. Which surgical intervention involves wrapping the fundus of the stomach around the esophagus?
Which of the following is a potential complication of long-standing GERD involving changes in the esophageal lining?
Which of the following is a potential complication of long-standing GERD involving changes in the esophageal lining?
A patient with GERD is experiencing respiratory symptoms. What nursing assessment is most relevant to this manifestation?
A patient with GERD is experiencing respiratory symptoms. What nursing assessment is most relevant to this manifestation?
A nurse is educating a patient with GERD on lifestyle modifications. Which of the following instructions is MOST appropriate?
A nurse is educating a patient with GERD on lifestyle modifications. Which of the following instructions is MOST appropriate?
Following interventions, what is a primary outcome to evaluate in the nursing management of a patient with GERD?
Following interventions, what is a primary outcome to evaluate in the nursing management of a patient with GERD?
Which of these signs and symptoms would the nurse correlate to a Type I hiatal hernia?
Which of these signs and symptoms would the nurse correlate to a Type I hiatal hernia?
When teaching a patient about esophageal manometry, which statement by the nurse is most accurate?
When teaching a patient about esophageal manometry, which statement by the nurse is most accurate?
A patient with a hiatal hernia and GERD asks which activity to avoid. Which response by the nurse is most appropriate?
A patient with a hiatal hernia and GERD asks which activity to avoid. Which response by the nurse is most appropriate?
When providing dietary education for a patient with GERD, which of the following should the nurse include?
When providing dietary education for a patient with GERD, which of the following should the nurse include?
According to epidemiological data, what age group accounts for the majority of hiatal hernia cases?
According to epidemiological data, what age group accounts for the majority of hiatal hernia cases?
Which of the following accurately describes the pathophysiology of a hiatal hernia?
Which of the following accurately describes the pathophysiology of a hiatal hernia?
Residing in which region is considered a risk factor for developing a hiatal hernia?
Residing in which region is considered a risk factor for developing a hiatal hernia?
Which specific clinical manifestation is commonly associated with Type 2 hiatal hernias?
Which specific clinical manifestation is commonly associated with Type 2 hiatal hernias?
Which diagnostic procedure involves using a contrast medium to visualize the esophagus and stomach?
Which diagnostic procedure involves using a contrast medium to visualize the esophagus and stomach?
Which class of medications is prescribed to alleviate the symptoms of a hiatal hernia?
Which class of medications is prescribed to alleviate the symptoms of a hiatal hernia?
Which surgical intervention is typically performed to correct a hiatal hernia?
Which surgical intervention is typically performed to correct a hiatal hernia?
Which potential complication is associated with hiatal hernias?
Which potential complication is associated with hiatal hernias?
Following a Nissen fundoplication, what is the rationale for helping a patient to understand post operative education?
Following a Nissen fundoplication, what is the rationale for helping a patient to understand post operative education?
What is a nursing diagnosis that could affect a patient with a hiatal hernia?
What is a nursing diagnosis that could affect a patient with a hiatal hernia?
What nursing interventions are important for patients with a hiatal hernia?
What nursing interventions are important for patients with a hiatal hernia?
What is an important action to implement for a patient with a hiatal hernia?
What is an important action to implement for a patient with a hiatal hernia?
When teaching a patient with a hiatal hernia, what should the nurse empathize to reduce symptoms?
When teaching a patient with a hiatal hernia, what should the nurse empathize to reduce symptoms?
Which of the following is critical in evaluating care outcomes for a patient with a hiatal hernia?
Which of the following is critical in evaluating care outcomes for a patient with a hiatal hernia?
Which finding is MOST indicative of a hiatal hernia?
Which finding is MOST indicative of a hiatal hernia?
What is are MOST common clinical manifestations for GERD?
What is are MOST common clinical manifestations for GERD?
What is TRUE about the epidemiology of GERD?
What is TRUE about the epidemiology of GERD?
What is a key difference in the pathophysiology of gastritis compared to gastroenteritis relating to fluid balance?
What is a key difference in the pathophysiology of gastritis compared to gastroenteritis relating to fluid balance?
A patient with gastritis reports changes in the color of stool. Which of the following assessment findings would be most concerning and warrant immediate notification of the provider?
A patient with gastritis reports changes in the color of stool. Which of the following assessment findings would be most concerning and warrant immediate notification of the provider?
Which diagnostic test is most useful for confirming the presence of H. pylori in a patient with suspected gastritis?
Which diagnostic test is most useful for confirming the presence of H. pylori in a patient with suspected gastritis?
A patient admitted with acute gastritis is prescribed intravenous fluids. What assessment finding would indicate a therapeutic response to this intervention?
A patient admitted with acute gastritis is prescribed intravenous fluids. What assessment finding would indicate a therapeutic response to this intervention?
A patient recovering from gastritis asks about dietary modifications to prevent recurrence. Which dietary recommendation is most appropriate?
A patient recovering from gastritis asks about dietary modifications to prevent recurrence. Which dietary recommendation is most appropriate?
What is the primary pathophysiological mechanism behind the rapid onset of clinical manifestations in gastroenteritis?
What is the primary pathophysiological mechanism behind the rapid onset of clinical manifestations in gastroenteritis?
When caring for a patient with suspected gastroenteritis, which assessment finding would be most indicative of severe dehydration?
When caring for a patient with suspected gastroenteritis, which assessment finding would be most indicative of severe dehydration?
A patient with gastroenteritis is diagnosed via stool culture with a bacterial infection. What treatment would the provider order?
A patient with gastroenteritis is diagnosed via stool culture with a bacterial infection. What treatment would the provider order?
A nurse is providing discharge teaching to a patient recovering from gastroenteritis. Which instruction regarding food preparation is most important to emphasize?
A nurse is providing discharge teaching to a patient recovering from gastroenteritis. Which instruction regarding food preparation is most important to emphasize?
The nurse is assessing a patient recovering from gastroenteritis. Which outcome indicates effective management of fluid losses?
The nurse is assessing a patient recovering from gastroenteritis. Which outcome indicates effective management of fluid losses?
What is the primary pathophysiological process involved in peptic ulcer disease (PUD)?
What is the primary pathophysiological process involved in peptic ulcer disease (PUD)?
A patient with a peptic ulcer reports that the pain is triggered and worsened by eating. Where is ulcer most likely located?
A patient with a peptic ulcer reports that the pain is triggered and worsened by eating. Where is ulcer most likely located?
A patient with suspected PUD is undergoing an upper GI endoscopy. What is the primary purpose of this diagnostic procedure?
A patient with suspected PUD is undergoing an upper GI endoscopy. What is the primary purpose of this diagnostic procedure?
A patient with PUD is prescribed antacids. What is the primary mechanism of action for antacids?
A patient with PUD is prescribed antacids. What is the primary mechanism of action for antacids?
A patient with PUD develops a GI hemorrhage. Which assessment finding is most indicative of this complication?
A patient with PUD develops a GI hemorrhage. Which assessment finding is most indicative of this complication?
A patient is being assessed for gastritis. Which factor in the patient's history is MOST likely associated with development of this condition?
A patient is being assessed for gastritis. Which factor in the patient's history is MOST likely associated with development of this condition?
A patient with acute gastritis is at risk for deficient fluid volume. Which nursing intervention is most appropriate to include in the plan of care?
A patient with acute gastritis is at risk for deficient fluid volume. Which nursing intervention is most appropriate to include in the plan of care?
A patient admitted with gastroenteritis is experiencing acute abdominal pain. Which nursing intervention is most appropriate to provide comfort?
A patient admitted with gastroenteritis is experiencing acute abdominal pain. Which nursing intervention is most appropriate to provide comfort?
An elderly patient is hospitalized with gastroenteritis. Which of the following demonstrates successful preventive behavior regarding transmission of infection?
An elderly patient is hospitalized with gastroenteritis. Which of the following demonstrates successful preventive behavior regarding transmission of infection?
A patient with peptic ulcer disease (PUD) is scheduled for a vagotomy. What mechanism explains how this procedure helps manage PUD?
A patient with peptic ulcer disease (PUD) is scheduled for a vagotomy. What mechanism explains how this procedure helps manage PUD?
A patient is being taught about peptic ulcer disease and strategies to minimize symptoms. What information should be included in the teaching?
A patient is being taught about peptic ulcer disease and strategies to minimize symptoms. What information should be included in the teaching?
Which of the following is an interprofessional intervention for treating Gastritis, Gastroenteritis, and Peptic Ulcer Disease?
Which of the following is an interprofessional intervention for treating Gastritis, Gastroenteritis, and Peptic Ulcer Disease?
Which laboratory assessment is crucial for a patient with gastritis or peptic ulcer disease who presents with coffee-ground emesis?
Which laboratory assessment is crucial for a patient with gastritis or peptic ulcer disease who presents with coffee-ground emesis?
In the differential diagnosis of abdominal pain, what characteristic helps distinguish gastritis from gastroenteritis?
In the differential diagnosis of abdominal pain, what characteristic helps distinguish gastritis from gastroenteritis?
A patient is being discharged after treatment for gastroenteritis. What sign would the nurse teach the patient to report indicating poor oral intake tolerance?
A patient is being discharged after treatment for gastroenteritis. What sign would the nurse teach the patient to report indicating poor oral intake tolerance?
A patient with Peptic Ulcer Disease is evaluated for perforation of the ulcer. Which clinical manifestation would be most important to monitor?
A patient with Peptic Ulcer Disease is evaluated for perforation of the ulcer. Which clinical manifestation would be most important to monitor?
A patient being treated for a peptic ulcer asks why their pain is worse when they haven't eaten. How does the nurse respond?
A patient being treated for a peptic ulcer asks why their pain is worse when they haven't eaten. How does the nurse respond?
What are the symptoms a patient should focus on reporting quickly to the provider when diagnosed with gastritis?
What are the symptoms a patient should focus on reporting quickly to the provider when diagnosed with gastritis?
What can a nurse teach a patient to reduce risk of Gastritis?
What can a nurse teach a patient to reduce risk of Gastritis?
Which is a goal a nurse wants to see for a patient recovering from gastritis?
Which is a goal a nurse wants to see for a patient recovering from gastritis?
What can cause gastroenteritis?
What can cause gastroenteritis?
A patient prescribed medication for PUD is experiencing mild nausea. How should a nurse respond to a patient's concern about taking the medication?
A patient prescribed medication for PUD is experiencing mild nausea. How should a nurse respond to a patient's concern about taking the medication?
What intervention will a provider implement when a patient with PUD has chronic non-healing of ulcers?
What intervention will a provider implement when a patient with PUD has chronic non-healing of ulcers?
What is true about a duodenal ulcer?
What is true about a duodenal ulcer?
What are complications of peptic ulcer disease?
What are complications of peptic ulcer disease?
A patient undergoing assessment for PUD reports the use of alcohol and medication use. Which statement made by the patient can the nurse teach further to help with the condition?
A patient undergoing assessment for PUD reports the use of alcohol and medication use. Which statement made by the patient can the nurse teach further to help with the condition?
Which nursing diagnosis is most applicable to a patient presenting with acute gastritis and persistent vomiting?
Which nursing diagnosis is most applicable to a patient presenting with acute gastritis and persistent vomiting?
A patient diagnosed with gastritis is prescribed a medication regimen. What key teaching point should the nurse emphasize regarding adherence?
A patient diagnosed with gastritis is prescribed a medication regimen. What key teaching point should the nurse emphasize regarding adherence?
A patient with gastroenteritis is being discharged. Which of the following instructions regarding dietary modifications is MOST appropriate for the nurse to include in the discharge teaching?
A patient with gastroenteritis is being discharged. Which of the following instructions regarding dietary modifications is MOST appropriate for the nurse to include in the discharge teaching?
A patient with peptic ulcer disease reports persistent epigastric pain, despite adhering to the prescribed medication and dietary restrictions. What is the MOST appropriate initial nursing intervention?
A patient with peptic ulcer disease reports persistent epigastric pain, despite adhering to the prescribed medication and dietary restrictions. What is the MOST appropriate initial nursing intervention?
When caring for a patient with gastritis, what assessment finding would indicate the HIGHEST risk for deficient fluid volume?
When caring for a patient with gastritis, what assessment finding would indicate the HIGHEST risk for deficient fluid volume?
A patient with gastroenteritis reports increased abdominal distention. What is the priority nursing intervention when assessing this patient?
A patient with gastroenteritis reports increased abdominal distention. What is the priority nursing intervention when assessing this patient?
Which of the following complications is MOST likely to occur if a peptic ulcer perforates?
Which of the following complications is MOST likely to occur if a peptic ulcer perforates?
What information is essential for the nurse to include when teaching a patient about managing gastritis to prevent recurrence?
What information is essential for the nurse to include when teaching a patient about managing gastritis to prevent recurrence?
A patient experiencing persistent diarrhea due to gastroenteritis is at risk. Which Electrolyte should the nurse monitor most closely?
A patient experiencing persistent diarrhea due to gastroenteritis is at risk. Which Electrolyte should the nurse monitor most closely?
Which of the following signs and symptoms are associated with a patient diagnosed with Peptic Ulcer Disease?
Which of the following signs and symptoms are associated with a patient diagnosed with Peptic Ulcer Disease?
A healthcare provider explains to Mrs. Taylor that the endoscopy performed allows visualization of the esophageal, gastric, and duodenal mucosal linings, as well as allows for what else to occur?
A healthcare provider explains to Mrs. Taylor that the endoscopy performed allows visualization of the esophageal, gastric, and duodenal mucosal linings, as well as allows for what else to occur?
How does obesity contribute to the risk of developing a hernia?
How does obesity contribute to the risk of developing a hernia?
How would the nurse differentiate between a reducible and incarcerated hernia during a physical examination?
How would the nurse differentiate between a reducible and incarcerated hernia during a physical examination?
Following a hernia repair, why is it important for the nurse to discourage coughing in the patient?
Following a hernia repair, why is it important for the nurse to discourage coughing in the patient?
What is the rationale behind advising patients with hemorrhoids to avoid straining during bowel movements?
What is the rationale behind advising patients with hemorrhoids to avoid straining during bowel movements?
Why are stimulant laxatives generally discouraged for long-term management of hemorrhoids?
Why are stimulant laxatives generally discouraged for long-term management of hemorrhoids?
What is the significance of monitoring bowel habits and intake/output postoperatively in a patient who has undergone hemorrhoidectomy?
What is the significance of monitoring bowel habits and intake/output postoperatively in a patient who has undergone hemorrhoidectomy?
What is the underlying reason for the recommendation to keep a food diary for individuals with irritable bowel syndrome (IBS)?
What is the underlying reason for the recommendation to keep a food diary for individuals with irritable bowel syndrome (IBS)?
What is the rationale for establishing a regular bowel routine as part of the nursing interventions for patients with irritable bowel syndrome (IBS)?
What is the rationale for establishing a regular bowel routine as part of the nursing interventions for patients with irritable bowel syndrome (IBS)?
Why is a psychosocial assessment an important component of the nursing assessment for patients with Irritable Bowel Syndrome (IBS)?
Why is a psychosocial assessment an important component of the nursing assessment for patients with Irritable Bowel Syndrome (IBS)?
What is the primary rationale for encouraging smaller, more frequent meals for patients with inflammatory bowel disease (IBD)?
What is the primary rationale for encouraging smaller, more frequent meals for patients with inflammatory bowel disease (IBD)?
Why is it important to provide meticulous skin care for patients with inflammatory bowel disease (IBD) who are experiencing frequent diarrhea?
Why is it important to provide meticulous skin care for patients with inflammatory bowel disease (IBD) who are experiencing frequent diarrhea?
What is the goal of encouraging patients with inflammatory bowel disease (IBD) to engage in mealtime with family?
What is the goal of encouraging patients with inflammatory bowel disease (IBD) to engage in mealtime with family?
What is the rationale for using a dishwasher as a nursing intervention when teaching patients with Celiac Disease?
What is the rationale for using a dishwasher as a nursing intervention when teaching patients with Celiac Disease?
Why is it important for patients with Celiac disease to have long-term follow-up and interdisciplinary management?
Why is it important for patients with Celiac disease to have long-term follow-up and interdisciplinary management?
What is the significance of a Dual-energy X-ray absorptiometry (DXA) scan in the assessment of a patient with Celiac Disease?
What is the significance of a Dual-energy X-ray absorptiometry (DXA) scan in the assessment of a patient with Celiac Disease?
What is the underlying cause of diverticulitis?
What is the underlying cause of diverticulitis?
Why is bowel rest, often achieved with NPO status, a common component of the medical management for acute diverticulitis?
Why is bowel rest, often achieved with NPO status, a common component of the medical management for acute diverticulitis?
What information would the nurse provide to a patient regarding dietary recommendations for managing diverticulitis?
What information would the nurse provide to a patient regarding dietary recommendations for managing diverticulitis?
Which of the following best explains the pathophysiology of appendicitis?
Which of the following best explains the pathophysiology of appendicitis?
Why is it essential to keep a patient NPO prior to appendectomy?
Why is it essential to keep a patient NPO prior to appendectomy?
Following an appendectomy, what instruction should the nurse provide to the patient regarding antibiotic use?
Following an appendectomy, what instruction should the nurse provide to the patient regarding antibiotic use?
Considering the pathophysiology of ulcerative colitis, how does colonoscopy aid in its diagnosis?
Considering the pathophysiology of ulcerative colitis, how does colonoscopy aid in its diagnosis?
In caring for a patient with ulcerative colitis and frequent bloody stools, which laboratory finding would be a priority to communicate to the healthcare provider?
In caring for a patient with ulcerative colitis and frequent bloody stools, which laboratory finding would be a priority to communicate to the healthcare provider?
Considering that clinical manifestations of ulcerative colitis fluctuate, how should the nurse advise the patient regarding fluid intake during periods of heightened diarrhea?
Considering that clinical manifestations of ulcerative colitis fluctuate, how should the nurse advise the patient regarding fluid intake during periods of heightened diarrhea?
Which of the following activities would have the greatest association with the development of a hernia?
Which of the following activities would have the greatest association with the development of a hernia?
Following a herniorrhaphy, which of these instructions are important for a nurse to give to a client?
Following a herniorrhaphy, which of these instructions are important for a nurse to give to a client?
Which assessment finding should the nurse prioritize in a client who is one day post-operative following a hemorrhoidectomy?
Which assessment finding should the nurse prioritize in a client who is one day post-operative following a hemorrhoidectomy?
What is the goal of encouraging regular exercise for patients who are suffering irritable bowel syndrome?
What is the goal of encouraging regular exercise for patients who are suffering irritable bowel syndrome?
It is important to establish a therapeutic relationship with a patient with Inflammatory Bowel Disease because...
It is important to establish a therapeutic relationship with a patient with Inflammatory Bowel Disease because...
What is the most important measure a nurse can take in order to assess the current knowledge a client has about the gluten-free diet for celiac patients?
What is the most important measure a nurse can take in order to assess the current knowledge a client has about the gluten-free diet for celiac patients?
In addition to broad spectrum antibiotics, which measure should the nurse prioritize for a patient who is diagnosed with diverticulitis?
In addition to broad spectrum antibiotics, which measure should the nurse prioritize for a patient who is diagnosed with diverticulitis?
Which symptom should be prioritized by the nurse after a client receives antibiotics for diverticulitis?
Which symptom should be prioritized by the nurse after a client receives antibiotics for diverticulitis?
A test to assess the abdomen demonstrates a blockage of the opening of the appendix. Which pathophysiological description describes this patient?
A test to assess the abdomen demonstrates a blockage of the opening of the appendix. Which pathophysiological description describes this patient?
What is the overall evaluation goal a nurse should have for a patient being discharged from appendicitis?
What is the overall evaluation goal a nurse should have for a patient being discharged from appendicitis?
A client presents with "droplets of fat" in the stool along with bloody stool. What cue indicates that this patient is experiencing?
A client presents with "droplets of fat" in the stool along with bloody stool. What cue indicates that this patient is experiencing?
A client presents with high fever. A test is ordered and ulcerations are found in the test. Which other piece of data must be accounted for in order to diagnose ulcerative colitis?
A client presents with high fever. A test is ordered and ulcerations are found in the test. Which other piece of data must be accounted for in order to diagnose ulcerative colitis?
A client has been diagnosed with ulcerative colitis. Should the nurse recommend a large meal at once or to "Spread out your meals to six times per a day."
A client has been diagnosed with ulcerative colitis. Should the nurse recommend a large meal at once or to "Spread out your meals to six times per a day."
A patient is scheduled for an inguinal hernia repair. What preoperative teaching should the nurse prioritize to minimize postoperative complications?
A patient is scheduled for an inguinal hernia repair. What preoperative teaching should the nurse prioritize to minimize postoperative complications?
A patient with an incarcerated hernia reports severe abdominal pain, nausea, and vomiting. Which nursing action is the MOST critical?
A patient with an incarcerated hernia reports severe abdominal pain, nausea, and vomiting. Which nursing action is the MOST critical?
A post-operative hernia repair patient reports increased pain at the incision site and a low-grade fever. What is the priority nursing intervention?
A post-operative hernia repair patient reports increased pain at the incision site and a low-grade fever. What is the priority nursing intervention?
Which instruction is MOST important for a patient being discharged after a hemorrhoidectomy?
Which instruction is MOST important for a patient being discharged after a hemorrhoidectomy?
A patient with hemorrhoids reports significant rectal pain and bleeding during bowel movements. Which intervention is most appropriate?
A patient with hemorrhoids reports significant rectal pain and bleeding during bowel movements. Which intervention is most appropriate?
During a routine assessment, a patient reports mild but persistent rectal itching. How should the nurse proceed to gather more data about potential hemorrhoids?
During a routine assessment, a patient reports mild but persistent rectal itching. How should the nurse proceed to gather more data about potential hemorrhoids?
A patient diagnosed with Irritable Bowel Syndrome (IBS) reports frequent abdominal bloating and gas. What dietary modification should the nurse recommend?
A patient diagnosed with Irritable Bowel Syndrome (IBS) reports frequent abdominal bloating and gas. What dietary modification should the nurse recommend?
A patient with IBS is experiencing increased stress, which exacerbates their symptoms. Which intervention is MOST appropriate?
A patient with IBS is experiencing increased stress, which exacerbates their symptoms. Which intervention is MOST appropriate?
A patient with Irritable Bowel Syndrome (IBS) is being discharged. What statement ensures the patient understands the self-management of their condition?
A patient with Irritable Bowel Syndrome (IBS) is being discharged. What statement ensures the patient understands the self-management of their condition?
A patient with Inflammatory Bowel Disease (IBD) is prescribed sulfasalazine. What should the nurse emphasize regarding medication management?
A patient with Inflammatory Bowel Disease (IBD) is prescribed sulfasalazine. What should the nurse emphasize regarding medication management?
A patient with IBD is experiencing an acute exacerbation of symptoms, including severe diarrhea and abdominal pain. What is the PRIORITY nursing intervention?
A patient with IBD is experiencing an acute exacerbation of symptoms, including severe diarrhea and abdominal pain. What is the PRIORITY nursing intervention?
A patient with Inflammatory Bowel Disease (IBD) is scheduled for a colonoscopy. What pre-procedure information is MOST crucial for the nurse to review?
A patient with Inflammatory Bowel Disease (IBD) is scheduled for a colonoscopy. What pre-procedure information is MOST crucial for the nurse to review?
A patient with Celiac Disease is overwhelmed by the complexity of adhering to a gluten-free diet. Which nursing action is MOST helpful?
A patient with Celiac Disease is overwhelmed by the complexity of adhering to a gluten-free diet. Which nursing action is MOST helpful?
Which assessment finding in a patient with Celiac Disease indicates the need for further intervention?
Which assessment finding in a patient with Celiac Disease indicates the need for further intervention?
A patient with Celiac Disease reports feeling socially isolated due to dietary restrictions. What intervention should the nurse suggest?
A patient with Celiac Disease reports feeling socially isolated due to dietary restrictions. What intervention should the nurse suggest?
A patient with diverticulitis is prescribed a clear liquid diet during the acute phase. Which food choice indicates the patient understands the dietary restrictions?
A patient with diverticulitis is prescribed a clear liquid diet during the acute phase. Which food choice indicates the patient understands the dietary restrictions?
What is the rationale for advising a patient with diverticulitis to avoid straining during bowel movements?
What is the rationale for advising a patient with diverticulitis to avoid straining during bowel movements?
A patient diagnosed with diverticulitis is being discharged. What dietary instruction is MOST appropriate?
A patient diagnosed with diverticulitis is being discharged. What dietary instruction is MOST appropriate?
A patient presents to the emergency department with suspected appendicitis. What assessment finding requires immediate notification of the healthcare provider?
A patient presents to the emergency department with suspected appendicitis. What assessment finding requires immediate notification of the healthcare provider?
A patient is scheduled for an appendectomy. What preoperative nursing intervention is of HIGHEST priority?
A patient is scheduled for an appendectomy. What preoperative nursing intervention is of HIGHEST priority?
Following an appendectomy, the patient reports pain and difficulty coughing. What BEST addresses this patient's needs?
Following an appendectomy, the patient reports pain and difficulty coughing. What BEST addresses this patient's needs?
Which intervention is MOST effective for patients after an appendectomy to avoid post-operative complications?
Which intervention is MOST effective for patients after an appendectomy to avoid post-operative complications?
Jack, a 17-year-old, has been diagnosed with ulcerative colitis. Which laboratory value directly correlates with ulcerative colitis?
Jack, a 17-year-old, has been diagnosed with ulcerative colitis. Which laboratory value directly correlates with ulcerative colitis?
What diagnostic test is an appropriate test to evaluate 17-year-old Jack with a diagnosis of Ulcerative Colitis?
What diagnostic test is an appropriate test to evaluate 17-year-old Jack with a diagnosis of Ulcerative Colitis?
Ulcerative colitis may result in which complication?
Ulcerative colitis may result in which complication?
What information should the surgeon discuss with Jack, a 17-year-old with a diagnosis of UIcerative Colitis?
What information should the surgeon discuss with Jack, a 17-year-old with a diagnosis of UIcerative Colitis?
What is a key element in Jack's plan of care post-colectomy surgery?
What is a key element in Jack's plan of care post-colectomy surgery?
A patient experiencing orthostatic hypotension, diarrhea, weight loss, and anemia is MOST likely presenting with symptoms of which intestinal disease?
A patient experiencing orthostatic hypotension, diarrhea, weight loss, and anemia is MOST likely presenting with symptoms of which intestinal disease?
A patient experiencing abdominal pain, nausea, fever, and vomiting with rebound tenderness at McBurney's point is MOST likely experiencing clinical manifestations of which intestinal disorder?
A patient experiencing abdominal pain, nausea, fever, and vomiting with rebound tenderness at McBurney's point is MOST likely experiencing clinical manifestations of which intestinal disorder?
A patient who presents with a bulge or visible swelling in the groin area, pain, nausea, vomiting, fever and tachycardia, is MOST likely presenting with symptoms of which intestinal disorder?
A patient who presents with a bulge or visible swelling in the groin area, pain, nausea, vomiting, fever and tachycardia, is MOST likely presenting with symptoms of which intestinal disorder?
A patient with a previous history of Crohn's Disease is MOST likely considered to have which intestinal disorder?
A patient with a previous history of Crohn's Disease is MOST likely considered to have which intestinal disorder?
A patient presents with bloody stools, itching and irritation in the anal region, and experiencing pain with bowel movement is MOST likely suffering from:
A patient presents with bloody stools, itching and irritation in the anal region, and experiencing pain with bowel movement is MOST likely suffering from:
A patient experiencing diarrhea, constipation, gas, and abdominal pain is MOST likely suffering from which intestinal disorder?
A patient experiencing diarrhea, constipation, gas, and abdominal pain is MOST likely suffering from which intestinal disorder?
According to the information provided, a patient who routinely strains during bowel movements might experience:
According to the information provided, a patient who routinely strains during bowel movements might experience:
Which of the following factors is a primary cause of obstruction and restricted bloodlfow that leads to appendicitis?
Which of the following factors is a primary cause of obstruction and restricted bloodlfow that leads to appendicitis?
According to the information that a nurse is relaying to a patient, which of the following can increase the complications of a hernia?
According to the information that a nurse is relaying to a patient, which of the following can increase the complications of a hernia?
Which medication is MOST appropriate for a patient to take when admitted with Ulcerative Colitis?
Which medication is MOST appropriate for a patient to take when admitted with Ulcerative Colitis?
Knowing that the exact cause of IBD is unknown, what triggers an inappropriate response in the intestinal tract which causes inflammation?
Knowing that the exact cause of IBD is unknown, what triggers an inappropriate response in the intestinal tract which causes inflammation?
A nurse is creating a teaching plan for a patient with diverticulitis. What should be included in dietary instructions?
A nurse is creating a teaching plan for a patient with diverticulitis. What should be included in dietary instructions?
In regards to stool characteristics, which of the following most accurate in assessing for steatorrhea?
In regards to stool characteristics, which of the following most accurate in assessing for steatorrhea?
What should a nurse include in the teaching instructions to a patient diagnosed with Celiac Disease?
What should a nurse include in the teaching instructions to a patient diagnosed with Celiac Disease?
What is the formula for calculating Body Mass Index (BMI)?
What is the formula for calculating Body Mass Index (BMI)?
According to the NIH classification, what BMI range is considered indicative of 'Obesity (class 2)'?
According to the NIH classification, what BMI range is considered indicative of 'Obesity (class 2)'?
What percentage of adults aged 40-59 are estimated to be affected by obesity?
What percentage of adults aged 40-59 are estimated to be affected by obesity?
A patient with a BMI of 32 is considered to have which of the following?
A patient with a BMI of 32 is considered to have which of the following?
Which of the following is a potential effect of long-term obesity on an individual's health and well-being?
Which of the following is a potential effect of long-term obesity on an individual's health and well-being?
What role does leptin play in the pathophysiology of obesity?
What role does leptin play in the pathophysiology of obesity?
A patient with obesity is undergoing diagnostic testing. What is the primary purpose of these tests?
A patient with obesity is undergoing diagnostic testing. What is the primary purpose of these tests?
Which of the following is a key component of the interprofessional management of obesity?
Which of the following is a key component of the interprofessional management of obesity?
Which surgical management approach for obesity involves creating a small gastric pouch and bypassing a portion of the small intestine?
Which surgical management approach for obesity involves creating a small gastric pouch and bypassing a portion of the small intestine?
What is a potential complication following bariatric surgery?
What is a potential complication following bariatric surgery?
A patient is two days post-op from bariatric surgery. Which intervention is most important for the nurse to implement?
A patient is two days post-op from bariatric surgery. Which intervention is most important for the nurse to implement?
Which nursing diagnosis is most likely to be included in the care plan of a patient following bariatric surgery?
Which nursing diagnosis is most likely to be included in the care plan of a patient following bariatric surgery?
What nursing assessment is crucial to include when caring for a patient after bariatric surgery?
What nursing assessment is crucial to include when caring for a patient after bariatric surgery?
Which action should the nurse prioritize when providing care to a patient who has undergone bariatric surgery?
Which action should the nurse prioritize when providing care to a patient who has undergone bariatric surgery?
A post-operative teaching plan for a patient who underwent bariatric surgery should include information about:
A post-operative teaching plan for a patient who underwent bariatric surgery should include information about:
Following bariatric surgery, what potential issue should be included in discharge teaching to ensure the patient reports to their healthcare provider?
Following bariatric surgery, what potential issue should be included in discharge teaching to ensure the patient reports to their healthcare provider?
In evaluating care outcomes for a patient post-bariatric surgery, which of these options best suggests treatment success?
In evaluating care outcomes for a patient post-bariatric surgery, which of these options best suggests treatment success?
Lauren Wiley is 5 ft 6 in. tall and weighs 256 lb. What is Ms. Wiley's BMI?
Lauren Wiley is 5 ft 6 in. tall and weighs 256 lb. What is Ms. Wiley's BMI?
Ms. Wiley expresses her concerns about being approved for bariatric surgery. Which of the following is the best response by the nurse?
Ms. Wiley expresses her concerns about being approved for bariatric surgery. Which of the following is the best response by the nurse?
Ms. Wiley wants to understand the key difference between Roux-en-Y gastric bypass (RYGB) and adjustable gastric banding. Which explanation by the nurse is most accurate?
Ms. Wiley wants to understand the key difference between Roux-en-Y gastric bypass (RYGB) and adjustable gastric banding. Which explanation by the nurse is most accurate?
After expressing her anxiety about possible complications of bariatric surgery, Ms. Wiley seeks reassurance from the nurse. Which action by the nurse is best?
After expressing her anxiety about possible complications of bariatric surgery, Ms. Wiley seeks reassurance from the nurse. Which action by the nurse is best?
Ms. Wiley is 2 hours post-gastric bypass surgery and has scant blood-tinged drainage coming from her nasogastric tube (NGT). The orders state that the NGT should be hooked up to low continuous suction. What should the nurse do?
Ms. Wiley is 2 hours post-gastric bypass surgery and has scant blood-tinged drainage coming from her nasogastric tube (NGT). The orders state that the NGT should be hooked up to low continuous suction. What should the nurse do?
A patient is diagnosed with metabolic syndrome as a comorbidity of obesity. Which diagnostic criterion is characterized by a waist circumference greater than 40 inches in men?
A patient is diagnosed with metabolic syndrome as a comorbidity of obesity. Which diagnostic criterion is characterized by a waist circumference greater than 40 inches in men?
Leptin, a hormone involved in the pathophysiology of obesity, primarily acts on which part of the brain to regulate appetite?
Leptin, a hormone involved in the pathophysiology of obesity, primarily acts on which part of the brain to regulate appetite?
Which laboratory finding is most indicative of a secondary cause of obesity related to endocrine dysfunction?
Which laboratory finding is most indicative of a secondary cause of obesity related to endocrine dysfunction?
In managing patients with obesity, which component of behavioral therapy focuses on identifying and modifying unhealthy eating patterns and triggers?
In managing patients with obesity, which component of behavioral therapy focuses on identifying and modifying unhealthy eating patterns and triggers?
What is the primary mechanism of 'restrictive' bariatric surgeries in promoting weight loss?
What is the primary mechanism of 'restrictive' bariatric surgeries in promoting weight loss?
What key intervention should the nurse perform to prevent respiratory complications in a patient immediately following bariatric surgery?
What key intervention should the nurse perform to prevent respiratory complications in a patient immediately following bariatric surgery?
Which nursing diagnosis takes highest priority in a patient recovering from bariatric surgery due to the procedure's impact on gastrointestinal anatomy and function?
Which nursing diagnosis takes highest priority in a patient recovering from bariatric surgery due to the procedure's impact on gastrointestinal anatomy and function?
What post-operative complication should a nurse be most vigilant for in a patient who has undergone a Roux-en-Y gastric bypass due to the alteration in gastrointestinal anatomy?
What post-operative complication should a nurse be most vigilant for in a patient who has undergone a Roux-en-Y gastric bypass due to the alteration in gastrointestinal anatomy?
In designing discharge instructions for a patient following bariatric surgery, which teaching point is MOST crucial to include to prevent long-term complications?
In designing discharge instructions for a patient following bariatric surgery, which teaching point is MOST crucial to include to prevent long-term complications?
A patient asks for recommendations on methods or support systems available for managing weight and maintaining lifestyle changes postsurgical procedure. What support is most adequate?
A patient asks for recommendations on methods or support systems available for managing weight and maintaining lifestyle changes postsurgical procedure. What support is most adequate?
What key recommendation should the nurse provide to a patient to avoid development of gallstones after bariatric surgery?
What key recommendation should the nurse provide to a patient to avoid development of gallstones after bariatric surgery?
When working with an obese patient, what is the ideal method for the nurse to maintain a safe environment and lower the chance of injury?
When working with an obese patient, what is the ideal method for the nurse to maintain a safe environment and lower the chance of injury?
What is a realistic outcome from surgery, should the nurse consider when evaluating care outcomes?
What is a realistic outcome from surgery, should the nurse consider when evaluating care outcomes?
When comparing anxiety and fear, which statement accurately differentiates the two?
When comparing anxiety and fear, which statement accurately differentiates the two?
A student awaiting NCLEX results experiences heightened focus and enhanced problem-solving abilities. Which level of anxiety is the student likely experiencing?
A student awaiting NCLEX results experiences heightened focus and enhanced problem-solving abilities. Which level of anxiety is the student likely experiencing?
During a stressful situation, a person begins to focus on only certain aspects of the environment, while their ability to think clearly is somewhat hampered. Which level of anxiety are they most likely experiencing?
During a stressful situation, a person begins to focus on only certain aspects of the environment, while their ability to think clearly is somewhat hampered. Which level of anxiety are they most likely experiencing?
A client in the emergency department exhibits severely reduced perceptual field and confused behavior. Which level of anxiety is the client experiencing?
A client in the emergency department exhibits severely reduced perceptual field and confused behavior. Which level of anxiety is the client experiencing?
A patient experiencing panic-level anxiety is MOST likely to exhibit which of the following behaviors?
A patient experiencing panic-level anxiety is MOST likely to exhibit which of the following behaviors?
A generally healthy adult has developed a new habit of chewing their fingernails to the point of bleeding during work meetings. Which defense mechanism is most likely being employed?
A generally healthy adult has developed a new habit of chewing their fingernails to the point of bleeding during work meetings. Which defense mechanism is most likely being employed?
A child refuses to attend school after repeated bullying incidents and consistently reports vague physical symptoms to stay home. What disorder is the child MOST likely exhibiting?
A child refuses to attend school after repeated bullying incidents and consistently reports vague physical symptoms to stay home. What disorder is the child MOST likely exhibiting?
A patient consistently avoids social gatherings due to intense fear of being judged or scrutinized by others. Which disorder is this patient MOST likely exhibiting?
A patient consistently avoids social gatherings due to intense fear of being judged or scrutinized by others. Which disorder is this patient MOST likely exhibiting?
A person experiences sudden episodes of intense fear accompanied by physical symptoms such as palpitations, shortness of breath, and dizziness. These episodes peak within minutes. Which condition is MOST consistent with these symptoms?
A person experiences sudden episodes of intense fear accompanied by physical symptoms such as palpitations, shortness of breath, and dizziness. These episodes peak within minutes. Which condition is MOST consistent with these symptoms?
An individual reports persistent, excessive worry about various aspects of life, such as work, health, and finances, which lasts for several months. Which diagnosis is MOST consistent with this presentation?
An individual reports persistent, excessive worry about various aspects of life, such as work, health, and finances, which lasts for several months. Which diagnosis is MOST consistent with this presentation?
Which of the following factors can contribute to the development of anxiety disorders?
Which of the following factors can contribute to the development of anxiety disorders?
The nurse is assessing a patient reporting symptoms of anxiety. Which assessment question is MOST important initially?
The nurse is assessing a patient reporting symptoms of anxiety. Which assessment question is MOST important initially?
A patient's care plan includes the goal: 'Patient will demonstrate the ability to use anxiety reduction techniques.' Which observation BEST indicates that the patient is meeting this goal?
A patient's care plan includes the goal: 'Patient will demonstrate the ability to use anxiety reduction techniques.' Which observation BEST indicates that the patient is meeting this goal?
Which nursing intervention is MOST appropriate for a patient experiencing moderate anxiety?
Which nursing intervention is MOST appropriate for a patient experiencing moderate anxiety?
A patient has been attending cognitive-behavioral therapy as an outpatient for anxiety. Which of the following statements BEST reflects a successful outcome of this therapy?
A patient has been attending cognitive-behavioral therapy as an outpatient for anxiety. Which of the following statements BEST reflects a successful outcome of this therapy?
A patient with a history of anxiety is starting behavioral therapy. What technique is MOST likely to be included in the initial sessions?
A patient with a history of anxiety is starting behavioral therapy. What technique is MOST likely to be included in the initial sessions?
A person has recurring thoughts, impulses or images that persist and recur, so that they cannot be dismissed from the mind, what term would you use to describe this?
A person has recurring thoughts, impulses or images that persist and recur, so that they cannot be dismissed from the mind, what term would you use to describe this?
A patient feels driven to perform ritualistic behaviors in an attempt to reduce anxiety, what term would you use to describe these?
A patient feels driven to perform ritualistic behaviors in an attempt to reduce anxiety, what term would you use to describe these?
According to DSM-5 criteria, what key criterion must be met for a diagnosis of obsessive-compulsive disorder?
According to DSM-5 criteria, what key criterion must be met for a diagnosis of obsessive-compulsive disorder?
Which of the following is an example of body-focused repetitive behavior seen in trichotillomania?
Which of the following is an example of body-focused repetitive behavior seen in trichotillomania?
What is a shared risk factor that contributes to both anxiety disorders and obsessive-compulsive disorder (OCD)?
What is a shared risk factor that contributes to both anxiety disorders and obsessive-compulsive disorder (OCD)?
What nursing diagnosis is most applicable for a patient with excoriation disorder?
What nursing diagnosis is most applicable for a patient with excoriation disorder?
Which nursing intervention is emphasized for patients with obsessive-compulsive disorder (OCD) who also have a co-existing urinary tract infection?
Which nursing intervention is emphasized for patients with obsessive-compulsive disorder (OCD) who also have a co-existing urinary tract infection?
A patient diagnosed with OCD is undergoing exposure and response prevention therapy. What key message should the nurse reinforce?
A patient diagnosed with OCD is undergoing exposure and response prevention therapy. What key message should the nurse reinforce?
Which medication classification is typically considered first-line for the treatment of obsessive-compulsive disorder (OCD)?
Which medication classification is typically considered first-line for the treatment of obsessive-compulsive disorder (OCD)?
When a patient diagnosed with OCD is prescribed Clomipramine (TCA), what condition should the nurse monitor for, due to the side effects of the medication?
When a patient diagnosed with OCD is prescribed Clomipramine (TCA), what condition should the nurse monitor for, due to the side effects of the medication?
A patient undergoing exposure and response prevention therapy is gradually exposed to triggers of their OCD symptoms. What should you educate the patient on?
A patient undergoing exposure and response prevention therapy is gradually exposed to triggers of their OCD symptoms. What should you educate the patient on?
Which of the following statements BEST describes the purpose of 'flooding' as a psychological therapy for obsessive-compulsive disorder (OCD)?
Which of the following statements BEST describes the purpose of 'flooding' as a psychological therapy for obsessive-compulsive disorder (OCD)?
What is an adaptive use of the defense mechanism of sublimation?
What is an adaptive use of the defense mechanism of sublimation?
Given the clinical presentation of a patient experiencing a phobia, which intervention aligns with the primary goal of diminishing distress?
Given the clinical presentation of a patient experiencing a phobia, which intervention aligns with the primary goal of diminishing distress?
A patient displays the maladaptive use of the defense mechanism of denial. Which outcome is the MOST appropriate to include in the plan of care?
A patient displays the maladaptive use of the defense mechanism of denial. Which outcome is the MOST appropriate to include in the plan of care?
A known risk factor in the development of body dysmorphic disorder is...
A known risk factor in the development of body dysmorphic disorder is...
Which statement underscores the significance of the advanced practice nurse's role in employing flooding as an OCD treatment?
Which statement underscores the significance of the advanced practice nurse's role in employing flooding as an OCD treatment?
What type of intervention is deep brain stimulation (DBS)?
What type of intervention is deep brain stimulation (DBS)?
In a patient with OCD, which statement by the nurse shows an understanding of the patient's potential need for a psychological therapy?
In a patient with OCD, which statement by the nurse shows an understanding of the patient's potential need for a psychological therapy?
A patient diagnosed with a hoarding disorder is being discharged. What intervention is MOST critical to include to address the patient's challenges with decision-making?
A patient diagnosed with a hoarding disorder is being discharged. What intervention is MOST critical to include to address the patient's challenges with decision-making?
Which statement accurately identifies the genetic component of first-degree relatives inheriting obsessive-compulsive disorder (OCD)?
Which statement accurately identifies the genetic component of first-degree relatives inheriting obsessive-compulsive disorder (OCD)?
Which statement considers a comprehensive plan for the anxiety and risk of self-destructive behavior for a patient diagnosed with trichotillomania?
Which statement considers a comprehensive plan for the anxiety and risk of self-destructive behavior for a patient diagnosed with trichotillomania?
Which statement accurately describes the therapeutic approach for a patient with an obsessive-compulsive disorder (OCD), utilizing a behavioral and cognitive intervention?
Which statement accurately describes the therapeutic approach for a patient with an obsessive-compulsive disorder (OCD), utilizing a behavioral and cognitive intervention?
A patient with moderate anxiety is preparing for a diagnostic procedure. What assessment finding would the nurse most likely observe?
A patient with moderate anxiety is preparing for a diagnostic procedure. What assessment finding would the nurse most likely observe?
A student who always gets good grades is worried about failing an upcoming exam, and starts excessively cleaning their room. Which defense mechanism is the student exhibiting?
A student who always gets good grades is worried about failing an upcoming exam, and starts excessively cleaning their room. Which defense mechanism is the student exhibiting?
A child refuses to go to school due to fear of leaving their parent. Which clinical manifestation is the child demonstrating?
A child refuses to go to school due to fear of leaving their parent. Which clinical manifestation is the child demonstrating?
A veteran is hyper-vigilant, irritable, and has difficulty concentrating, following their deployment. Which risk factor influenced the client's anxiety disorder?
A veteran is hyper-vigilant, irritable, and has difficulty concentrating, following their deployment. Which risk factor influenced the client's anxiety disorder?
During an assessment, a patient reports experiencing sudden, intense fear accompanied by palpitations, dizziness, and a sense of impending doom. What condition are these symptoms most indicative of?
During an assessment, a patient reports experiencing sudden, intense fear accompanied by palpitations, dizziness, and a sense of impending doom. What condition are these symptoms most indicative of?
A patient displays ritualistic behavior of repeatedly checking if the door is locked, causing significant distress and impairment in daily functioning. According to the DSM-5 criteria, this pattern is considered time-consuming if it exceeds how long per day?
A patient displays ritualistic behavior of repeatedly checking if the door is locked, causing significant distress and impairment in daily functioning. According to the DSM-5 criteria, this pattern is considered time-consuming if it exceeds how long per day?
When caring for a patient with an anxiety disorder, which nursing diagnosis addresses the patient's ability to effectively manage stressors and adapt to changing situations?
When caring for a patient with an anxiety disorder, which nursing diagnosis addresses the patient's ability to effectively manage stressors and adapt to changing situations?
Which intervention promotes positive outcomes for a patient with anxiety?
Which intervention promotes positive outcomes for a patient with anxiety?
A patient with OCD feels driven to repeatedly wash their hands to reduce anxiety. Which aspect defines this behavior as a compulsion?
A patient with OCD feels driven to repeatedly wash their hands to reduce anxiety. Which aspect defines this behavior as a compulsion?
A patient with a history of childhood trauma exhibits symptoms of OCD and anxiety. In this scenario, what is a significant risk factor?
A patient with a history of childhood trauma exhibits symptoms of OCD and anxiety. In this scenario, what is a significant risk factor?
A patient with OCD expresses feeling shame. Which nursing intervention is most essential to establish a therapeutic relationship?
A patient with OCD expresses feeling shame. Which nursing intervention is most essential to establish a therapeutic relationship?
Which intervention is considered a basic-level nursing intervention for patients with obsessive-compulsive disorder (OCD)?
Which intervention is considered a basic-level nursing intervention for patients with obsessive-compulsive disorder (OCD)?
A patient diagnosed with body dysmorphic disorder is considering treatment options. Which classification should the nurse include when explaining the exception for biological treatments?
A patient diagnosed with body dysmorphic disorder is considering treatment options. Which classification should the nurse include when explaining the exception for biological treatments?
A patient experiencing OCD benefits from exposure and response prevention therapy. How should the nurse explain the central message of this therapy?
A patient experiencing OCD benefits from exposure and response prevention therapy. How should the nurse explain the central message of this therapy?
Which behavior is most characteristic of Reactive Attachment Disorder?
Which behavior is most characteristic of Reactive Attachment Disorder?
What is a key difference between reactive attachment disorder and disinhibited social engagement disorder?
What is a key difference between reactive attachment disorder and disinhibited social engagement disorder?
What is the most important initial intervention for children with attachment disorders?
What is the most important initial intervention for children with attachment disorders?
How might PTSD manifest differently in preschool children compared to adults?
How might PTSD manifest differently in preschool children compared to adults?
Which of the following is a common comorbidity seen in children and adolescents with PTSD?
Which of the following is a common comorbidity seen in children and adolescents with PTSD?
How does trauma impact neural pathways in individuals with PTSD?
How does trauma impact neural pathways in individuals with PTSD?
According to the polyvagal theory, what physiological state might be triggered by trauma?
According to the polyvagal theory, what physiological state might be triggered by trauma?
What role does neuroplasticity play in children's response to trauma?
What role does neuroplasticity play in children's response to trauma?
What is the primary focus of nursing interventions in the initial stage of treating children with PTSD?
What is the primary focus of nursing interventions in the initial stage of treating children with PTSD?
Why is it important to teach relaxation techniques to children with PTSD before trauma exploration?
Why is it important to teach relaxation techniques to children with PTSD before trauma exploration?
What is the rationale behind involving the child's caretakers in the treatment of PTSD?
What is the rationale behind involving the child's caretakers in the treatment of PTSD?
Which treatment outcome indicates effective treatment for a child with PTSD?
Which treatment outcome indicates effective treatment for a child with PTSD?
What is a common treatment modality for children with PTSD?
What is a common treatment modality for children with PTSD?
What percentage range of adults experiences at least one traumatic event in their lifetime?
What percentage range of adults experiences at least one traumatic event in their lifetime?
How do risk factors for PTSD in adults compare to those in children?
How do risk factors for PTSD in adults compare to those in children?
What screening tools are commonly used to assess PTSD in adults?
What screening tools are commonly used to assess PTSD in adults?
What is the primary goal of implementation in the nursing process for adults with PTSD?
What is the primary goal of implementation in the nursing process for adults with PTSD?
Which patient behavior indicates successful treatment of PTSD?
Which patient behavior indicates successful treatment of PTSD?
What class of medications is typically used in the biological treatment of PTSD?
What class of medications is typically used in the biological treatment of PTSD?
What is a defining feature of Acute Stress Disorder (ASD) that distinguishes it from PTSD?
What is a defining feature of Acute Stress Disorder (ASD) that distinguishes it from PTSD?
According to diagnostic criteria, how many symptoms must be present to diagnose Acute Stress Disorder?
According to diagnostic criteria, how many symptoms must be present to diagnose Acute Stress Disorder?
What is a common symptom of Acute Stress Disorder (ASD)?
What is a common symptom of Acute Stress Disorder (ASD)?
What is the primary focus of interventions for Acute Stress Disorder (ASD)?
What is the primary focus of interventions for Acute Stress Disorder (ASD)?
How does Adjustment Disorder generally differ from Acute Stress Disorder (ASD) and PTSD in terms of symptom severity and specificity?
How does Adjustment Disorder generally differ from Acute Stress Disorder (ASD) and PTSD in terms of symptom severity and specificity?
What forms of distress can be seen with Adjustment Disorder?
What forms of distress can be seen with Adjustment Disorder?
What is the primary characteristic of dissociative amnesia?
What is the primary characteristic of dissociative amnesia?
What differentiates dissociative fugue from typical dissociative amnesia?
What differentiates dissociative fugue from typical dissociative amnesia?
What is the focus of depersonalization?
What is the focus of depersonalization?
What is a defining characteristic of depersonalization/derealization disorder?
What is a defining characteristic of depersonalization/derealization disorder?
What is the primary characteristic of Dissociative Identity Disorder (DID)?
What is the primary characteristic of Dissociative Identity Disorder (DID)?
What is "alter" in the context of Dissociative Identity Disorder (DID)?
What is "alter" in the context of Dissociative Identity Disorder (DID)?
What broad areas should the nurse assess for a patient with Dissociative Identity Disorder?
What broad areas should the nurse assess for a patient with Dissociative Identity Disorder?
What is the primary goal of treatment planning for individuals with Dissociative Identity Disorder?
What is the primary goal of treatment planning for individuals with Dissociative Identity Disorder?
What nursing intervention is most appropriate to normalize experiences for a patient with Dissociative Identity Disorder (DID)
What nursing intervention is most appropriate to normalize experiences for a patient with Dissociative Identity Disorder (DID)
What is the ultimate goal of treatment for Dissociative Identity Disorder?
What is the ultimate goal of treatment for Dissociative Identity Disorder?
Which treatment modality is MOST associated with dissociative identity disorder?
Which treatment modality is MOST associated with dissociative identity disorder?
What is an appropriate assessment for a patient diagnosed with dissociative identity disorder?
What is an appropriate assessment for a patient diagnosed with dissociative identity disorder?
A child displays a consistent pattern of emotionally withdrawn behavior and does not seek comfort from caregivers when distressed. Which disorder is most consistent with these behaviors?
A child displays a consistent pattern of emotionally withdrawn behavior and does not seek comfort from caregivers when distressed. Which disorder is most consistent with these behaviors?
What is a key difference between Reactive Attachment Disorder and Disinhibited Social Engagement Disorder in children?
What is a key difference between Reactive Attachment Disorder and Disinhibited Social Engagement Disorder in children?
Which statement best reflects how trauma can affect a child's play in the context of PTSD?
Which statement best reflects how trauma can affect a child's play in the context of PTSD?
What neurobiological change is commonly associated with trauma, potentially leading to dissociation?
What neurobiological change is commonly associated with trauma, potentially leading to dissociation?
In the context of environmental risk factors for PTSD in children, what does the concept of 'neuroplasticity' primarily refer to?
In the context of environmental risk factors for PTSD in children, what does the concept of 'neuroplasticity' primarily refer to?
When providing initial nursing care for a 6-year-old child who has experienced trauma, which intervention is the priority?
When providing initial nursing care for a 6-year-old child who has experienced trauma, which intervention is the priority?
Why is it important for nurses to use developmentally appropriate language when interacting with children who have experienced trauma?
Why is it important for nurses to use developmentally appropriate language when interacting with children who have experienced trauma?
Which of the following best describes the role of art and play in promoting expression of feelings for a child with PTSD?
Which of the following best describes the role of art and play in promoting expression of feelings for a child with PTSD?
What is a primary indicator that treatment for a child with PTSD has been effective?
What is a primary indicator that treatment for a child with PTSD has been effective?
Which of the following is a typical biological treatment modality for children with PTSD?
Which of the following is a typical biological treatment modality for children with PTSD?
What percentage range best represents the approximate lifetime prevalence of experiencing at least one traumatic event in men?
What percentage range best represents the approximate lifetime prevalence of experiencing at least one traumatic event in men?
Why would a nurse use a screening tool like the Primary Care PTSD Screen (PC-PTSD) or the PTSD Checklist (PCL-5) when assessing a patient?
Why would a nurse use a screening tool like the Primary Care PTSD Screen (PC-PTSD) or the PTSD Checklist (PCL-5) when assessing a patient?
What is the MOST accurate explanation for why the 'stage model of treatment' is used for adults with PTSD?
What is the MOST accurate explanation for why the 'stage model of treatment' is used for adults with PTSD?
A patient is being evaluated for successful outcomes in PTSD treatment. What indicates the patient is experiencing enhanced self-esteem?
A patient is being evaluated for successful outcomes in PTSD treatment. What indicates the patient is experiencing enhanced self-esteem?
Ms. S, who was robbed at gunpoint, is diagnosed with Acute Stress Disorder. Why is it crucial to collaborate for coordination of care in this case?
Ms. S, who was robbed at gunpoint, is diagnosed with Acute Stress Disorder. Why is it crucial to collaborate for coordination of care in this case?
Which of the following differentiates Adjustment Disorder from Acute Stress Disorder and PTSD?
Which of the following differentiates Adjustment Disorder from Acute Stress Disorder and PTSD?
What is the defining characteristic of dissociative amnesia?
What is the defining characteristic of dissociative amnesia?
Which of the following accurately describes the experience of derealization?
Which of the following accurately describes the experience of derealization?
What is a key feature of Dissociative Identity Disorder (DID)?
What is a key feature of Dissociative Identity Disorder (DID)?
What is the primary goal of treatment for individuals with Dissociative Identity Disorder (DID)?
What is the primary goal of treatment for individuals with Dissociative Identity Disorder (DID)?
What unique sensory experience is often reported by individuals with anorexia nervosa that contributes to the perpetuation of restrictive eating behaviors?
What unique sensory experience is often reported by individuals with anorexia nervosa that contributes to the perpetuation of restrictive eating behaviors?
During an assessment, a client with suspected anorexia nervosa admits to using laxatives and diuretics to control their weight. How would you classify their eating disorder?
During an assessment, a client with suspected anorexia nervosa admits to using laxatives and diuretics to control their weight. How would you classify their eating disorder?
What factor complicates the determination of the true prevalence of anorexia nervosa in the general population?
What factor complicates the determination of the true prevalence of anorexia nervosa in the general population?
What co-occurring mental health condition should be prioritized for assessment in a patient newly diagnosed with anorexia nervosa?
What co-occurring mental health condition should be prioritized for assessment in a patient newly diagnosed with anorexia nervosa?
Which statement integrates the roles of genetics and neurobiology as potential risk factors for anorexia nervosa?
Which statement integrates the roles of genetics and neurobiology as potential risk factors for anorexia nervosa?
When assessing a patient admitted for anorexia nervosa, which question should the nurse ask to best understand the patient's perception of the problem?
When assessing a patient admitted for anorexia nervosa, which question should the nurse ask to best understand the patient's perception of the problem?
What physical assessment finding should prompt the nurse to further investigate for potential self-induced vomiting in a patient with suspected bulimia nervosa?
What physical assessment finding should prompt the nurse to further investigate for potential self-induced vomiting in a patient with suspected bulimia nervosa?
A 16-year-old patient with anorexia nervosa is resistant to gaining weight. What is the MOST important initial nursing intervention to help the patient progress toward weight restoration?
A 16-year-old patient with anorexia nervosa is resistant to gaining weight. What is the MOST important initial nursing intervention to help the patient progress toward weight restoration?
During the care planning phase for a patient with anorexia nervosa, what is the priority nursing diagnosis that addresses the most immediate risk to the patient's well-being?
During the care planning phase for a patient with anorexia nervosa, what is the priority nursing diagnosis that addresses the most immediate risk to the patient's well-being?
What is the primary reason for closely monitoring a patient with anorexia nervosa who is beginning a refeeding program?
What is the primary reason for closely monitoring a patient with anorexia nervosa who is beginning a refeeding program?
What specific cognitive characteristic often presents a significant challenge in the treatment of a patient with anorexia nervosa?
What specific cognitive characteristic often presents a significant challenge in the treatment of a patient with anorexia nervosa?
A nurse is educating a group of adolescent girls about the risks associated with eating disorders. Which statement regarding treatment should be included?
A nurse is educating a group of adolescent girls about the risks associated with eating disorders. Which statement regarding treatment should be included?
Which of the following is a goal of cognitive behavioral therapy (CBT) in the treatment of bulimia nervosa?
Which of the following is a goal of cognitive behavioral therapy (CBT) in the treatment of bulimia nervosa?
According to the DSM-5 criteria, what is the minimum frequency of binge eating and inappropriate compensatory behaviors required for a diagnosis of bulimia nervosa?
According to the DSM-5 criteria, what is the minimum frequency of binge eating and inappropriate compensatory behaviors required for a diagnosis of bulimia nervosa?
What statement accurately describes the genetic component of the risk factors associated with bulimia nervosa?
What statement accurately describes the genetic component of the risk factors associated with bulimia nervosa?
What element included in an assessment demonstrates that the nurse understands the needs of a patient with bulimia nervosa?
What element included in an assessment demonstrates that the nurse understands the needs of a patient with bulimia nervosa?
What nursing diagnosis is specific to patients with bulimia nervosa due to their recurrent binge-purge cycles?
What nursing diagnosis is specific to patients with bulimia nervosa due to their recurrent binge-purge cycles?
Which of the following interventions has the greatest impact if a patient is admitted to an inpatient unit for bulimia nervosa?
Which of the following interventions has the greatest impact if a patient is admitted to an inpatient unit for bulimia nervosa?
What medication is approved for bulimia nervosa?
What medication is approved for bulimia nervosa?
When providing care for a patient with bulimia nervosa, what should be the initial focus of advanced practice psychotherapy interventions?
When providing care for a patient with bulimia nervosa, what should be the initial focus of advanced practice psychotherapy interventions?
According to DSM-5, what criterion has to be met for a patient to be diagnosed with binge eating disorder?
According to DSM-5, what criterion has to be met for a patient to be diagnosed with binge eating disorder?
What behavior is associated with binge eating episodes?
What behavior is associated with binge eating episodes?
Which of the following is known to be a common comorbidity with binge eating disorder?
Which of the following is known to be a common comorbidity with binge eating disorder?
To assist a client with binge eating disorder using the nursing process, which action would be the first step in assessing the client?
To assist a client with binge eating disorder using the nursing process, which action would be the first step in assessing the client?
Which nursing diagnosis would be most applicable for someone being admitted with binge eating disorder?
Which nursing diagnosis would be most applicable for someone being admitted with binge eating disorder?
In the implementation phase of treating a client with binge eating disorder, which statement made by the nurse would be most helpful?
In the implementation phase of treating a client with binge eating disorder, which statement made by the nurse would be most helpful?
Weight tends to return after treatment in patients with binge eating disorder if what biological treatment is used?
Weight tends to return after treatment in patients with binge eating disorder if what biological treatment is used?
Which statement reflects the role of incremental goal setting in a support group for clients with binge eating disorder?
Which statement reflects the role of incremental goal setting in a support group for clients with binge eating disorder?
Which statement best describes the eating behaviors associated with Pica?
Which statement best describes the eating behaviors associated with Pica?
What key feature differentiates rumination disorder from other eating disorders in adults?
What key feature differentiates rumination disorder from other eating disorders in adults?
What characteristic describes avoidant/restrictive food intake disorder (ARFID)?
What characteristic describes avoidant/restrictive food intake disorder (ARFID)?
A client presents with significant weight loss, dry skin, and lanugo. What laboratory finding would correlate with these physical signs?
A client presents with significant weight loss, dry skin, and lanugo. What laboratory finding would correlate with these physical signs?
A patient with dementia living at home is repeatedly found consuming powdered laundry detergent. What condition is MOST likely suggested by this behavior?
A patient with dementia living at home is repeatedly found consuming powdered laundry detergent. What condition is MOST likely suggested by this behavior?
A nurse is caring for a client with anorexia nervosa who is struggling with distorted body image. Which of the following interventions would be MOST appropriate?
A nurse is caring for a client with anorexia nervosa who is struggling with distorted body image. Which of the following interventions would be MOST appropriate?
A patient with bulimia nervosa has been engaging in self-induced vomiting after meals for the past two years. What is the MOST likely electrolyte imbalance that the nurse would expect to see in this patient?
A patient with bulimia nervosa has been engaging in self-induced vomiting after meals for the past two years. What is the MOST likely electrolyte imbalance that the nurse would expect to see in this patient?
A nurse suspects a patient has anorexia nervosa. Which assessment question is MOST appropriate to initiate the conversation and gather information?
A nurse suspects a patient has anorexia nervosa. Which assessment question is MOST appropriate to initiate the conversation and gather information?
A teenage girl is admitted to the hospital for anorexia nervosa. What is the MOST appropriate and realistic initial goal?
A teenage girl is admitted to the hospital for anorexia nervosa. What is the MOST appropriate and realistic initial goal?
Which of the following BEST describes the primary goal of nutritional interventions in a patient with anorexia nervosa?
Which of the following BEST describes the primary goal of nutritional interventions in a patient with anorexia nervosa?
What is the first nursing intervention when a patient begins to express that they are suicidal?
What is the first nursing intervention when a patient begins to express that they are suicidal?
A client with bulimia nervosa has been prescribed fluoxetine (Prozac). What statement explains to the patient the purpose of this medication?
A client with bulimia nervosa has been prescribed fluoxetine (Prozac). What statement explains to the patient the purpose of this medication?
The parent of a young patient with avoidant/restrictive food intake disorder (ARFID) expresses frustration. Which of the following should the nurse explain to the parent?
The parent of a young patient with avoidant/restrictive food intake disorder (ARFID) expresses frustration. Which of the following should the nurse explain to the parent?
A nurse finds a 22-year-old client with bulimia nervosa in the bathroom vomiting. Which nursing intervention is MOST appropriate?
A nurse finds a 22-year-old client with bulimia nervosa in the bathroom vomiting. Which nursing intervention is MOST appropriate?
A 17-year-old female with anorexia nervosa is admitted to the hospital for severe malnutrition. During the initial assessment, which finding would be of greatest concern?
A 17-year-old female with anorexia nervosa is admitted to the hospital for severe malnutrition. During the initial assessment, which finding would be of greatest concern?
A nurse is caring for a patient with bulimia nervosa who is engaging in self-induced vomiting after meals. Which electrolyte imbalance is the patient MOST at risk for developing?
A nurse is caring for a patient with bulimia nervosa who is engaging in self-induced vomiting after meals. Which electrolyte imbalance is the patient MOST at risk for developing?
A 22-year-old patient with binge eating disorder is starting cognitive behavioral therapy (CBT). What initial intervention is MOST important for the therapist to implement?
A 22-year-old patient with binge eating disorder is starting cognitive behavioral therapy (CBT). What initial intervention is MOST important for the therapist to implement?
Which statement BEST describes the role of genetics in the development of anorexia nervosa?
Which statement BEST describes the role of genetics in the development of anorexia nervosa?
A patient with bulimia nervosa is prescribed fluoxetine (Prozac). What is the primary rationale for using this medication in the treatment of bulimia?
A patient with bulimia nervosa is prescribed fluoxetine (Prozac). What is the primary rationale for using this medication in the treatment of bulimia?
A patient with binge eating disorder is concerned about weight gain. Which intervention is the MOST appropriate to address this concern?
A patient with binge eating disorder is concerned about weight gain. Which intervention is the MOST appropriate to address this concern?
A nurse is assessing a patient with suspected anorexia nervosa. Which physical sign is MOST indicative of the body's attempt to conserve energy in response to starvation?
A nurse is assessing a patient with suspected anorexia nervosa. Which physical sign is MOST indicative of the body's attempt to conserve energy in response to starvation?
Which statement accurately differentiates between anorexia nervosa, restricting type, and bulimia nervosa?
Which statement accurately differentiates between anorexia nervosa, restricting type, and bulimia nervosa?
A nurse providing education to a community group about eating disorders should include which statement about avoidant/restrictive food intake disorder (ARFID)?
A nurse providing education to a community group about eating disorders should include which statement about avoidant/restrictive food intake disorder (ARFID)?
A patient recently diagnosed with Pica is noted to compulsively eating paper. What is the MOST appropriate initial nursing intervention?
A patient recently diagnosed with Pica is noted to compulsively eating paper. What is the MOST appropriate initial nursing intervention?
How do nonsteroidal anti-inflammatory drugs (NSAIDs) exert their anti-inflammatory effects?
How do nonsteroidal anti-inflammatory drugs (NSAIDs) exert their anti-inflammatory effects?
Which property is unique to aspirin among all the NSAIDs?
Which property is unique to aspirin among all the NSAIDs?
Why should aspirin be administered at the first sign of a myocardial infarction?
Why should aspirin be administered at the first sign of a myocardial infarction?
A patient with a known aspirin allergy is prescribed an NSAID for arthritis pain. What is the most appropriate nursing action?
A patient with a known aspirin allergy is prescribed an NSAID for arthritis pain. What is the most appropriate nursing action?
Which condition presents a contraindication for NSAID use due to increased bleeding risk?
Which condition presents a contraindication for NSAID use due to increased bleeding risk?
What is the mechanism by which NSAIDs can lead to acute renal failure?
What is the mechanism by which NSAIDs can lead to acute renal failure?
Why does the FDA require a Black Box Warning for all NSAIDs (except aspirin)?
Why does the FDA require a Black Box Warning for all NSAIDs (except aspirin)?
What is the purpose of misoprostol when prescribed with NSAIDs?
What is the purpose of misoprostol when prescribed with NSAIDs?
Which drug interaction is most important to monitor when a patient is prescribed NSAIDs?
Which drug interaction is most important to monitor when a patient is prescribed NSAIDs?
In addition to standard assessment parameters, what specific assessment should be included for a patient taking high-dose ibuprofen?
In addition to standard assessment parameters, what specific assessment should be included for a patient taking high-dose ibuprofen?
Which condition is indomethacin (Indocin) used to treat?
Which condition is indomethacin (Indocin) used to treat?
What is the maximum recommended duration for ketorolac (Toradol) administration to prevent severe adverse effects?
What is the maximum recommended duration for ketorolac (Toradol) administration to prevent severe adverse effects?
What is a rare but potentially severe adverse effect associated with salicylate use in children and teenagers?
What is a rare but potentially severe adverse effect associated with salicylate use in children and teenagers?
What is the primary mechanism of action of colchicine in treating gout?
What is the primary mechanism of action of colchicine in treating gout?
What is the main mechanism of action of probenecid in the treatment of gout?
What is the main mechanism of action of probenecid in the treatment of gout?
Allopurinol is prescribed to prevent which condition?
Allopurinol is prescribed to prevent which condition?
What information should the nurse reinforce to a patient taking lesinurad for hyperuricemia?
What information should the nurse reinforce to a patient taking lesinurad for hyperuricemia?
What is a key difference between febuxostat and allopurinol?
What is a key difference between febuxostat and allopurinol?
Which laboratory finding is MOST important for the nurse to monitor in a patient taking allopurinol to prevent tumor lysis syndrome?
Which laboratory finding is MOST important for the nurse to monitor in a patient taking allopurinol to prevent tumor lysis syndrome?
A patient taking ibuprofen reports taking glucosamine and chondroitin for osteoarthritis. What instruction is MOST important for the nurse to provide?
A patient taking ibuprofen reports taking glucosamine and chondroitin for osteoarthritis. What instruction is MOST important for the nurse to provide?
A patient with osteoarthritis is prescribed a topical cream for pain relief. What information should the nurse include when educating the patient??
A patient with osteoarthritis is prescribed a topical cream for pain relief. What information should the nurse include when educating the patient??
A patient taking colchicine has an indication of short-term leukopenia. Which precaution should the nurse take?
A patient taking colchicine has an indication of short-term leukopenia. Which precaution should the nurse take?
What should the nurse complete prior to beginning antiinflammatory drug therapy?
What should the nurse complete prior to beginning antiinflammatory drug therapy?
What should the nurse do to assure the therapeutic effects of antiinflammatory therapy?
What should the nurse do to assure the therapeutic effects of antiinflammatory therapy?
What intervention should be included in regards to drug administration of antiinflammatory therapy?
What intervention should be included in regards to drug administration of antiinflammatory therapy?
A patient is being educated on the usage of antiinflammatory drugs, what information includes appropriate education?
A patient is being educated on the usage of antiinflammatory drugs, what information includes appropriate education?
What should you as the nurse educate the patient on regarding enteric-coated tablets?
What should you as the nurse educate the patient on regarding enteric-coated tablets?
Which manifestation indicates salicylate toxicity?
Which manifestation indicates salicylate toxicity?
What is the classification of nabumetone?
What is the classification of nabumetone?
How long may therapeutic effects be seen with NSAIDs?
How long may therapeutic effects be seen with NSAIDs?
A patient reports taking acetaminophen with a dose of ASA, caffeine, and acetaminophen. What interaction should the nurse be most concerned about?
A patient reports taking acetaminophen with a dose of ASA, caffeine, and acetaminophen. What interaction should the nurse be most concerned about?
What is the most commonly used NSAID?
What is the most commonly used NSAID?
What classification of NSAID is naproxen?
What classification of NSAID is naproxen?
When planning care for a patient receiving NSAIDs, which intervention is most appropriate to minimize the risk of gastrointestinal (GI) distress?
When planning care for a patient receiving NSAIDs, which intervention is most appropriate to minimize the risk of gastrointestinal (GI) distress?
Which therapeutic action are NSAIDs known for?
Which therapeutic action are NSAIDs known for?
A person is being given a low dose of Aspirin as prophylatic therapy due to risk factors, what dosage will they be given?
A person is being given a low dose of Aspirin as prophylatic therapy due to risk factors, what dosage will they be given?
What are 3 common properties that all NSAIDs share?
What are 3 common properties that all NSAIDs share?
A patient has mild to moderate osteoarthritis, R.A., and gouty arthritis. What medication is common for this indication?
A patient has mild to moderate osteoarthritis, R.A., and gouty arthritis. What medication is common for this indication?
When the body has what adverse affect does it need to be given NSAIDs?
When the body has what adverse affect does it need to be given NSAIDs?
What are some adverse effects of COX-2 inhibitors?
What are some adverse effects of COX-2 inhibitors?
A patient is on long-term medication needs, what is true regarding Total Joint Arthroplasty(TJA)?
A patient is on long-term medication needs, what is true regarding Total Joint Arthroplasty(TJA)?
A patient with a known history of peptic ulcer disease is prescribed an NSAID for chronic pain. Which intervention is most important to include in the patient's plan of care?
A patient with a known history of peptic ulcer disease is prescribed an NSAID for chronic pain. Which intervention is most important to include in the patient's plan of care?
A nurse is educating a patient about the long-term use of NSAIDs. Which potential adverse effect is most critical to emphasize?
A nurse is educating a patient about the long-term use of NSAIDs. Which potential adverse effect is most critical to emphasize?
A patient is prescribed low-dose aspirin (81 mg daily). Which statement indicates an understanding of the purpose for this treatment?
A patient is prescribed low-dose aspirin (81 mg daily). Which statement indicates an understanding of the purpose for this treatment?
A child has a fever and is experiencing mild pain. Which medication is the safest to administer for fever and pain relief?
A child has a fever and is experiencing mild pain. Which medication is the safest to administer for fever and pain relief?
A patient taking aspirin reports tinnitus. What is the nurse's most appropriate action?
A patient taking aspirin reports tinnitus. What is the nurse's most appropriate action?
A 60-year-old patient is prescribed indomethacin for acute gouty arthritis. Considering the patient's age, what potential adverse effect requires careful monitoring?
A 60-year-old patient is prescribed indomethacin for acute gouty arthritis. Considering the patient's age, what potential adverse effect requires careful monitoring?
A patient is prescribed ketorolac after surgery. What teaching should the nurse emphasize related to this medication?
A patient is prescribed ketorolac after surgery. What teaching should the nurse emphasize related to this medication?
A patient with osteoarthritis is prescribed ibuprofen. What indicates to the nurse that the medication is effective?
A patient with osteoarthritis is prescribed ibuprofen. What indicates to the nurse that the medication is effective?
A patient with a known sulfa allergy is prescribed an anti-inflammatory medication. Which medication should the nurse question?
A patient with a known sulfa allergy is prescribed an anti-inflammatory medication. Which medication should the nurse question?
A patient is prescribed allopurinol for chronic gout. The nurse recognizes that allopurinol:
A patient is prescribed allopurinol for chronic gout. The nurse recognizes that allopurinol:
A patient taking probenecid for gout should be taught:
A patient taking probenecid for gout should be taught:
A patient is prescribed colchicine. What adverse effect is most important for the nurse to teach the patient?
A patient is prescribed colchicine. What adverse effect is most important for the nurse to teach the patient?
A patient report taking glucosamine and warfarin (Coumadin). Why should this patient by monitored?
A patient report taking glucosamine and warfarin (Coumadin). Why should this patient by monitored?
Which laboratory values are most important for the nurse to monitor when a patient is on a high dose of ibuprofen?
Which laboratory values are most important for the nurse to monitor when a patient is on a high dose of ibuprofen?
The nurse is caring for a patient taking an NSAID. Which statement should the nurse include in the patient's education plan?
The nurse is caring for a patient taking an NSAID. Which statement should the nurse include in the patient's education plan?
What is the primary mechanism by which antacids provide relief from acid-related disorders?
What is the primary mechanism by which antacids provide relief from acid-related disorders?
Why are magnesium salts often combined with aluminum or calcium salts in antacid preparations?
Why are magnesium salts often combined with aluminum or calcium salts in antacid preparations?
What is the MOST concerning risk associated with the use of magnesium-containing antacids in patients with renal failure?
What is the MOST concerning risk associated with the use of magnesium-containing antacids in patients with renal failure?
Why should long-term self-medication with antacids be discouraged without medical evaluation?
Why should long-term self-medication with antacids be discouraged without medical evaluation?
How do H₂ antagonists relieve symptoms of hyperacidity-related conditions?
How do H₂ antagonists relieve symptoms of hyperacidity-related conditions?
What is a crucial step to consider when administering H₂ receptor antagonists to a patient who is also taking antacids?
What is a crucial step to consider when administering H₂ receptor antagonists to a patient who is also taking antacids?
How do proton pump inhibitors (PPIs) achieve gastric acid suppression?
How do proton pump inhibitors (PPIs) achieve gastric acid suppression?
What is the rationale behind administering proton pump inhibitors (PPIs) in conjunction with antibiotics for treating H. pylori-induced ulcers?
What is the rationale behind administering proton pump inhibitors (PPIs) in conjunction with antibiotics for treating H. pylori-induced ulcers?
How does sucralfate (Carafate) protect gastrointestinal ulcers?
How does sucralfate (Carafate) protect gastrointestinal ulcers?
Misoprostol (Cytotec) is prescribed to patients taking NSAIDs to prevent gastric ulcers. What is the mechanism of action of misoprostol?
Misoprostol (Cytotec) is prescribed to patients taking NSAIDs to prevent gastric ulcers. What is the mechanism of action of misoprostol?
What is the primary mechanism of action for adsorbent antidiarrheals?
What is the primary mechanism of action for adsorbent antidiarrheals?
Which of the following is the mechanism of action of antimotility drugs?
Which of the following is the mechanism of action of antimotility drugs?
What is the main action of probiotics in treating diarrhea?
What is the main action of probiotics in treating diarrhea?
Which of the following is an example of an antidiarrheal medication that works by coating the walls of the gastrointestinal tract and binding to the causative bacteria?
Which of the following is an example of an antidiarrheal medication that works by coating the walls of the gastrointestinal tract and binding to the causative bacteria?
The antidiarrheal class of medications that is commonly used with adsorbents and opiates are:
The antidiarrheal class of medications that is commonly used with adsorbents and opiates are:
A patient with diarrhea is prescribed Lactobacillus acidophilus. The nurse understands that this medication will impact the diarrhea how?
A patient with diarrhea is prescribed Lactobacillus acidophilus. The nurse understands that this medication will impact the diarrhea how?
A patient is prescribed bismuth subsalicylate for diarrhea. Which potential adverse effect should the nurse educate the patient about?
A patient is prescribed bismuth subsalicylate for diarrhea. Which potential adverse effect should the nurse educate the patient about?
A patient is taking an anticholinergic medication for diarrhea. Which adverse effect is most important for the nurse to teach the patient about?
A patient is taking an anticholinergic medication for diarrhea. Which adverse effect is most important for the nurse to teach the patient about?
When providing care for a patient who is prescribed an opiate antidiarrheal, the nurse should monitor for which adverse effect?
When providing care for a patient who is prescribed an opiate antidiarrheal, the nurse should monitor for which adverse effect?
A patient taking digoxin is prescribed an adsorbent antidiarrheal. What is a potential concern with this combination?
A patient taking digoxin is prescribed an adsorbent antidiarrheal. What is a potential concern with this combination?
What is the primary reason bismuth subsalicylate should not be given to children or teenagers with chickenpox or influenza?
What is the primary reason bismuth subsalicylate should not be given to children or teenagers with chickenpox or influenza?
A patient with a history of narrow-angle glaucoma is experiencing diarrhea. Which type of antidiarrheal agent should the nurse avoid administering?
A patient with a history of narrow-angle glaucoma is experiencing diarrhea. Which type of antidiarrheal agent should the nurse avoid administering?
What assessment is most important for the nurse to complete for a patient being treated for diarrhea with antidiarrheals?
What assessment is most important for the nurse to complete for a patient being treated for diarrhea with antidiarrheals?
A patient is prescribed an antidiarrheal medication. When should the patient be instructed to notify their healthcare provider?
A patient is prescribed an antidiarrheal medication. When should the patient be instructed to notify their healthcare provider?
Which of these is considered a non-surgical treatment for constipation?
Which of these is considered a non-surgical treatment for constipation?
Which of the following describes the action of bulk-forming laxatives?
Which of the following describes the action of bulk-forming laxatives?
A patient is prescribed an emollient laxative for constipation. What is the mechanism of action of this type of laxative?
A patient is prescribed an emollient laxative for constipation. What is the mechanism of action of this type of laxative?
A patient is receiving polyethylene glycol (PEG) for constipation. What type of laxative is PEG?
A patient is receiving polyethylene glycol (PEG) for constipation. What type of laxative is PEG?
Which of the following explains how saline laxatives work?
Which of the following explains how saline laxatives work?
Which type of laxative works by increasing peristalsis via intestinal nerve stimulation?
Which type of laxative works by increasing peristalsis via intestinal nerve stimulation?
What is the primary action of peripherally acting opioid antagonists in treating constipation?
What is the primary action of peripherally acting opioid antagonists in treating constipation?
A patient is prescribed methylcellulose for constipation. What patient teaching is important?
A patient is prescribed methylcellulose for constipation. What patient teaching is important?
A patient with a history of renal insufficiency is prescribed a laxative for occasional constipation. Which type of laxative should be avoided?
A patient with a history of renal insufficiency is prescribed a laxative for occasional constipation. Which type of laxative should be avoided?
What is the primary reason patients should be taught to avoid long-term use of laxatives?
What is the primary reason patients should be taught to avoid long-term use of laxatives?
A patient reports chronic constipation. What is an important nonpharmacological recommendation?
A patient reports chronic constipation. What is an important nonpharmacological recommendation?
A patient is prescribed bisacodyl. What should the nurse include in patient's education?
A patient is prescribed bisacodyl. What should the nurse include in patient's education?
A patient reports severe abdominal pain, muscle weakness, cramps, and dizziness after using a laxative. What is the most important action?
A patient reports severe abdominal pain, muscle weakness, cramps, and dizziness after using a laxative. What is the most important action?
Irritable Bowel Syndrome is a digestive disorder characterized by:
Irritable Bowel Syndrome is a digestive disorder characterized by:
What is a typical approach that patients with Irritable Bowel Syndrome utilize to cope with symptoms?
What is a typical approach that patients with Irritable Bowel Syndrome utilize to cope with symptoms?
A patient takes Alosetron to treat Irritable Bowel Syndrome, what symptom are they most likely suffering from?
A patient takes Alosetron to treat Irritable Bowel Syndrome, what symptom are they most likely suffering from?
What assessment should the nurse prioritize for a patient who is starting Alosetron(Lotronex) for IBS?
What assessment should the nurse prioritize for a patient who is starting Alosetron(Lotronex) for IBS?
Why would a patient be prescribed Lubiprostone for Irritable Bowel Syndrome?
Why would a patient be prescribed Lubiprostone for Irritable Bowel Syndrome?
A patient presents to the clinic with Irritable Bowel Syndrome. What medication should the nurse expect to administer to this patient?
A patient presents to the clinic with Irritable Bowel Syndrome. What medication should the nurse expect to administer to this patient?
During what type of diarrhea duration would you not recommend over the counter treatments, and would refer the client to their primary care provider?
During what type of diarrhea duration would you not recommend over the counter treatments, and would refer the client to their primary care provider?
What is the primary goal of diarrhea treatment?
What is the primary goal of diarrhea treatment?
Which of the following is a common cause of acute diarrhea?
Which of the following is a common cause of acute diarrhea?
Which of the following is a common cause of chronic diarrhea?
Which of the following is a common cause of chronic diarrhea?
A patient with a history of diabetes mellitus is experiencing chronic diarrhea. What is the primary underlying mechanism by which diabetes contributes to chronic diarrhea?
A patient with a history of diabetes mellitus is experiencing chronic diarrhea. What is the primary underlying mechanism by which diabetes contributes to chronic diarrhea?
When a patient is prescribed bismuth subsalicylate (Pepto-Bismol) for mild diarrhea, what education should the nurse provide regarding side effects?
When a patient is prescribed bismuth subsalicylate (Pepto-Bismol) for mild diarrhea, what education should the nurse provide regarding side effects?
A patient is prescribed loperamide for diarrhea. What mechanism of action should the nurse use to explain how this medication works?
A patient is prescribed loperamide for diarrhea. What mechanism of action should the nurse use to explain how this medication works?
A patient who has been taking an antidiarrheal medication complains of blurred vision and difficulty urinating. Which class of antidiarrheal medications is most likely responsible for these side effects?
A patient who has been taking an antidiarrheal medication complains of blurred vision and difficulty urinating. Which class of antidiarrheal medications is most likely responsible for these side effects?
A patient is prescribed lactobacillus acidophilus (Bacid) to manage antibiotic-induced diarrhea. How does this medication work to achieve a therapeutic effect?
A patient is prescribed lactobacillus acidophilus (Bacid) to manage antibiotic-induced diarrhea. How does this medication work to achieve a therapeutic effect?
A patient is prescribed an adsorbent antidiarrheal medication. What instructions should the nurse include when administering this medication?
A patient is prescribed an adsorbent antidiarrheal medication. What instructions should the nurse include when administering this medication?
A patient with chronic diarrhea also has a history of glaucoma. Which type of antidiarrheal medication should be avoided for this patient?
A patient with chronic diarrhea also has a history of glaucoma. Which type of antidiarrheal medication should be avoided for this patient?
When providing dietary guidance for a patient experiencing constipation, which of the following strategies should the nurse recommend as the initial approach?
When providing dietary guidance for a patient experiencing constipation, which of the following strategies should the nurse recommend as the initial approach?
Lactulose helps to reduce blood ammonia levels by converting ammonia to ammonium, which can then be eliminated by the body. For a patient taking lactulose four times a day, what therapeutic effect would indicate that the medication is working effectively?
Lactulose helps to reduce blood ammonia levels by converting ammonia to ammonium, which can then be eliminated by the body. For a patient taking lactulose four times a day, what therapeutic effect would indicate that the medication is working effectively?
A patient experiencing constipation is prescribed docusate sodium. What should the nurse explain is the primary mechanism of action for this medication?
A patient experiencing constipation is prescribed docusate sodium. What should the nurse explain is the primary mechanism of action for this medication?
An elderly patient with a history of renal insufficiency is prescribed Milk of Magnesia for occasional constipation. What potential adverse effect should the nurse monitor for?
An elderly patient with a history of renal insufficiency is prescribed Milk of Magnesia for occasional constipation. What potential adverse effect should the nurse monitor for?
A patient is prescribed bisacodyl for constipation. What instructions should the nurse provide regarding its administration?
A patient is prescribed bisacodyl for constipation. What instructions should the nurse provide regarding its administration?
A patient who has been using stimulant laxatives long-term is now experiencing decreased bowel tone and dependency. What is the best approach to address this issue?
A patient who has been using stimulant laxatives long-term is now experiencing decreased bowel tone and dependency. What is the best approach to address this issue?
Before administering belladonna alkaloids, it is most important for the nurse to assess the patient for a history of which condition?
Before administering belladonna alkaloids, it is most important for the nurse to assess the patient for a history of which condition?
A patient is newly diagnosed with Irritable Bowel Syndrome. What OTC treatments should the nurse inform them of that will minimize symptoms?
A patient is newly diagnosed with Irritable Bowel Syndrome. What OTC treatments should the nurse inform them of that will minimize symptoms?
A patient taking warfarin has been prescribed an adsorbent for diarrhea. It is important for the nurse to assess the patient for bruising because use of warfarin with adsorbents interferes with the absorption of which vitamin?
A patient taking warfarin has been prescribed an adsorbent for diarrhea. It is important for the nurse to assess the patient for bruising because use of warfarin with adsorbents interferes with the absorption of which vitamin?
A hospitalized patient is experiencing diarrhea. Which of the following does the nurse identify as adverse side effects of diphenoxylate with atropine therapy? (Select all that apply.)
A hospitalized patient is experiencing diarrhea. Which of the following does the nurse identify as adverse side effects of diphenoxylate with atropine therapy? (Select all that apply.)
A 48-year-old patient has been admitted with abdominal pain and states that she has not had a bowel movement for 4 days. Her abdomen is distended and slightly tender. Which laxative would be appropriate for this patient?
A 48-year-old patient has been admitted with abdominal pain and states that she has not had a bowel movement for 4 days. Her abdomen is distended and slightly tender. Which laxative would be appropriate for this patient?
Which antidiarrheal does the nurse associate with the development of adverse effects of urinary retention, headache, confusion, dry skin, rash, and blurred vision?
Which antidiarrheal does the nurse associate with the development of adverse effects of urinary retention, headache, confusion, dry skin, rash, and blurred vision?
What is the primary mechanism by which anticholinergic drugs like scopolamine reduce nausea?
What is the primary mechanism by which anticholinergic drugs like scopolamine reduce nausea?
A patient is prescribed promethazine (Phenergan) for nausea. What is an important consideration regarding the administration route?
A patient is prescribed promethazine (Phenergan) for nausea. What is an important consideration regarding the administration route?
A patient is prescribed metoclopramide (Reglan). What should the nurse monitor for, especially with long-term use?
A patient is prescribed metoclopramide (Reglan). What should the nurse monitor for, especially with long-term use?
A patient with chemotherapy-induced nausea and vomiting is prescribed aprepitant. What is the mechanism of action of this medication?
A patient with chemotherapy-induced nausea and vomiting is prescribed aprepitant. What is the mechanism of action of this medication?
What is the primary use of tetrahydrocannabinoids like dronabinol?
What is the primary use of tetrahydrocannabinoids like dronabinol?
Which antiemetic class is most associated with the potential to cause a prolonged QTc interval?
Which antiemetic class is most associated with the potential to cause a prolonged QTc interval?
A patient taking warfarin reports using ginger for nausea. What is the primary concern with this combination?
A patient taking warfarin reports using ginger for nausea. What is the primary concern with this combination?
A 2-month pregnant patient is experiencing severe morning sickness. After nonpharmacologic measures have failed, which antiemetic drug is MOST appropriate?
A 2-month pregnant patient is experiencing severe morning sickness. After nonpharmacologic measures have failed, which antiemetic drug is MOST appropriate?
What nursing intervention is most important when administering antiemetics, considering their common adverse effects?
What nursing intervention is most important when administering antiemetics, considering their common adverse effects?
When should antiemetics be administered to a patient receiving chemotherapy to prevent nausea and vomiting?
When should antiemetics be administered to a patient receiving chemotherapy to prevent nausea and vomiting?
A patient is prescribed scopolamine for motion sickness. What condition in the patient's history would be a contraindication for this drug?
A patient is prescribed scopolamine for motion sickness. What condition in the patient's history would be a contraindication for this drug?
Prokinetic drugs are prescribed for a patient receiving continuous tube feedings. What is the primary benefit of using these drugs in this situation?
Prokinetic drugs are prescribed for a patient receiving continuous tube feedings. What is the primary benefit of using these drugs in this situation?
What is the primary mechanism of action of antihistamine drugs in preventing nausea and vomiting?
What is the primary mechanism of action of antihistamine drugs in preventing nausea and vomiting?
For a patient taking antiemetics, what is a critical instruction regarding alcohol consumption?
For a patient taking antiemetics, what is a critical instruction regarding alcohol consumption?
What is a general use for antiemetic drugs?
What is a general use for antiemetic drugs?
What is the main ingredient of Tetrahydrocannabinoids?
What is the main ingredient of Tetrahydrocannabinoids?
Which of the following is an example of Antidopaminergic drugs?
Which of the following is an example of Antidopaminergic drugs?
In what ways does tetrahydrocannabinoids affect nausea and vomiting?
In what ways does tetrahydrocannabinoids affect nausea and vomiting?
Why aren't Hydroxyzine injections done via IV?
Why aren't Hydroxyzine injections done via IV?
A patient who has been using Metoclopramide long term is now exhibiting involuntary movements of their tongue. What is the correct course of action?
A patient who has been using Metoclopramide long term is now exhibiting involuntary movements of their tongue. What is the correct course of action?
What adverse effects does ginger have?
What adverse effects does ginger have?
Which condition is amisulpride used for?
Which condition is amisulpride used for?
What is the main action of prokinetic drugs?
What is the main action of prokinetic drugs?
Which of the following drugs is of most concern due to associated cardiac dysrhythmias?
Which of the following drugs is of most concern due to associated cardiac dysrhythmias?
What should nurses assess for when pertaining to antiemetic drugs?
What should nurses assess for when pertaining to antiemetic drugs?
A 54 year old male patient starts to feel discomfort in his chest. What type of drug would MOST cause concern?
A 54 year old male patient starts to feel discomfort in his chest. What type of drug would MOST cause concern?
Which statement best describes emesis?
Which statement best describes emesis?
Which of the following best describes nausea?
Which of the following best describes nausea?
Which of the following is NOT one of the three types of nausea and vomiting?
Which of the following is NOT one of the three types of nausea and vomiting?
What do Vomiting Center (VC) and Chemoreceptor trigger zone (CTZ) share in common?
What do Vomiting Center (VC) and Chemoreceptor trigger zone (CTZ) share in common?
How do most antiemetics function?
How do most antiemetics function?
A patient gets a prescription of a transdermal patch for nausea, how long will it release for.
A patient gets a prescription of a transdermal patch for nausea, how long will it release for.
Which of the following is true regarding adverse effects and antiemetic drugs?
Which of the following is true regarding adverse effects and antiemetic drugs?
A patient is diagnosed with terminal cancer and has no chemotherapy induced nausea or vomiting. Which of the following should the nurse expect from dronabinol twice a day?
A patient is diagnosed with terminal cancer and has no chemotherapy induced nausea or vomiting. Which of the following should the nurse expect from dronabinol twice a day?
Flashcards
Osteoarthritis (OA)
Osteoarthritis (OA)
Progressive, degenerative joint disease characterized by the breakdown of cartilage.
Rheumatoid Arthritis (RA)
Rheumatoid Arthritis (RA)
Chronic, systemic autoimmune disease causing inflammation of the joints.
Musculoskeletal Assessment
Musculoskeletal Assessment
Techniques to assess musculoskeletal health, including expected and unexpected findings, electrolyte levels, and post-arthrocentesis care.
Musculoskeletal Disorders
Musculoskeletal Disorders
Signup and view all the flashcards
GI Assessment
GI Assessment
Signup and view all the flashcards
GI Disorders
GI Disorders
Signup and view all the flashcards
GI Medications
GI Medications
Signup and view all the flashcards
Mental Health Care
Mental Health Care
Signup and view all the flashcards
Bariatric surgery
Bariatric surgery
Signup and view all the flashcards
Review materials
Review materials
Signup and view all the flashcards
Clinical Math
Clinical Math
Signup and view all the flashcards
Strains
Strains
Signup and view all the flashcards
Sprains
Sprains
Signup and view all the flashcards
Severe strains
Severe strains
Signup and view all the flashcards
Severe sprains
Severe sprains
Signup and view all the flashcards
RICE
RICE
Signup and view all the flashcards
Fracture
Fracture
Signup and view all the flashcards
Complete fracture
Complete fracture
Signup and view all the flashcards
Incomplete fracture
Incomplete fracture
Signup and view all the flashcards
Closed fracture
Closed fracture
Signup and view all the flashcards
Open fracture
Open fracture
Signup and view all the flashcards
Avulsion fracture
Avulsion fracture
Signup and view all the flashcards
Comminuted fracture
Comminuted fracture
Signup and view all the flashcards
Compression fracture
Compression fracture
Signup and view all the flashcards
Depressed fracture
Depressed fracture
Signup and view all the flashcards
Displaced fracture
Displaced fracture
Signup and view all the flashcards
Greenstick fracture
Greenstick fracture
Signup and view all the flashcards
Oblique fracture
Oblique fracture
Signup and view all the flashcards
Spiral fracture
Spiral fracture
Signup and view all the flashcards
Impacted fracture
Impacted fracture
Signup and view all the flashcards
Closed reduction
Closed reduction
Signup and view all the flashcards
Open reduction
Open reduction
Signup and view all the flashcards
Internal fixation
Internal fixation
Signup and view all the flashcards
External fixation
External fixation
Signup and view all the flashcards
Traction
Traction
Signup and view all the flashcards
Skeletal traction
Skeletal traction
Signup and view all the flashcards
Skin traction
Skin traction
Signup and view all the flashcards
Neurovascular compromise
Neurovascular compromise
Signup and view all the flashcards
Compartment syndrome
Compartment syndrome
Signup and view all the flashcards
Fasciotomy
Fasciotomy
Signup and view all the flashcards
Venous Thromboemboli (VTE)
Venous Thromboemboli (VTE)
Signup and view all the flashcards
Musculoskeletal Injuries Impact
Musculoskeletal Injuries Impact
Signup and view all the flashcards
Tendons
Tendons
Signup and view all the flashcards
Ligaments
Ligaments
Signup and view all the flashcards
First-degree strain
First-degree strain
Signup and view all the flashcards
Second-degree strain
Second-degree strain
Signup and view all the flashcards
Third-degree strain
Third-degree strain
Signup and view all the flashcards
First-degree sprain
First-degree sprain
Signup and view all the flashcards
Second-degree sprain
Second-degree sprain
Signup and view all the flashcards
Third-degree sprain
Third-degree sprain
Signup and view all the flashcards
Sprain/Strain Diagnosis
Sprain/Strain Diagnosis
Signup and view all the flashcards
RICE acronym
RICE acronym
Signup and view all the flashcards
Sprain/Strain Complications
Sprain/Strain Complications
Signup and view all the flashcards
Disorders increasing fracture risk
Disorders increasing fracture risk
Signup and view all the flashcards
High-energy fracture causes
High-energy fracture causes
Signup and view all the flashcards
Low-energy fracture causes
Low-energy fracture causes
Signup and view all the flashcards
Bone
Bone
Signup and view all the flashcards
Fracture Antibiotics
Fracture Antibiotics
Signup and view all the flashcards
Fracture Movement/Function
Fracture Movement/Function
Signup and view all the flashcards
Hemorrhagic Shock Symptoms
Hemorrhagic Shock Symptoms
Signup and view all the flashcards
Neurovascular Compromise Signs
Neurovascular Compromise Signs
Signup and view all the flashcards
Rhabdomyolysis Indication
Rhabdomyolysis Indication
Signup and view all the flashcards
Rhabdomyolysis Urine
Rhabdomyolysis Urine
Signup and view all the flashcards
Malunion
Malunion
Signup and view all the flashcards
Nonunion
Nonunion
Signup and view all the flashcards
Fracture edema management
Fracture edema management
Signup and view all the flashcards
Compartment syndrome causes
Compartment syndrome causes
Signup and view all the flashcards
What is a strain?
What is a strain?
Signup and view all the flashcards
What is a sprain?
What is a sprain?
Signup and view all the flashcards
What are first-degree strains?
What are first-degree strains?
Signup and view all the flashcards
What are second-degree strains?
What are second-degree strains?
Signup and view all the flashcards
What are first-degree sprains?
What are first-degree sprains?
Signup and view all the flashcards
What are second-degree sprains?
What are second-degree sprains?
Signup and view all the flashcards
How are sprains and strains diagnosed?
How are sprains and strains diagnosed?
Signup and view all the flashcards
Who is most at risk for fractures?
Who is most at risk for fractures?
Signup and view all the flashcards
What causes high-energy trauma?
What causes high-energy trauma?
Signup and view all the flashcards
What is bone?
What is bone?
Signup and view all the flashcards
How do you treat pain associated with fractures?
How do you treat pain associated with fractures?
Signup and view all the flashcards
What is a closed reduction?
What is a closed reduction?
Signup and view all the flashcards
What are signs of excessive bleeding?
What are signs of excessive bleeding?
Signup and view all the flashcards
What does myoglobin measure in fractures?
What does myoglobin measure in fractures?
Signup and view all the flashcards
How does rhabdomyolysis affect urine?
How does rhabdomyolysis affect urine?
Signup and view all the flashcards
What is Malunion?
What is Malunion?
Signup and view all the flashcards
How can you reduce pain and edema from a fracture?
How can you reduce pain and edema from a fracture?
Signup and view all the flashcards
Why are pulmonary emboli so dangerous?
Why are pulmonary emboli so dangerous?
Signup and view all the flashcards
Rhabdomyolysis symptoms
Rhabdomyolysis symptoms
Signup and view all the flashcards
Why is wound care so vital?
Why is wound care so vital?
Signup and view all the flashcards
Connective Tissue
Connective Tissue
Signup and view all the flashcards
Ground substance
Ground substance
Signup and view all the flashcards
Fibroblasts
Fibroblasts
Signup and view all the flashcards
Weight-bearing joints
Weight-bearing joints
Signup and view all the flashcards
Modifiable risk factor
Modifiable risk factor
Signup and view all the flashcards
Occupational Risk (OA)
Occupational Risk (OA)
Signup and view all the flashcards
OA: Knee Demographic
OA: Knee Demographic
Signup and view all the flashcards
OA: Hand Demographic
OA: Hand Demographic
Signup and view all the flashcards
Viscoelastic
Viscoelastic
Signup and view all the flashcards
Cartilage Composition
Cartilage Composition
Signup and view all the flashcards
Cartilage Function
Cartilage Function
Signup and view all the flashcards
Osteophytes
Osteophytes
Signup and view all the flashcards
Heberden's nodes
Heberden's nodes
Signup and view all the flashcards
Bouchard's nodes
Bouchard's nodes
Signup and view all the flashcards
Cartilage Loss
Cartilage Loss
Signup and view all the flashcards
Synovium
Synovium
Signup and view all the flashcards
OA Synovium Effects
OA Synovium Effects
Signup and view all the flashcards
Chronic Effusions
Chronic Effusions
Signup and view all the flashcards
Joint Laxity
Joint Laxity
Signup and view all the flashcards
Muscle Atrophy
Muscle Atrophy
Signup and view all the flashcards
OA Clinical Signs
OA Clinical Signs
Signup and view all the flashcards
Crepitus in Joints
Crepitus in Joints
Signup and view all the flashcards
OA Diagnosis Criteria
OA Diagnosis Criteria
Signup and view all the flashcards
Osteoarthritis Diagnosis
Osteoarthritis Diagnosis
Signup and view all the flashcards
Radiographic OA sign
Radiographic OA sign
Signup and view all the flashcards
OA Treatment Goals
OA Treatment Goals
Signup and view all the flashcards
Non-Pharm OA Therapy
Non-Pharm OA Therapy
Signup and view all the flashcards
Hand OA Criteria
Hand OA Criteria
Signup and view all the flashcards
OA Pharmacological Interventions
OA Pharmacological Interventions
Signup and view all the flashcards
NSAID Action
NSAID Action
Signup and view all the flashcards
NSAID Monitoring Risks
NSAID Monitoring Risks
Signup and view all the flashcards
Acetaminophen Action
Acetaminophen Action
Signup and view all the flashcards
Geriatric NSAID Cautions
Geriatric NSAID Cautions
Signup and view all the flashcards
OA Complications
OA Complications
Signup and view all the flashcards
Surgical Fusion
Surgical Fusion
Signup and view all the flashcards
Total Joint Replacement
Total Joint Replacement
Signup and view all the flashcards
Osteoarthritis Prevalence
Osteoarthritis Prevalence
Signup and view all the flashcards
Osteoarthritis Joint Impact
Osteoarthritis Joint Impact
Signup and view all the flashcards
Shoulder and Elbow OA
Shoulder and Elbow OA
Signup and view all the flashcards
Osteoarthritis Risk Factors
Osteoarthritis Risk Factors
Signup and view all the flashcards
Obesity and Osteoarthritis
Obesity and Osteoarthritis
Signup and view all the flashcards
Occupation and OA
Occupation and OA
Signup and view all the flashcards
Osteoarthritis and Age
Osteoarthritis and Age
Signup and view all the flashcards
OA Joint Dynamics
OA Joint Dynamics
Signup and view all the flashcards
OA Affected Joint Structures
OA Affected Joint Structures
Signup and view all the flashcards
Cartilage's Role
Cartilage's Role
Signup and view all the flashcards
Osteophytes Formation
Osteophytes Formation
Signup and view all the flashcards
Hand Joint Deformities
Hand Joint Deformities
Signup and view all the flashcards
Synovium Definition
Synovium Definition
Signup and view all the flashcards
Synovial Fluid and Laxity
Synovial Fluid and Laxity
Signup and view all the flashcards
Osteoarthritis Symptoms
Osteoarthritis Symptoms
Signup and view all the flashcards
Crepitus Definition
Crepitus Definition
Signup and view all the flashcards
OA Lab Testing Purpose
OA Lab Testing Purpose
Signup and view all the flashcards
OA Non-drug Therapies
OA Non-drug Therapies
Signup and view all the flashcards
Non-Pharmacologic OA Treatments
Non-Pharmacologic OA Treatments
Signup and view all the flashcards
Hip OA Criteria
Hip OA Criteria
Signup and view all the flashcards
Knee OA Criteria
Knee OA Criteria
Signup and view all the flashcards
OA Pain Relief
OA Pain Relief
Signup and view all the flashcards
NSAID Side Effects
NSAID Side Effects
Signup and view all the flashcards
OA Refractory Treatment
OA Refractory Treatment
Signup and view all the flashcards
Intra-articular Action
Intra-articular Action
Signup and view all the flashcards
NSAID Concomitant Use
NSAID Concomitant Use
Signup and view all the flashcards
Opioid Analgesics Action
Opioid Analgesics Action
Signup and view all the flashcards
NSAID Risks
NSAID Risks
Signup and view all the flashcards
NSAID Monitoring
NSAID Monitoring
Signup and view all the flashcards
NSAID Contraindications
NSAID Contraindications
Signup and view all the flashcards
Polypharmacy Definition
Polypharmacy Definition
Signup and view all the flashcards
NSAIDs Interactions
NSAIDs Interactions
Signup and view all the flashcards
OA Surgical Interventions
OA Surgical Interventions
Signup and view all the flashcards
Arthroscopic Irrigation
Arthroscopic Irrigation
Signup and view all the flashcards
Synovectomy Action
Synovectomy Action
Signup and view all the flashcards
Surgical Fusion Goal
Surgical Fusion Goal
Signup and view all the flashcards
NSAID Action Explanation
NSAID Action Explanation
Signup and view all the flashcards
Home Healthcare Referral
Home Healthcare Referral
Signup and view all the flashcards
Osteoporosis
Osteoporosis
Signup and view all the flashcards
Primary Risk Factors (Osteoporosis)
Primary Risk Factors (Osteoporosis)
Signup and view all the flashcards
Secondary Risk Factors (Osteoporosis)
Secondary Risk Factors (Osteoporosis)
Signup and view all the flashcards
Bone Loss (Osteoporosis)
Bone Loss (Osteoporosis)
Signup and view all the flashcards
Common Fracture Sites (Osteoporosis)
Common Fracture Sites (Osteoporosis)
Signup and view all the flashcards
Treatment Focus (Osteoporosis)
Treatment Focus (Osteoporosis)
Signup and view all the flashcards
Osteomyelitis
Osteomyelitis
Signup and view all the flashcards
Hematogenous Osteomyelitis
Hematogenous Osteomyelitis
Signup and view all the flashcards
Nonhematogenous Osteomyelitis
Nonhematogenous Osteomyelitis
Signup and view all the flashcards
Osteomyelitis Diagnosis
Osteomyelitis Diagnosis
Signup and view all the flashcards
Medical Management (Osteomyelitis)
Medical Management (Osteomyelitis)
Signup and view all the flashcards
Surgical Debridement (Osteomyelitis)
Surgical Debridement (Osteomyelitis)
Signup and view all the flashcards
Manifestations of Osteomyelitis
Manifestations of Osteomyelitis
Signup and view all the flashcards
Primary Risk Factors for Osteoporosis
Primary Risk Factors for Osteoporosis
Signup and view all the flashcards
Secondary Risk Factors for Osteoporosis
Secondary Risk Factors for Osteoporosis
Signup and view all the flashcards
Pathophysiology of Osteoporosis
Pathophysiology of Osteoporosis
Signup and view all the flashcards
Clinical Manifestation of Osteoporosis
Clinical Manifestation of Osteoporosis
Signup and view all the flashcards
Treatment Focus for Osteoporosis
Treatment Focus for Osteoporosis
Signup and view all the flashcards
Lifestyle Changes for Osteoporosis Prevention
Lifestyle Changes for Osteoporosis Prevention
Signup and view all the flashcards
Complications of Osteoporosis
Complications of Osteoporosis
Signup and view all the flashcards
Classifications of Osteomyelitis
Classifications of Osteomyelitis
Signup and view all the flashcards
Common Pathogens in Contiguous Osteomyelitis
Common Pathogens in Contiguous Osteomyelitis
Signup and view all the flashcards
Clinical Hallmark of Osteomyelitis
Clinical Hallmark of Osteomyelitis
Signup and view all the flashcards
Manifestations of Acute Osteomyelitis
Manifestations of Acute Osteomyelitis
Signup and view all the flashcards
H2-Receptor Antigen Antagonists (H2RAs)
H2-Receptor Antigen Antagonists (H2RAs)
Signup and view all the flashcards
Proton Pump Inhibitors (PPIs)
Proton Pump Inhibitors (PPIs)
Signup and view all the flashcards
Common H2-blockers
Common H2-blockers
Signup and view all the flashcards
OTC PPIs
OTC PPIs
Signup and view all the flashcards
Drug Interactions
Drug Interactions
Signup and view all the flashcards
H2RAs Action
H2RAs Action
Signup and view all the flashcards
PPIs Action
PPIs Action
Signup and view all the flashcards
Proton pump inhibitors action
Proton pump inhibitors action
Signup and view all the flashcards
Antacid Administration
Antacid Administration
Signup and view all the flashcards
H2-Receptor Antagonists (H2RAs)
H2-Receptor Antagonists (H2RAs)
Signup and view all the flashcards
Cimetidine Interactions
Cimetidine Interactions
Signup and view all the flashcards
PPI Interactions
PPI Interactions
Signup and view all the flashcards
H2RAs Onset and Duration
H2RAs Onset and Duration
Signup and view all the flashcards
PPIs Onset and Duration
PPIs Onset and Duration
Signup and view all the flashcards
Antacids
Antacids
Signup and view all the flashcards
How PPIs Work
How PPIs Work
Signup and view all the flashcards
How H2 Blockers Work
How H2 Blockers Work
Signup and view all the flashcards
Misoprostol action
Misoprostol action
Signup and view all the flashcards
Osteoarthritis (Epidemiology)
Osteoarthritis (Epidemiology)
Signup and view all the flashcards
OA: Risk Factors
OA: Risk Factors
Signup and view all the flashcards
Osteoarthritis: Clinical Manifestations
Osteoarthritis: Clinical Manifestations
Signup and view all the flashcards
OA: Diagnosis
OA: Diagnosis
Signup and view all the flashcards
OA: Medications
OA: Medications
Signup and view all the flashcards
Osteoarthritis: Complications
Osteoarthritis: Complications
Signup and view all the flashcards
OA: Surgical Management
OA: Surgical Management
Signup and view all the flashcards
Osteoarthritis: Nursing Interventions
Osteoarthritis: Nursing Interventions
Signup and view all the flashcards
Rheumatoid Arthritis: Pathophysiology
Rheumatoid Arthritis: Pathophysiology
Signup and view all the flashcards
Rheumatoid Arthritis: Risk factors
Rheumatoid Arthritis: Risk factors
Signup and view all the flashcards
RA: Clinical Manifestations
RA: Clinical Manifestations
Signup and view all the flashcards
Rheumatoid Arthritis: Diagnosis
Rheumatoid Arthritis: Diagnosis
Signup and view all the flashcards
RA: Treatment
RA: Treatment
Signup and view all the flashcards
Rheumatoid Arthritis: Medications
Rheumatoid Arthritis: Medications
Signup and view all the flashcards
Rheumatoid Arthritis: Surgical Management
Rheumatoid Arthritis: Surgical Management
Signup and view all the flashcards
Rheumatoid Arthritis: Complications
Rheumatoid Arthritis: Complications
Signup and view all the flashcards
Rheumatoid Arthritis: Nursing Diagnoses
Rheumatoid Arthritis: Nursing Diagnoses
Signup and view all the flashcards
RA: Nursing Interventions-Teaching
RA: Nursing Interventions-Teaching
Signup and view all the flashcards
Musculoskeletal functions
Musculoskeletal functions
Signup and view all the flashcards
Periosteum
Periosteum
Signup and view all the flashcards
Hormonal bone influences
Hormonal bone influences
Signup and view all the flashcards
Muscle Types
Muscle Types
Signup and view all the flashcards
Joint Definition
Joint Definition
Signup and view all the flashcards
Synovial Joints
Synovial Joints
Signup and view all the flashcards
Cartilage Definition
Cartilage Definition
Signup and view all the flashcards
Bursa
Bursa
Signup and view all the flashcards
Ligament Function
Ligament Function
Signup and view all the flashcards
Tendon Function
Tendon Function
Signup and view all the flashcards
Fascia Definition
Fascia Definition
Signup and view all the flashcards
Gait
Gait
Signup and view all the flashcards
Goniometry
Goniometry
Signup and view all the flashcards
Neurovascular Assessment
Neurovascular Assessment
Signup and view all the flashcards
Relevant Lab Studies
Relevant Lab Studies
Signup and view all the flashcards
X-Ray
X-Ray
Signup and view all the flashcards
Computed Tomography
Computed Tomography
Signup and view all the flashcards
Magnetic Resonance Imaging
Magnetic Resonance Imaging
Signup and view all the flashcards
Arthroscopy
Arthroscopy
Signup and view all the flashcards
Arthrocentesis
Arthrocentesis
Signup and view all the flashcards
Sarcopenia
Sarcopenia
Signup and view all the flashcards
Bones function
Bones function
Signup and view all the flashcards
Spinal curvatures
Spinal curvatures
Signup and view all the flashcards
Osteoporosis Pathophysiology
Osteoporosis Pathophysiology
Signup and view all the flashcards
Osteoblasts
Osteoblasts
Signup and view all the flashcards
Osteoclasts
Osteoclasts
Signup and view all the flashcards
Primary Osteoporosis Risk Factors
Primary Osteoporosis Risk Factors
Signup and view all the flashcards
Secondary Osteoporosis Risk Factors
Secondary Osteoporosis Risk Factors
Signup and view all the flashcards
Pathologic Fractures
Pathologic Fractures
Signup and view all the flashcards
Bone Density Tests
Bone Density Tests
Signup and view all the flashcards
Quantitative Computed Tomography (QCT)
Quantitative Computed Tomography (QCT)
Signup and view all the flashcards
Vitamin D and Calcium
Vitamin D and Calcium
Signup and view all the flashcards
Surgical Debridement
Surgical Debridement
Signup and view all the flashcards
Surgical Management of Osteomyelitis
Surgical Management of Osteomyelitis
Signup and view all the flashcards
Arthroplasty
Arthroplasty
Signup and view all the flashcards
Osteomyelitis Pain
Osteomyelitis Pain
Signup and view all the flashcards
Radiological Studies (Osteomyelitis)
Radiological Studies (Osteomyelitis)
Signup and view all the flashcards
Antibiotic therapy
Antibiotic therapy
Signup and view all the flashcards
Musculoskeletal trauma
Musculoskeletal trauma
Signup and view all the flashcards
Epidemiology
Epidemiology
Signup and view all the flashcards
Pathophysiological processes
Pathophysiological processes
Signup and view all the flashcards
Ecchymosis
Ecchymosis
Signup and view all the flashcards
Meniscus Injury: Medial Tear
Meniscus Injury: Medial Tear
Signup and view all the flashcards
Meniscus Injury: Lateral Tear
Meniscus Injury: Lateral Tear
Signup and view all the flashcards
McMurray's test
McMurray's test
Signup and view all the flashcards
Carpal Tunnel Syndrome
Carpal Tunnel Syndrome
Signup and view all the flashcards
Phalen's test
Phalen's test
Signup and view all the flashcards
Immobilization devices
Immobilization devices
Signup and view all the flashcards
Fat Embolism symptoms
Fat Embolism symptoms
Signup and view all the flashcards
Phantom limb pain
Phantom limb pain
Signup and view all the flashcards
Painful nerve tumor after amputation
Painful nerve tumor after amputation
Signup and view all the flashcards
GI System Function
GI System Function
Signup and view all the flashcards
Mouth Function
Mouth Function
Signup and view all the flashcards
Esophagus Function
Esophagus Function
Signup and view all the flashcards
Stomach's Role
Stomach's Role
Signup and view all the flashcards
Liver Function
Liver Function
Signup and view all the flashcards
Gallbladder Function
Gallbladder Function
Signup and view all the flashcards
Pancreas
Pancreas
Signup and view all the flashcards
Small Intestine
Small Intestine
Signup and view all the flashcards
Large Intestine
Large Intestine
Signup and view all the flashcards
GI System History
GI System History
Signup and view all the flashcards
MUST Tool Factors
MUST Tool Factors
Signup and view all the flashcards
Abdominal Assessment Order
Abdominal Assessment Order
Signup and view all the flashcards
Oral Mucosa Inspection
Oral Mucosa Inspection
Signup and view all the flashcards
Auscultating Bowel Sounds
Auscultating Bowel Sounds
Signup and view all the flashcards
Percussion Sounds
Percussion Sounds
Signup and view all the flashcards
Abdominal Palpation
Abdominal Palpation
Signup and view all the flashcards
Splenomegaly Definition
Splenomegaly Definition
Signup and view all the flashcards
GI Lab Studies
GI Lab Studies
Signup and view all the flashcards
GI Imaging Studies
GI Imaging Studies
Signup and view all the flashcards
Hiatal Hernia
Hiatal Hernia
Signup and view all the flashcards
Hiatal Hernia Epidemiology
Hiatal Hernia Epidemiology
Signup and view all the flashcards
Hiatal Hernia Type 1 Symptoms
Hiatal Hernia Type 1 Symptoms
Signup and view all the flashcards
Hiatal Hernia Type 2 Symptoms
Hiatal Hernia Type 2 Symptoms
Signup and view all the flashcards
Hiatal Hernia Diagnosis
Hiatal Hernia Diagnosis
Signup and view all the flashcards
Hiatal Hernia Medications
Hiatal Hernia Medications
Signup and view all the flashcards
Hiatal Hernia Nursing Assessments
Hiatal Hernia Nursing Assessments
Signup and view all the flashcards
Hiatal Hernia Teaching
Hiatal Hernia Teaching
Signup and view all the flashcards
GERD Pathophysiology
GERD Pathophysiology
Signup and view all the flashcards
GERD Symptoms
GERD Symptoms
Signup and view all the flashcards
GERD Risk Factors
GERD Risk Factors
Signup and view all the flashcards
GERD Diagnosis
GERD Diagnosis
Signup and view all the flashcards
GERD Medications
GERD Medications
Signup and view all the flashcards
GERD Nursing Actions
GERD Nursing Actions
Signup and view all the flashcards
GERD Assessment
GERD Assessment
Signup and view all the flashcards
Gastritis
Gastritis
Signup and view all the flashcards
Epigastric pain
Epigastric pain
Signup and view all the flashcards
Biopsy (Gastric)
Biopsy (Gastric)
Signup and view all the flashcards
Upper GI X-ray Series
Upper GI X-ray Series
Signup and view all the flashcards
Urea Breath Testing
Urea Breath Testing
Signup and view all the flashcards
Vagotomy
Vagotomy
Signup and view all the flashcards
Gastrectomy
Gastrectomy
Signup and view all the flashcards
Gastroenteritis
Gastroenteritis
Signup and view all the flashcards
Stool Culture
Stool Culture
Signup and view all the flashcards
Poor Skin Turgor
Poor Skin Turgor
Signup and view all the flashcards
Hyperactive Bowel Sounds
Hyperactive Bowel Sounds
Signup and view all the flashcards
Peptic Ulcer Disease (PUD)
Peptic Ulcer Disease (PUD)
Signup and view all the flashcards
Damage to Gastric Mucosa
Damage to Gastric Mucosa
Signup and view all the flashcards
Erosions Due to Gastric Acid
Erosions Due to Gastric Acid
Signup and view all the flashcards
Upper GI Endoscopy
Upper GI Endoscopy
Signup and view all the flashcards
H2 Receptor Antagonists
H2 Receptor Antagonists
Signup and view all the flashcards
Nonhealing Ulcers Indications
Nonhealing Ulcers Indications
Signup and view all the flashcards
Bleeding Ulcers Indications
Bleeding Ulcers Indications
Signup and view all the flashcards
Peptic Ulcer
Peptic Ulcer
Signup and view all the flashcards
Gastric Juice corrosive action
Gastric Juice corrosive action
Signup and view all the flashcards
Medical management of PUD
Medical management of PUD
Signup and view all the flashcards
Hernia definition
Hernia definition
Signup and view all the flashcards
Hernia causes
Hernia causes
Signup and view all the flashcards
Hernia diagnosis
Hernia diagnosis
Signup and view all the flashcards
Hernia symptoms
Hernia symptoms
Signup and view all the flashcards
Hemorrhoids Definition
Hemorrhoids Definition
Signup and view all the flashcards
Hemorrhoid diagnosis
Hemorrhoid diagnosis
Signup and view all the flashcards
Hemorrhoid symptoms
Hemorrhoid symptoms
Signup and view all the flashcards
IBS definition
IBS definition
Signup and view all the flashcards
IBS Diagnosis
IBS Diagnosis
Signup and view all the flashcards
Medical Management for IBS
Medical Management for IBS
Signup and view all the flashcards
IBS symptoms
IBS symptoms
Signup and view all the flashcards
IBD Definition
IBD Definition
Signup and view all the flashcards
IBD diagnosis
IBD diagnosis
Signup and view all the flashcards
IBD symptoms
IBD symptoms
Signup and view all the flashcards
Celiac Disease Definition
Celiac Disease Definition
Signup and view all the flashcards
Celiac diagnosis
Celiac diagnosis
Signup and view all the flashcards
Celiac treatment
Celiac treatment
Signup and view all the flashcards
Celiac Symptoms
Celiac Symptoms
Signup and view all the flashcards
Diverticulitis definition
Diverticulitis definition
Signup and view all the flashcards
Diverticulitis Diagnosis
Diverticulitis Diagnosis
Signup and view all the flashcards
Diverticulitis Treatment
Diverticulitis Treatment
Signup and view all the flashcards
Diverticulitis Symptoms
Diverticulitis Symptoms
Signup and view all the flashcards
Appendicitis definition
Appendicitis definition
Signup and view all the flashcards
Appendicitis diagnosis
Appendicitis diagnosis
Signup and view all the flashcards
Appendicitis symptoms
Appendicitis symptoms
Signup and view all the flashcards
Who Does Celiac Disease Affect
Who Does Celiac Disease Affect
Signup and view all the flashcards
Diverticulitis diet
Diverticulitis diet
Signup and view all the flashcards
What is BMI?
What is BMI?
Signup and view all the flashcards
Underweight BMI
Underweight BMI
Signup and view all the flashcards
Normal weight BMI
Normal weight BMI
Signup and view all the flashcards
Overweight BMI
Overweight BMI
Signup and view all the flashcards
Obesity (Class 1) BMI
Obesity (Class 1) BMI
Signup and view all the flashcards
Obesity (Class 2) BMI
Obesity (Class 2) BMI
Signup and view all the flashcards
Extreme Obesity (Class 3) BMI
Extreme Obesity (Class 3) BMI
Signup and view all the flashcards
Long-term Obesity Effects
Long-term Obesity Effects
Signup and view all the flashcards
Obesity Pathophysiology
Obesity Pathophysiology
Signup and view all the flashcards
What is Leptin?
What is Leptin?
Signup and view all the flashcards
Diagnosis Laboratory Testing
Diagnosis Laboratory Testing
Signup and view all the flashcards
Obesity Treatment
Obesity Treatment
Signup and view all the flashcards
Bariatric Surgery Types
Bariatric Surgery Types
Signup and view all the flashcards
Adjustable Gastric Banding (AGB)
Adjustable Gastric Banding (AGB)
Signup and view all the flashcards
Roux-en-Y Gastric Bypass (RYGB)
Roux-en-Y Gastric Bypass (RYGB)
Signup and view all the flashcards
Bariatric Surgery Complications
Bariatric Surgery Complications
Signup and view all the flashcards
Nursing Management
Nursing Management
Signup and view all the flashcards
Post-Op Nursing Diagnoses
Post-Op Nursing Diagnoses
Signup and view all the flashcards
Post-Op Assessments
Post-Op Assessments
Signup and view all the flashcards
Nursing Interventions: Actions
Nursing Interventions: Actions
Signup and view all the flashcards
Post-Op Teaching
Post-Op Teaching
Signup and view all the flashcards
Desired Outcomes
Desired Outcomes
Signup and view all the flashcards
Metabolic Syndrome
Metabolic Syndrome
Signup and view all the flashcards
Positive Energy Balance
Positive Energy Balance
Signup and view all the flashcards
Negative Energy Balance
Negative Energy Balance
Signup and view all the flashcards
Calculate BMI
Calculate BMI
Signup and view all the flashcards
Anxiety
Anxiety
Signup and view all the flashcards
Fear
Fear
Signup and view all the flashcards
Normal anxiety
Normal anxiety
Signup and view all the flashcards
Mild Anxiety
Mild Anxiety
Signup and view all the flashcards
Moderate Anxiety
Moderate Anxiety
Signup and view all the flashcards
Severe Anxiety
Severe Anxiety
Signup and view all the flashcards
Panic
Panic
Signup and view all the flashcards
Defense mechanisms
Defense mechanisms
Signup and view all the flashcards
Adaptive defense mechanisms
Adaptive defense mechanisms
Signup and view all the flashcards
Maladaptive defense mechanisms
Maladaptive defense mechanisms
Signup and view all the flashcards
Separation anxiety disorder
Separation anxiety disorder
Signup and view all the flashcards
Specific Phobia
Specific Phobia
Signup and view all the flashcards
Social anxiety disorder
Social anxiety disorder
Signup and view all the flashcards
Panic disorder
Panic disorder
Signup and view all the flashcards
Panic attacks
Panic attacks
Signup and view all the flashcards
Agoraphobia
Agoraphobia
Signup and view all the flashcards
Generalized anxiety disorder
Generalized anxiety disorder
Signup and view all the flashcards
Selective mutism
Selective mutism
Signup and view all the flashcards
Substance-induced anxiety disorder
Substance-induced anxiety disorder
Signup and view all the flashcards
Anxiety due to a medical condition
Anxiety due to a medical condition
Signup and view all the flashcards
Genetic risk factors (anxiety)
Genetic risk factors (anxiety)
Signup and view all the flashcards
Biological risk factors (anxiety)
Biological risk factors (anxiety)
Signup and view all the flashcards
Psychological risk factors (anxiety)
Psychological risk factors (anxiety)
Signup and view all the flashcards
Cultural risk factors (anxiety)
Cultural risk factors (anxiety)
Signup and view all the flashcards
Nursing process steps
Nursing process steps
Signup and view all the flashcards
Assessment (anxiety)
Assessment (anxiety)
Signup and view all the flashcards
Anxiety assessment (detailed)
Anxiety assessment (detailed)
Signup and view all the flashcards
Anxiety & Fear-related nursing outcome
Anxiety & Fear-related nursing outcome
Signup and view all the flashcards
Difficulty coping-related nursing outcome
Difficulty coping-related nursing outcome
Signup and view all the flashcards
Impaired socialization & low self-esteem-related nursing outcome
Impaired socialization & low self-esteem-related nursing outcome
Signup and view all the flashcards
Planning (anxiety)
Planning (anxiety)
Signup and view all the flashcards
Implementation (anxiety)
Implementation (anxiety)
Signup and view all the flashcards
Nursing implementation (anxiety)
Nursing implementation (anxiety)
Signup and view all the flashcards
Evaluation (anxiety)
Evaluation (anxiety)
Signup and view all the flashcards
Psychological therapies (anxiety)
Psychological therapies (anxiety)
Signup and view all the flashcards
Obsessions
Obsessions
Signup and view all the flashcards
Compulsions
Compulsions
Signup and view all the flashcards
DSM-5 Criteria for Obsessive-Compulsive Disorder
DSM-5 Criteria for Obsessive-Compulsive Disorder
Signup and view all the flashcards
Body Dysmorphic Disorder
Body Dysmorphic Disorder
Signup and view all the flashcards
Hoarding Disorder
Hoarding Disorder
Signup and view all the flashcards
Trichotillomania and Excoriation Disorder
Trichotillomania and Excoriation Disorder
Signup and view all the flashcards
Other Compulsive Disorders
Other Compulsive Disorders
Signup and view all the flashcards
Risk Factors for OCD
Risk Factors for OCD
Signup and view all the flashcards
Outcomes Identification (Patients)
Outcomes Identification (Patients)
Signup and view all the flashcards
Basic-Level Nursing Interventions
Basic-Level Nursing Interventions
Signup and view all the flashcards
Advanced Practice Nursing Interventions
Advanced Practice Nursing Interventions
Signup and view all the flashcards
Surgical treatment ocd
Surgical treatment ocd
Signup and view all the flashcards
Brain stimulation ocd
Brain stimulation ocd
Signup and view all the flashcards
exposure and response prevention
exposure and response prevention
Signup and view all the flashcards
Trauma-Related Disorders
Trauma-Related Disorders
Signup and view all the flashcards
Reactive Attachment Disorder
Reactive Attachment Disorder
Signup and view all the flashcards
Disinhibited Social Engagement Disorder
Disinhibited Social Engagement Disorder
Signup and view all the flashcards
Neuroplasticity
Neuroplasticity
Signup and view all the flashcards
Neurobiological Trauma Impact
Neurobiological Trauma Impact
Signup and view all the flashcards
Flashbacks (PTSD)
Flashbacks (PTSD)
Signup and view all the flashcards
Avoidance (PTSD)
Avoidance (PTSD)
Signup and view all the flashcards
Hypervigilance (PTSD)
Hypervigilance (PTSD)
Signup and view all the flashcards
Acute Stress Disorder
Acute Stress Disorder
Signup and view all the flashcards
Numbing (ASD)
Numbing (ASD)
Signup and view all the flashcards
Derealization
Derealization
Signup and view all the flashcards
Adjustment Disorder
Adjustment Disorder
Signup and view all the flashcards
Dissociative Amnesia
Dissociative Amnesia
Signup and view all the flashcards
Dissociative Fugue
Dissociative Fugue
Signup and view all the flashcards
Depersonalization
Depersonalization
Signup and view all the flashcards
Dissociative Identity Disorder
Dissociative Identity Disorder
Signup and view all the flashcards
Anorexia Nervosa
Anorexia Nervosa
Signup and view all the flashcards
Purging
Purging
Signup and view all the flashcards
Anorexia Nervosa Criteria
Anorexia Nervosa Criteria
Signup and view all the flashcards
Restricting Type Anorexia
Restricting Type Anorexia
Signup and view all the flashcards
Binge-eating/Purging Type Anorexia
Binge-eating/Purging Type Anorexia
Signup and view all the flashcards
Bulimia Nervosa
Bulimia Nervosa
Signup and view all the flashcards
Lifetime Prevalence
Lifetime Prevalence
Signup and view all the flashcards
Comorbidity
Comorbidity
Signup and view all the flashcards
Perception of the Problem
Perception of the Problem
Signup and view all the flashcards
Refeeding Syndrome
Refeeding Syndrome
Signup and view all the flashcards
Suicidal Ideation (Anorexia)
Suicidal Ideation (Anorexia)
Signup and view all the flashcards
Anorexia Nervosa Interventions
Anorexia Nervosa Interventions
Signup and view all the flashcards
Pharmacotherapy
Pharmacotherapy
Signup and view all the flashcards
Binge Eating Disorder
Binge Eating Disorder
Signup and view all the flashcards
Inpatient Care (Bulimia)
Inpatient Care (Bulimia)
Signup and view all the flashcards
Pharmacotherapy (Bulimia)
Pharmacotherapy (Bulimia)
Signup and view all the flashcards
Rumination Disorder
Rumination Disorder
Signup and view all the flashcards
Avoidant/Restrictive Food Intake Disorder
Avoidant/Restrictive Food Intake Disorder
Signup and view all the flashcards
Hypoalbuminemia
Hypoalbuminemia
Signup and view all the flashcards
Inflammation
Inflammation
Signup and view all the flashcards
NSAIDs
NSAIDs
Signup and view all the flashcards
NSAIDs uses
NSAIDs uses
Signup and view all the flashcards
Antipyretic
Antipyretic
Signup and view all the flashcards
Salicylates
Salicylates
Signup and view all the flashcards
Aspirin for MI
Aspirin for MI
Signup and view all the flashcards
NSAIDs Adverse Effects
NSAIDs Adverse Effects
Signup and view all the flashcards
Misoprostol (Cytotec)
Misoprostol (Cytotec)
Signup and view all the flashcards
NSAIDs and Renal Function
NSAIDs and Renal Function
Signup and view all the flashcards
NSAIDs Black Box Warning
NSAIDs Black Box Warning
Signup and view all the flashcards
Salicylic Acid
Salicylic Acid
Signup and view all the flashcards
Salicylates Risk
Salicylates Risk
Signup and view all the flashcards
Aspirin Uses
Aspirin Uses
Signup and view all the flashcards
Aspirin Reye's Syndrome
Aspirin Reye's Syndrome
Signup and view all the flashcards
Salicylate Toxicity
Salicylate Toxicity
Signup and view all the flashcards
Indomethacin (Indocin)
Indomethacin (Indocin)
Signup and view all the flashcards
Ketorolac (Toradol)
Ketorolac (Toradol)
Signup and view all the flashcards
Celecoxib (Celebrex)
Celecoxib (Celebrex)
Signup and view all the flashcards
Enolic Acid Derivatives
Enolic Acid Derivatives
Signup and view all the flashcards
Allopurinol (Zyloprim)
Allopurinol (Zyloprim)
Signup and view all the flashcards
Probenecid (Benemid)
Probenecid (Benemid)
Signup and view all the flashcards
Febuxostat (Uloric)
Febuxostat (Uloric)
Signup and view all the flashcards
Colchicine
Colchicine
Signup and view all the flashcards
Lesinurad Patient Teaching
Lesinurad Patient Teaching
Signup and view all the flashcards
Glucosamine & Chondroitin
Glucosamine & Chondroitin
Signup and view all the flashcards
Pre-NSAID Assessment
Pre-NSAID Assessment
Signup and view all the flashcards
NSAIDs Assessment
NSAIDs Assessment
Signup and view all the flashcards
Glucosamine & Chondroitin Adverse Effects
Glucosamine & Chondroitin Adverse Effects
Signup and view all the flashcards
Hydrochloric Acid (HCl)
Hydrochloric Acid (HCl)
Signup and view all the flashcards
Glands of the Stomach
Glands of the Stomach
Signup and view all the flashcards
Parietal cells
Parietal cells
Signup and view all the flashcards
Chief cells
Chief cells
Signup and view all the flashcards
Mucous cells
Mucous cells
Signup and view all the flashcards
Helicobacter pylori (H. pylori)
Helicobacter pylori (H. pylori)
Signup and view all the flashcards
H₂ Antagonists
H₂ Antagonists
Signup and view all the flashcards
Diarrhea
Diarrhea
Signup and view all the flashcards
Acute Diarrhea
Acute Diarrhea
Signup and view all the flashcards
Chronic Diarrhea
Chronic Diarrhea
Signup and view all the flashcards
Causes of Acute Diarrhea
Causes of Acute Diarrhea
Signup and view all the flashcards
Causes of Chronic Diarrhea
Causes of Chronic Diarrhea
Signup and view all the flashcards
Goals of Diarrhea Treatment
Goals of Diarrhea Treatment
Signup and view all the flashcards
Antidiarrheals: Adsorbents
Antidiarrheals: Adsorbents
Signup and view all the flashcards
Examples of Adsorbents
Examples of Adsorbents
Signup and view all the flashcards
Antimotility drugs: Anticholinergics
Antimotility drugs: Anticholinergics
Signup and view all the flashcards
Antimotility drugs: Opiates
Antimotility drugs: Opiates
Signup and view all the flashcards
Antidiarrheals: Probiotics
Antidiarrheals: Probiotics
Signup and view all the flashcards
Antidiarrheals: Probiotics
Antidiarrheals: Probiotics
Signup and view all the flashcards
Adsorbents: Adverse Effects
Adsorbents: Adverse Effects
Signup and view all the flashcards
Anticholinergics: Adverse Effects
Anticholinergics: Adverse Effects
Signup and view all the flashcards
Opiates: Adverse Effects
Opiates: Adverse Effects
Signup and view all the flashcards
Adsorbents Interactions
Adsorbents Interactions
Signup and view all the flashcards
Adsorbents + Anticoagulants Interaction
Adsorbents + Anticoagulants Interaction
Signup and view all the flashcards
Adsorbent Contraindications
Adsorbent Contraindications
Signup and view all the flashcards
Adsorbent Nursing Care
Adsorbent Nursing Care
Signup and view all the flashcards
Constipation
Constipation
Signup and view all the flashcards
Nonsurgical Constipation Treatment
Nonsurgical Constipation Treatment
Signup and view all the flashcards
Laxatives: Bulk Forming
Laxatives: Bulk Forming
Signup and view all the flashcards
Laxatives: Emollient
Laxatives: Emollient
Signup and view all the flashcards
Laxatives: Hyperosmotic
Laxatives: Hyperosmotic
Signup and view all the flashcards
Laxatives: Saline
Laxatives: Saline
Signup and view all the flashcards
Laxatives: Stimulant
Laxatives: Stimulant
Signup and view all the flashcards
Opioid Antagonists
Opioid Antagonists
Signup and view all the flashcards
Peripherally Acting
Peripherally Acting
Signup and view all the flashcards
Bulk Forming Uses
Bulk Forming Uses
Signup and view all the flashcards
Emollient Uses
Emollient Uses
Signup and view all the flashcards
Hyperosmotic Uses
Hyperosmotic Uses
Signup and view all the flashcards
Saline Uses
Saline Uses
Signup and view all the flashcards
Stimulant Uses
Stimulant Uses
Signup and view all the flashcards
Bulk Forming Downsides
Bulk Forming Downsides
Signup and view all the flashcards
Emollient Downsides
Emollient Downsides
Signup and view all the flashcards
Hyperosmotic Downsides
Hyperosmotic Downsides
Signup and view all the flashcards
Saline Downsides
Saline Downsides
Signup and view all the flashcards
Stimulant Downsides
Stimulant Downsides
Signup and view all the flashcards
Laxative Teaching
Laxative Teaching
Signup and view all the flashcards
Laxative encouragement
Laxative encouragement
Signup and view all the flashcards
Irritable Bowel Syndrome
Irritable Bowel Syndrome
Signup and view all the flashcards
Drugs for IBS-D (Diarrhea)
Drugs for IBS-D (Diarrhea)
Signup and view all the flashcards
Drugs for IBS-C (Constipation)
Drugs for IBS-C (Constipation)
Signup and view all the flashcards
Guideline Usage
Guideline Usage
Signup and view all the flashcards
Nausea
Nausea
Signup and view all the flashcards
Emesis (Vomiting)
Emesis (Vomiting)
Signup and view all the flashcards
Antiemetic drugs
Antiemetic drugs
Signup and view all the flashcards
Vomiting center (VC)
Vomiting center (VC)
Signup and view all the flashcards
Chemoreceptor trigger zone (CTZ)
Chemoreceptor trigger zone (CTZ)
Signup and view all the flashcards
Chemotherapy induced N/V
Chemotherapy induced N/V
Signup and view all the flashcards
Postoperative N/V
Postoperative N/V
Signup and view all the flashcards
Anticholinergic drugs
Anticholinergic drugs
Signup and view all the flashcards
Anticholinergic MOA
Anticholinergic MOA
Signup and view all the flashcards
Scopolamine
Scopolamine
Signup and view all the flashcards
Antihistamine antiemetic
Antihistamine antiemetic
Signup and view all the flashcards
Hydroxyzine
Hydroxyzine
Signup and view all the flashcards
Antidopaminergic drugs
Antidopaminergic drugs
Signup and view all the flashcards
Narrow-angle glaucoma
Narrow-angle glaucoma
Signup and view all the flashcards
Neurokinin antagonists
Neurokinin antagonists
Signup and view all the flashcards
Metoclopramide
Metoclopramide
Signup and view all the flashcards
Serotonin blockers
Serotonin blockers
Signup and view all the flashcards
Tetrahydrocannabinoids
Tetrahydrocannabinoids
Signup and view all the flashcards
Dronabinol
Dronabinol
Signup and view all the flashcards
Phosphorated carbohydrate solution
Phosphorated carbohydrate solution
Signup and view all the flashcards
Nursing intervention
Nursing intervention
Signup and view all the flashcards
Nursing intervention
Nursing intervention
Signup and view all the flashcards
Dronabinol's Use
Dronabinol's Use
Signup and view all the flashcards
Serotonin Blockers Use
Serotonin Blockers Use
Signup and view all the flashcards
QTc interval prolongation
QTc interval prolongation
Signup and view all the flashcards
Anticoagulant interaction
Anticoagulant interaction
Signup and view all the flashcards
Study Notes
- Nausea is an unpleasant feeling that often precedes vomiting.
- Emesis (vomiting) is the forcible emptying of gastric and sometimes intestinal contents.
- Antiemetic drugs are used to relieve nausea and vomiting.
- The vomiting center (VC) and chemoreceptor trigger zone (CTZ) are located in the brain.
- Stimulation of VC and CTZ can cause the vomiting reflex.
- Types of nausea and vomiting include:
- Chemotherapy induced
- Postoperative
- General
- Antiemetic and antinausea drugs include:
- Anticholinergic drugs
- Antihistamines (histamine 1 [H₁] receptor blockers)
- Antidopaminergic drugs
- Neurokinin antagonists
- Prokinetic drugs
- Serotonin blockers
- Tetrahydrocannabinoids
- Antiemetics and antinausea drugs have many different mechanisms of action.
- Most antiemetics work by blocking one of the vomiting pathways, thus blocking the stimulus that induces vomiting.
- Specific indications for antiemetics vary per class.
- General use for each type of antiemetic is the prevention and reduction of nausea and vomiting.
Mechanism of Action and Other Indications
- Anticholinergic drugs (acetylcholine [ACh] blockers):
- Bind to and block ACh receptors in the inner ear labyrinth.
- Block transmission of nauseating stimuli to CTZ and from the reticular formation to the VC.
- Scopolamine (Transderm-Scop, Scopace) is a 72-hour transdermal patch that releases 1 mg.
- Antihistamine drugs (H₁ receptor blockers):
- Inhibit ACh by binding to H₁ receptors.
- They prevent cholinergic stimulation in vestibular and reticular areas, thus preventing nausea and vomiting.
- Also used for motion sickness, nonproductive cough, allergy symptoms, and sedation.
- Examples:
- Dimenhydrinate (Dramamine)
- Diphenhydramine (Benadryl)
- Meclizine (Antivert)
- Hydroxyzine (Vistaril): Give PO or IM only, never IV
- Antidopaminergic drugs:
- Block dopamine receptors in the CTZ.
- They are used for psychotic disorders and intractable hiccups.
- Prochlorperazine (Compazine)
- Promethazine (Phenergan): PO & IM routes preferred; IV route common, not preferred, never administer via intraarterial route
- Amisulpride (Barhemsys)--newest
- Droperidol: Use is controversial because of associated cardiac dysrhythmias and is banned in some facilities
- Neurokinin receptor antagonists:
- Inhibit substance P/neurokinin 1 receptors in the brainstem.
- Used in conjunction with serotonin blockers and glucocorticoids.
- Use augments the serotonin blockers and glucocorticoids to inhibit acute and delayed phases of chemotherapy-induced emesis.
- Aprepitant, Fosaprepitant, Rolapitant
- Prokinetic drugs:
- Block dopamine receptors in the CTZ.
- Desensitize the CTZ to impulses it receives from the gastrointestinal (GI) tract.
- Stimulate peristalsis in the GI tract, enhancing emptying of stomach contents.
- Also used for gastroesophageal reflux disease and delayed gastric emptying.
- Metoclopramide (Reglan): Long-term use may cause irreversible tardive dyskinesia
- Serotonin blockers:
- Block serotonin receptors in the GI tract, CTZ, and VC.
- Used for nausea and vomiting in patients receiving chemotherapy and for postoperative nausea and vomiting.
- Dolasetron (Anzemet)
- Granisetron (Kytril)
- Ondansetron (Zofran)
- Palonosetron (Aloxi)
- Tetrahydrocannabinoids:
- Major psychoactive substance in marijuana.
- Inhibitory effects on reticular formation, thalamus, and cerebral cortex.
- Alter mood and body's perception of its surroundings to help relieve nausea and vomiting.
- Dronabinol (Marinol): Used for nausea and vomiting associated with chemotherapy and for anorexia associated with weight loss in AIDS patients.
Miscellaneous Antinausea Drugs
- Phosphorated carbohydrate solution (Emetrol):
- Mint-flavored oral solution used off label for treatment of morning sickness.
Herbal Products: Ginger
- Used for nausea and vomiting, including that caused by chemotherapy, morning sickness, and motion sickness.
- Adverse effects:
- Anorexia, nausea and vomiting, and skin reactions.
- Drug interactions:
- May increase absorption of oral medications or increase bleeding risk with anticoagulants.
Adverse Effects of Antiemetics
- Vary according to the drug used.
- Stem from their nonselective blockade of various receptors.
Nursing Implications for Antiemetics
- Assess complete nausea and vomiting history, including precipitating factors.
- Assess current medications and for contraindications and potential drug interactions.
- Many antiemetics cause severe drowsiness; warn patients about driving or performing any hazardous tasks.
- Taking antiemetics with alcohol may cause severe central nervous system depression.
- Teach patients to change positions slowly to avoid hypotensive effects.
- For chemotherapy, antiemetics are often given 30 to 60 minutes before chemotherapy begins.
- Monitor for therapeutic and adverse effects.
- Scopolamine should only be administered after careful assessment of the patient's health and medication history; it is contraindicated in patients with narrow-angle glaucoma.
- Metoclopramide is a prokinetic drug that promotes the movement of substances through the GI tract and increases GI motility, which is helpful in preventing aspiration in those receiving tube feedings.
- Dronabinol is used as an appetite stimulant in patients who are experiencing nutritional wasting caused by cancer and cancer treatment.
- Phosphorated carbohydrate solution (Emetrol) is a mint-flavored, pleasant-tasting oral solution used to relieve nausea; it works by direct local action on the walls of the GI tract, where it reduces cramping caused by excessive smooth muscle contraction and can be used to control mild cases of nausea and vomiting.
- Serotonin blockers can cause a prolonged QTc interval.
- The use of ginger can increase the absorption of all oral medications and may theoretically increase the bleeding risk with anticoagulants (e.g., warfarin [Coumadin]) or antiplatelet drugs (e.g., clopidogrel [Plavix]).
Studying That Suits You
Use AI to generate personalized quizzes and flashcards to suit your learning preferences.